Você está na página 1de 363

Electrical Engineering Pre-Board Compilations

(2012-2018)
\
AUGUST 2012
1. A coil has a 50 ohms resistance and a 70 ohms reactance. A capacitor is
connected in parallel to produce resonance. The source voltage is 120 V. What is
the power drawn by the circuit?
A. 162 W B. 132 W C. 97 W D. 52 W

2. Two electrons in a vacuum experience a force of 2 x 10 -15 N. How far apart are
the electrons?
A. 3.4 x 10-7m B. 9.6 x 10-7m C. 2.84 x 10-12m D. 5.05 x 10-12m

3. The pitch factor for third harmonic in an alternator having 18 slots per pole and
coil span 5/6 of pole pitch is _.
A. 0.707 B. 0.259 C. 0.966 D. 1

4. When two capacitors of reactances 10ohms and 20 ohms are connected in


parallel across a 100 V supply, total VAR drawn will be _.
A. 500 B. 2 C. 1,500 D. 15

5. Overvoltage relay device function number.


A. 64 B. 56 C. 60 D. 59

6. A test used to determine the equivalent resistance of an induction motor is _.


A. No-load Test B. Short-circuit Test C. Blocked-rotor Test D. Polarity Test

7. A transformer has 4% resistance and 6% reactance drop. What is the voltage


regulation at full load 0.8 pf lagging?
A. 6.0% B. 6.4% C. 6.8% D. 7.2%
8. The unit of retentivity is
A. Ampere turn B. ampere turn/weber C. ampere turn/meter D. dimensionless

9. Three 10 ohm resistances are connected delta on a balanced three phase


source. If the equation of the phase Van = 120 sin ωt, what is the equation of the
phase a line current?
A. 36 sinωt C 11 22 sin ωt 30
B. 13 15 sin ωt - 30) D 11 88 sin ωt -56.6)

10. A 250/6,600 V, 50 Hz single – phase transformer has a core of cross section 600
cm2 and the flux density in it is 0.707 Tesla. Determine the number of turns in the
primary winding.
A. 27 B. 19 C. 23 D. 17

11. In running condition the rotor reactance of 3Φ induction motor is proportional to


A. induced emf B. slip C. supply voltage D. rotor resistance

12. The ratio of the total flux to useful flux is called _.


A. Mutual flux B. Flux Factor C. Leakage Factor D. Magnetic Factor
REGISTERED ELECTRICAL ENGINEERS PRE-BOARD EXAMINATION
AUGUST 2012
PROFESSIONAL ELECTRICAL ENGINEERING SUBJECTS

13. An electric motor converted 700 W of electrical input into work at 58.186 kg-m/s.
The speed was 1750 rpm. Find the kg-m of driven torque.
A. 0.183 B. 0.381 C. 0.138 D. 0.318

14. What is the annual capacity factor of the plant if the annual energy produced in a
150 MW power plant is 500 x 106 kWhr?
A. 56.785% B. 34.44% C. 44.04% D. 38.05%

15. With reduction of load on alternator


A. the frequency decreases C. the frequency oscillates
B. the frequency increases D. the frequency remains the same

16. What is the quality factor Q of an RLC series circuit having R = 5 ohms, L = 10
mH and C = 100μF?
A. 2 B. 5 C. 10 D. 20

17. A load of 10, 000 kVA, 80% lagging is connected to a 13, 200 V line. How much
capacitive reactive power is needed to correct the power factor to 97% lagging?
A. 3, 995 kVAR B. 3, 138 kVAR C. 5, 156 kVAR D. 2, 547 kVAR

18. Two No. 8 copper conductors are placed 15 cm apart with a GMR of 1.27 x 10 -3
m. If the length of the line is 4 km, find the value of the line inductance.
A. 5.78 mH B. 6.85 mH C. 7.63 mH D. 9.65 mH

19. The process by which metallic parts are demagnetized is called


A. degreasing B. degaussing C. degrading D. demagnetizing

20. The gas inside the inner discharge tube of a sodium vapor lamp is
A. Argon B. Neon C. Nitrogen D. Helium

21. The following are the symmetrical components of phase B current:


Positive Sequence Component = 10 cis 45
Negative Sequence Component = 20 cis -30
Zero Sequence Component = 0.5 + j0.9
Determine the positive sequence component of Phase A.
A. 20 cis -195 B 10 cis 195 is - D 10 cis -30

22. What is the force between two wires carrying currents 10 and 50 A each? The
lengths of the wires are 50 cm and are separated by 5 cm distance.
A. 0.005 N B. 0.002 N C. 0.001 N D. 0.004 N

23. What is the nature of the current flowing in the armature of DC machine?
A. pulsating dc B. pure dc C. rectified dc D. alternating current
REGISTERED ELECTRICAL ENGINEERS PRE-BOARD EXAMINATION
AUGUST 2012
PROFESSIONAL ELECTRICAL ENGINEERING SUBJECTS

24. Which of the following frequencies has the longest period?


A. 1 Hz B. 10 Hz C. 1 kH D. 100 kHz

25. Two coils of inductance 8 H and 10 H are connected in parallel. If their mutual
inductance is 4H, determine the equivalent inductance of the combination if
mutual inductance assists the self- inductance.
A. 3.46 H B. 2.46 H C. 4.6 H D. 6.4 H

26. The voltage applied to the stator of a three phase, 4 pole induction motor has a
frequency of 50 Hz. The frequency of the emf induced in the rotor is 1.5 Hz.
Determine the speed at which motor is running.
A. 1, 450 rpm B. 1,455 rpm C. 1, 460 rpm D. 1,465 rpm

27. If one of the resistors in a parallel circuit is removed, what happens to the total
resistance?
A. Decreases B. increases C. exactly doubles D. remains constant

28. What is the purpose of using oil in the transformer?


A. insulation B. cooling C. cooling and insulation D. lubrication

29. An air capacitor consists of two parallel square plates of 60 cm side. When the
two plates are 2 mm apart the capacitor is charged to a voltage of 240 V.
Determine the work done in separating the plates from 2 mm to 5 mm.
A. 27 4 μJ B 24 75 μJ C 25 74 μJ D 42 75 μJ

30. Skin effect is proportional to


A. ( diameter of conductor )2 C. ( diameter of conductor )3
B. diameter of conductor D. ( diameter of conductor )1/2

31. A balanced delta connected load of ( 4 + j3 ) ohms per phase , 230 V supply.
Find the total kVA.
A. 19.04 kVA B. 37.01 kVA C. 31.74 kVA D. 25.39 kVA

32. A 400 V, 10 kW series motor drives a fan when running at 800 rpm. The motor
draws 50 A from the supply. The resistances of the armature and series field are
0.2 ohms and 0.1 ohms, respectively. Determine the electromagnetic torque
developed by the motor.
A. 215 N-m B. 220 N-m C. 225 N-m D. 230 N-

33. The three primary colors in the chrominance signal of a color TV are
A. red, green, orange C. blue, green, magenta
B. red, green, blue D. yellow, green, cyan
REGISTERED ELECTRICAL ENGINEERS PRE-BOARD EXAMINATION
AUGUST 2012
PROFESSIONAL ELECTRICAL ENGINEERING SUBJECTS

34. An alternator running in parallel with the other alternators all having automatic
voltage regulators is to be taken off the bus. The usual procedure before opening
the switch is to
A. reduce alternator excitation C. reduce the power fed to the prime
mover
B. increase alternator excitation D. do nothing

35. In a 4 pole, 35 slot, dc armature, 180 electrical degrees coil span will be obtained
when coils occupy _ slots.
A. 1 and 10 B. 1 and 9 C. 2 and 11 D. 3 and 12

36. A circuit which has ω0 = 106 rad/sec, C = 10 pF and Q = 100, must have a
resistance of _ kῼ.
A. 10 B. 5 C. 1 D. 100

37. A lamp produces an illumination of 10 lux at a point 10 m directly below it. What
is its luminous intensity?
A. 100 Cd B. 10 Cd C. 10, 000 Cd D. 1, 000 Cd

38. A lead acid battery having a 100 Ah capacity is 25% charged. After being
charged at 10 A for 5 hours, it will be approximately _ charged.
A. ½ B. ¾ C. fully D. over

39. Four 100 W, 100 V heaters are connected in series across a 600 volt circuit. The
total heat given off by the four heaters will be _ watts.
A. 900 B. 800 C. 400 D. 2000

40. Which one of the following is not one of the relays of transmission line?
A. Phase comparison relay C. Distance relay
B. Negative sequence relay D. Ground relay

41. In an electric system the available short circuit MVA is 600 MVA. What is the
Thevenin’s equivalent reactance at 34 5 kV?
A. 1.984 ῼ B. 1.587 ῼ C. 2.561 ῼ D. 0.046 ῼ

42. The area of hysteresis loop is the measure of


A. permittivity B. permeance C. energy loss per cycle D. magnetic flux
43. If the secondary terminals of a 5:1 stepdown transformer is connected to the
primary of a 2:1 stepdown transformer then the total stepdown ratio of both
transformers is
A. 2.5 : 1 B. 3 : 1 C. 10 : 1 D. 7 : 1

44. Slip rings are made of


A. Steel B. Copper C. Bakelite D. Mica
REGISTERED ELECTRICAL ENGINEERS PRE-BOARD EXAMINATION
AUGUST 2012
PROFESSIONAL ELECTRICAL ENGINEERING SUBJECTS

45. In a winding of a 3Φ, 6 pole induction motor, the space angle between two
consecutive phases is
A. 600 mech. B. 1200 mech. C. 1800 mech. D. 400 mech.

46. The tailwater and the headwater of a hydro- electric power plant are 150 m and
200 m, respectively. What is the water power if the flow is 15 m 3/sec and a head
loss of 10% of the gross head?
A. 4, 621.56 kW B. 7, 621.65 kW C. 5, 621.76 kW D. 6, 621.75 kW

47. A 2,000 V/ 400 V single phase transformer has an equivalent resistance of 0.03
pu and an equivalent reactance of 0.08 pu. Find the full load voltage regulation at
0.8 pf lagging if the primary voltage is 1,500 V.
A. 6.6% B. 6.8% C. 7% D. 7.2 %

48. Determine the value of R for which power consumed in it is maximum when it is
connected in series with a coil of resistance 3 ῼ and reactance 20 ῼ across 230
V, 60 Hz AC supply.
A. 20 ῼ B. 3 ῼ C. 6.67 ῼ D. 20.22 ῼ

49. Moving coil instruments are used


A. In ac circuits only C. Both in ac and dc circuits
B. In dc circuits only D. For measuring voltage only

50. The efficiency of thermal plant is approximately _.


A. 10% B. 30% C. 60% D. 80%

51. Merz – Price protection is used on


A. Substations B. Capacitor Banks C. induction motors D. Generator

52. A balanced delta connected load draws 100 A line current from a balanced 3
phase supply. Determine the zero sequence components of the currents.
A. -50 – j86.6 B. 50 – j86.6 C. 0 D. 86.6 + j50

53. Impedance z1 = 10 + j15 ohms and another impedance z2 = 6 – j8 ohms are


connected in parallel across a 60 Hz source. Find the total power in watts if the
total current is 15 A.
A. 1,845 W B. 2,176 W C. 3,188 W D. 2,380 W

54. A magnetizing force of 1000 AT/m will produce a flux density of _ in air.
A. 0.63 mWb/m2 B. 0.63 Wb/m3 C. 1.257 mWb/m3 D. 1.257 Wb/m3
REGISTERED ELECTRICAL ENGINEERS PRE-BOARD EXAMINATION
AUGUST 2012
PROFESSIONAL ELECTRICAL ENGINEERING SUBJECTS

55. A 13 8 kV distribution line has a Thevenin’s equivalent impedance 2 63% If a


phase to phase fault occurs between phase B and C, determine the fault current
at 10 MVA base.
A. 13,000 A B. 7,950 A C. 5,300 A D. 15,900 A

56. A transmission line has two aluminum conductors in bundle. The self GMD of
each conductor is 8.90 mm and the distance between the conductors and phase
is 40 cm and 9 m, respectively. What is the inductance of the line per phase?
A. 2 μH/m B 4 μH/m μH/m D 3 μH/m

57. The greater percentage of materials are _ .


A. Diamagnetic B. Paramagnetic C. Ferromagnetic D. Non- magnetic

58. The electric potential at appoint in air due to a charge is 21 V. If air is replaced by
a medium of relative permittivity of 3, then the electric potential at that point will
be _ .
A. 63 V B. 7 V C. 21 V D. 42 V

59. The greatest percentage of power loss in dc motor is due to


A. Windage loss B. Core Loss C. Copper Loss D. Friction Loss

60. A galvanometer with a 20 ohms coil resistance has a full scale deflection of 10
mA. A 0.02 ohm is placed across the meter to increase its rated capacity. What is
the new full scale current for the meter?
A. 100.1 A B. 1.01 A C. 10.10 A D. 10.01 A

61. A 3Φ, 6 pole, 50 Hz induction motor develops 3 73 kW at 960 rpm. What will be
the stator input if the stator loss is 280 watts?
A. 4,615 W B. 4,165 W C. 3,855 W D. 4,516 W

62. Calculate the average voltage generated in a six turn full pitch coil of a 60 cycle
alternator if the flux per pole is 7.2 x 10 5 maxwells.
A. 4.32 V B. 23.4 V C. 17.30 V D. 10.37 V

63. The charge of a hole is _.


A. Zero C. equal to that of an electron
B. Equal to that of a proton D. equal to that of a netron

64. Transient current in an RLC circuit is oscillatory when


A. R = 0 B. R = 2 sq.rt. L/C C. R > 2 sq.rt. L/C D. R < 2 sq.rt. L/C

65. Three units of 1:5 transformers are connected in Δ/Y to supply a 3 phase load
from a 480 V, 3 phase source. The line voltage on the load side is _ volts.
REGISTERED ELECTRICAL ENGINEERS PRE-BOARD EXAMINATION
AUGUST 2012
PROFESSIONAL ELECTRICAL ENGINEERING SUBJECTS

A. 2,400 V B. 832 V C. 4,160 V D. 1.386 V

66. As load pf of an alternator becomes more leading, the value of the generated
voltage required to give rated terminal voltage _.
A. Decreases B. Increases C. Remains constant D. Varies with rotor speed

67. This device is used to reflect back surges into the line.
A. Deion gaps B. Overhead Ground C. Choke Coil D. Arc Horns

68. Some of the arrangement for switch/panel boards are as follows, except _.
A. Vertical B Linear C. Semi – Circular D. Truck Type

69. A 220 V, 3 phase, 4 wire abc system supplies unbalanced wye connected
resistances of Ra = 10 ohms, Rb = 15 ohms, and Rc = 20 ohms. Determine the
total power of the system.
A. 3, 495 W B. 3,385 W C. 3,560 W D. 3,895 W

70. If a 440 V heater is used on 110 V supply, heat produced by it will be as _ much.
A. ¼ B. 4 times C. 16 times D. 1/16

71. A complex current wave is represented by the equation i = 5 5sin 120π t A


Find its average value in amperes.
A. 4.5 B. 6 C. 5 D. 4
72. The fuse used for the overhead primary distributors is
A. Fast acting B. Dual Element C. Expulsion D. Cartridge

73. A transformer has a maximum efficiency of 95% at a load of 100 kW. What is the
constant loss of the transformer?
A. 6. 32 kW B. 3.26 kW C. 3.62 kW D. 2. 63 kW
74. A string of four insulators has a self-capacitance equal to 5 times of pin to earth
capacitance. Find the string efficiency.
A. 64.86% B. 66.48% C. 68.64% D. 84.66%

75. Porcelain insulators are protected against creepage of surge current by the use
of _.
A. Surge gaps B. Arcing Horns C. Ground Conductors D. surge arrester

76. An amplifier works best into an impedance of 500 ohms but is required to drive a
speaker of 20 ohms impedance. The primary to secondary turn ratio of the
matching transformer should be
A. 1/5 B. 25 C. 5 D. 1/25

77. The rms value of the complex voltage given by v = 16 sq.rt 2 sin ωt 12 sq rt 2
sin 3ωt is
REGISTERED ELECTRICAL ENGINEERS PRE-BOARD EXAMINATION
AUGUST 2012
PROFESSIONAL ELECTRICAL ENGINEERING SUBJECTS

A. 20 sqrt 2 V B. 20 V C. 28 sqrt 2 V D. 192 V

78. A uniform 2 wire dc distributor 500 meters long is loaded with 0.4 ampere/meter
and is fed at one end. If the maximum permissible voltage drop per wire is not to
exceed 10 V, find the cross-sectional area of the distributor Take ρ = 1 7 x 10-6
ῼ-cm.
A. 1.0 cm2 B. 1.2 cm2 C. 1.5 cm2 D. 1.7 cm2

79. This is a lighting system wherein the percentage of upward and downward
brightness is the same.
A. General Diffuse B. Direct C. Indirect D. Semidirect

80. A coil having a resistance of 10 ohms and an inductance of 15 H is connected


across a dc voltage of 150 V. When the supply is switched off, find the time for
the current to drop to 9 A.
A. 0.667 sec B. 0.607 sec C. 0.706 sec D. 0.766 sec

81. An iron ring 100 cm mean circumference is made from around iron of cross
section 10-3 m2. Its relative permeability is 500. If it is wound with 250 turns, what
will be required to produce a flux of 2 x 10-3 Wb?
A. 21.47 A B. 14.27 A C. 12.74 A D. 17.42 A
82. An 8 pole lap wound dc generator has 1000 conductors in its armature. The flux
produced per pole is 20 mWb and emf generated is 400 v. What is the speed of
the machine?
A. 1160 rpm B. 1200 rpm C. 1240 rpm D. 1300 rpm

83. Power in a balanced 3 phase system is measured by two wattmeter method and
it is found that the ratio of the two wattmeter readings is 2 is to 1. What is the
power factor of the system?
A. 0.9 B. 0.82 C. 0.866 D. 0.707

84. One ampere-turn is equivalent to _ gilberts.


A. 1.16 B. 1.26 C. 1.36 D. 1.46

85. What value of R is needed with 0 05 μF capacitor for an RC time constant of 0 02


sec?
A. 400 ῼ B. 400 kῼ C. 400 Mῼ D. 400 Gῼ

86. Which motor is used to start heavy loads?


A. Series C. Cumulative Compound
B. Shunt D. Differential Compound

87. The capacitor in a capacitor start induction run ac motor is connected in series
with _ winding?
REGISTERED ELECTRICAL ENGINEERS PRE-BOARD EXAMINATION
AUGUST 2012
PROFESSIONAL ELECTRICAL ENGINEERING SUBJECTS

A. Compensating B. Squirrel Cage C. Starting D. Running

88. A 150 MVA, 13.8 kV, Y- connected, 3Φ, 60 Hz alternator is operating at rated
voltage and at no load when a three phase fault occurs at its terminals. If the
subtransient reactance of the alternator is j0.2 pu , calculate the current at the
instant the fault occurs.
A. 10 460 A B. 18 117 A C. 6 040 A D. 31 380 A

89. What is the total energy generated by a station in kWhr in a year if its maximum
demand and annual load factor is 150 kW and 45%, respectively?
A. 513,900 kWhr B. 531,900 kWhr C. 593,100 kWhr D. 591,300 kWhr

90. Power circuit breakers can be classified according to operation, some of these
are, except
A. Inclined Break B. Horizontal Break C. Gravity Open D. Gravity Close

91. The lowest KAIC available commercially for miniature circuit breakers is
A. 3 B. 5 C. 10 D. 14
92. A 100 km transmission line has a 1000 ohms shunt capacitive reactance. What is
the capacitive reactance per km?
A. 1000 ῼ B. 10 ῼ C. 100 000 ῼ D. 100 ῼ

93. The shaft power of a shunt motor is 7.8 HP. Its armature draws 50 A from 120 V.
The field winding draws 1.2 A. What is the efficiency of the motor?
A. 94.7% B. 96.5% C. 93% D. 98.2%

94. This is the least prevalent shunt fault among transmission and distribution lines.
A. LG B. LL C. LLG D. LLL

95. In what year was the first 500 kV line in the world became operational?
A. 1936 B. 1953 C. 1965 D. 1969

96. A single phase transformer is rated 110/440 V, 2.5 kVA. Leakage reactance
measured from the low tension side is 0.06 ῼ. What is the leakage reactance in
per unit?
A. 0.0124 B. 0.0142 C. 0.0241 D. 0.0421

97. The rms value of a half wave rectified current is 10 A. Its value for full wave
rectification would be _ amperes.
A. 20 B. 14.14 C 20/π D 40/π

98. If starting voltage of a SCIM is reduced to 50% of its rated vlue, torque developed
is reduced by _ percent of its full load value.
A. 57.7 B. 50 C. 25 D. 75
REGISTERED ELECTRICAL ENGINEERS PRE-BOARD EXAMINATION
AUGUST 2012
PROFESSIONAL ELECTRICAL ENGINEERING SUBJECTS

99. A standard candle emits a total luminous flux of _ lumens.


A. 157.90 B.78.96 C. 25.14 D. 12.57

100. A generator of 50 MVA, 4kV, Xd” = 25% and transformer of 50 MVA, 4 2


kV, delta/115 kV wye grounded, operating on 3.8 kV tap where X T” = 10% are to
be connected into single equivalent reactance. What is the single equivalent
reactance on a 100 MVA, 110 kV base?
A. j0.6039 pu B. j0.6667 pu C. j0.8241 pu D. j0.8663 pu

SOLUTIONS:
1. YT = GL
RL RL 50
GL = 2
= 2 2
= 2
ZRL RL XL 50 702

GL = 0.00676 mho

PT = ET2VT2 = (1202)(0.00676) = 97.344 W

2
kQ1 Q2 kQ1 Q2 (9x109 )(1 609x10-19 )
=√ -7
2. F= 2 ;d=√ = 4 m
d F 2x10 -15

3. kp=sin (90)x pitch


18 slots/pole
5/6 pole pitch
5 1
kp= sin (90x x x18* = 7 7
6 2

(10) 20
4. X= = -j6 667
10 20
1002
Qr = =1499 23 VARS
j6 667

5. D. 59
REGISTERED ELECTRICAL ENGINEERS PRE-BOARD EXAMINATION
AUGUST 2012
PROFESSIONAL ELECTRICAL ENGINEERING SUBJECTS

6. C. Blocked-rotor Test

7. Rpu = .04
Xpu = .06

@
2 2
VR=√(Rpu pf) Xpu rf 1

2
=√( 04 8)2 06 6 -1

%VR= 6.97 % = 6.8%

8. B. ampere turn/weber

AB 100 10
9. ZY= A = =
B C 30 3
Van = 120 sinωt

Van 120
Ian = = = sinωt
Zy 10
3
10. 250 / 6 600 V, 50 Hz
A= 600 cm2

β=0 707

E= 4.44fɸN

ɸ=βmA

E1 =4 44fβmAN1

200
N1 = =2 27
(4 44)(50) 707 600x10 4

11. B. slip

12. C. Leakage Factor

9 55 Pd 9 55 58 186
13. T= = = -m
N 1750
REGISTERED ELECTRICAL ENGINEERS PRE-BOARD EXAMINATION
AUGUST 2012
PROFESSIONAL ELECTRICAL ENGINEERING SUBJECTS

Annual Energy in kWhr 500 x106


14. PCF= Installed Capacity in kW x 8760 = x 100%=
150 x103 8760
15. A. the frequency increases

1 L 1 10mH
16. Qfactor = R √C = 5 √100μF = 2
17. Q = PL tanθold –tanθnew) = (10000)(.8)(tan36.87 – tan 14.07) = 3995 kVAR

d 15
18. L=4x10-7 ln (GMR) =4x10-7 ln ( -3
) = (1 9086 x10-3 ) 4000m
1 27 x 10
=7 mH

19. B. degaussing
20. B. Neon
21. F = (2x10-7)(I1)(I2)(1/D)
F = (2x10-7)(10)(50)(50/5) = .001N

22. C. rectified dc
23. 1 Hz
L1 L2 -M2 (8 x 10)-42
24. La = = = 4H
L1 L2 -2M 8 10 -(2) 4

120 x 50
25. Ns = =1500
4
4 1500 Nr
Ns =
120
Nr = 1455 rpm

26. B. increases
27. C. cooling and insulation
28.
29. ( diameter of conductor )2
V
30. I= Z = 46 angle - 36 83
S=(462)(5)(3) = 31.74 kVA

31. Eb= VL –IP ( Ra + Rs) = 400 – 50( .1+.2) = 385


IL = Ia =Ise

Pd = EbIa = 385 x 50 = 19 250 W


REGISTERED ELECTRICAL ENGINEERS PRE-BOARD EXAMINATION
AUGUST 2012
PROFESSIONAL ELECTRICAL ENGINEERING SUBJECTS

9 55Pd 9 55 19250
T= = =2 m
N 800
32. red, green, blue
33. C. reduce the power fed to the prime mover
34. B. 1 and 9
1 1
35. XC = ωC = 6
=100000
10 10X10-12
100000
R= =1000 =
100
36. I =10 lux
D=10 m
L
I= L=( 10 )(102 )=
d2

37. Battery : 100 Ah, 25 % charged = 25 Ah charged


I = 10 A

t= 5 hrs

charged = (10 A) (5 hrs) = 50Ah

charged = 50 +25 =75 Ah

%charged =

38. P=100W
E=100V

Es=600V

1002
R= =100
100

V2 6002
H= = =
4R 4x100
39. C. Negative sequence relay
40. S= 600 kVA
V= 345 kV
REGISTERED ELECTRICAL ENGINEERS PRE-BOARD EXAMINATION
AUGUST 2012
PROFESSIONAL ELECTRICAL ENGINEERING SUBJECTS

X=

41. C. energy loss per cycle


43. 5:1 = 2:1

(5)(2) = 1

10:1

44. B. Copper

45. 2 pole = 360

6 pole = 180

180
m=2x =60 mech
6
46. P=Qwh
h= 200 – 150 =50m

Q=15m3/sec

HL=10%

P= 15x 9810 x 50 = (7357500) – (10%)(7357500) = 6621750

= 6621.75 kW

47. Pf=.8 lag


%R=.03pu

%X= .08pu

Vpu cosθ Xpu sinθ


%VR=(Vpu cosθ Xpu sinθ)
200
( 08)(cos36 87) ( 03) cos36 87
=[( 03)(cos36 87) ( 08)(sin36 87)]
200
7 22

48. R=√32 202 = 2 22


REGISTERED ELECTRICAL ENGINEERS PRE-BOARD EXAMINATION
AUGUST 2012
PROFESSIONAL ELECTRICAL ENGINEERING SUBJECTS

49. B. In dc circuits only


50. B. 30%
51. A.Substations
52. Ia = 100
Iab = 3In

Iab = 3 [(100angle0) + (100angle240) + (100angle120)]

Iab = 0

53. Z1 = 10 + j15 ῼ
Z2 = 6 – j8 ῼ
10 j15 6 j8
ZT =
10 j15 6 j8

ZT = 9.67 – j3 6

PT = (9.67)(152)

PT = 2176. 15 W

54. H=100AT/m

β=μrμoH = (91.26x10-6)(1000) = 1.257mWb/m2

S 10x106
55. IFL-L = VB = =13 776 4 13 000 A
B 13 8 x 103
56. GMD= 8.90 mm
Ds1 = 40 cm

Ds2 = 9m

40
L1=2x10 ln (7100 ) =7 61 x 10 7
8 90
1000

L2=2x10 7 ln ( ) =1 38 x 10 6
8 90
1000

LT = L1 +L2 = 7 61 x 10-7 +1 38 x 10-6 = μH/m

57. B. Paramagnetic
REGISTERED ELECTRICAL ENGINEERS PRE-BOARD EXAMINATION
AUGUST 2012
PROFESSIONAL ELECTRICAL ENGINEERING SUBJECTS

58. V=21V
Q=1.60x10-19

At k=3, V=?

kQ
V=
d

9x109 1 6x10 19
d=
21
d= 6.87x10-11

at k=3

1 6x10 19
V= =6 99 = 7
4π 0 (5) 6 87x10 11

59. C. Copper Loss


60. Pg=20
Rsh= 02

Ig = 10mA

I = Ig x multiplier

Rg Rsh 20 02
multiplier= = 1001
Rsh 02
I = 10 mA x 1001= 10.01 A

61. 3ø, 6pole, 50 Hz, Induction Motor


Pd = 3.73 kW @ Nr= 960 rpm

SL= 280W

Req’d : Ps

3 73x103
Pr = =3885 42W
1 04
Ps = Pr + SL = 3885.42+280 = 4165.42 W
REGISTERED ELECTRICAL ENGINEERS PRE-BOARD EXAMINATION
AUGUST 2012
PROFESSIONAL ELECTRICAL ENGINEERING SUBJECTS

62. N=6 turns


f= 60 Hz

ø= 7.2x 105 maxwells

Eave = 4føN = (4)(60)(7.2x105)(x10-8) = 10.37 V

63. Equal to that of a proton


64. D. R < 2 sq.rt. L/C
65. a= 1:5
Δ/Y

VP=480 V

Vs = ?

Ep 1
=
ES/Q 5

ES/Q = (5)(4800)

ES/L-L = √3 2400 = 4156 9= 4160 V

66. A. Decreases
67. C. Choke Coil
68. A. Vertical
2202 2202 2202
69. PT = √3 10 =6054 48
√3 15 √3 20
6054 48
PT = = 4
√3
70. VH = 440 V
VS = 110 V

Req’d: Heat Produced HH)


VH H
= HH
VS S

440HS
HH = =4HS
110
4 times

71. i = 5 5sin(120π)t A
REGISTERED ELECTRICAL ENGINEERS PRE-BOARD EXAMINATION
AUGUST 2012
PROFESSIONAL ELECTRICAL ENGINEERING SUBJECTS

IDC IAC 5 5
iave = = =
2 2
72. C. Expulsion
73. ɳmax = .95
PL= 100 kW

@ɳmax :Pcu = Pco

Req’d: Pco

100x103
Smax 100x103
ɳmax = = 95 =2
Smax Pco 2

74. String efficiency =ɳst


v1 V2 V3 Vn
ɳst = ;k=1/5
n xVn
V2 = V1 (1 + k) = 1.2 V1
V3 = V1 ( 1 +3k + k2) = 1.64 V1
V4 = V1 (1 + 6k + 5k2 + k3) = 2.408 V1
V1 1 2V1 1 64V1 2 408V1
ɳST = x100
4x2 408V1
ɳST = 64.867%

75. B. Arcing Horns


Z N1 2
76. Z1 = =a2
2 N2
500 2
=a =
20
77. V = 16√2 sinωt 12√2sin 3ωt
2 2 2 2
Vrms Vrms √(16√2) 12√2
Vrms =√Vdc = =2
2 2 2 2

V 10
78. R= = 100 = 05
I
ρL 1 7 x10 6 50 000cm
R= ; 05=
A A
A= 1.7 cm2.
REGISTERED ELECTRICAL ENGINEERS PRE-BOARD EXAMINATION
AUGUST 2012
PROFESSIONAL ELECTRICAL ENGINEERING SUBJECTS

79. A.General Diffuse


80. R = 10 ohms
L = 15 H
E = 150 V
I=9A
@ switched off, t=infinity
E 150
I= = =15 A
R 10
@ I= 9 A, t=?
I = 15 – 9 = 6 A
E R 10
I= (1 e L * ;6=15 1 e 15t
(t)
R
t = 0.766 s
Nɸ μr μo AN2 500 4πx10-7 10-3 2502
81. L= = =
I l 1
1
L= π=0 0393 π
80

Nɸ 250 2x102
I= = = 27
L 1
π
80
PZɸN 60aE 60x8x400
82. E= ;N= = = 2 pm
60a PZɸ 8x1000 20x10-3
83. Two wattmeter reading ratio
2 W1
=
1 W2

√3(w1 -w2 ) √3(2-1)


θ=tan-1 ( ) =tan-1 ( ) =30 ;cosθ=pf=
w1 w2 1 2

1 256 gilberts
84. =1amp turn x = 2 i ts
1 amp turn
t 02
85. R= C = =4
05x10-6
86. A.Series
87. C. Starting
Sbase 150 x106
88. Ifault = = 3 = 77 7
√3 Vbase Xthpu √3(13 8 x10 ) 2
AL
89. LF= PL ;AL=LFxPL=(150x103 )( 45)= 67500
REGISTERED ELECTRICAL ENGINEERS PRE-BOARD EXAMINATION
AUGUST 2012
PROFESSIONAL ELECTRICAL ENGINEERING SUBJECTS

Since annual average load therefore

Hour = 8760

AL= 67500 X 8760 = 591 300 kWhr

90. A. Inclined Break


91. B. 5
92. XC/RM= 100 x 1000 =
93. IL = Ia + Ish
= 50 + 1.2 = 51.2 A

Po 7 8 x 746
ɳ= x 100%= x 100= 4 7
Pi 51 2 x120

94. D. LLL
95. C. 1965
Vbase 2 1102
96. Zbase = = = 4 84
Sbase 440
Zact 06
Zpu = = = 24
Zbase 9 84
10
97. Fullwave Amax= = 4 4
707
98. VST1 = 100%
VST2 = 50%
T 1 V1 2
=( *
T 2 V2
T1 1 2
=( *
T2 5
T2 = .25T1
T2 = 100 – 25 = 75%

99.
( )
( ( ))

100. C. j0.8241 pu
MARCH 2013
REGISTERED ELECTRICAL ENGINEERS PRE-BOARD EXAMINATION
MARCH 2013
PROFESSIONAL ELECTRICAL ENGINEERING SUBJECTS

1. A three-phase, 60 Hz wye-connected wound rotor synchronous generator


rated 10 KVA, 230 V has a synchronous reactance of 1.2 ohm per phase and
an armature resistance of 0.5 ohm per phase. Calculate the percent
regulation at full load with 0.8 lagging power factor.
A.18.3% B.20.8% C.21.8% D.22.5%

( √ )
ZL = = 5 29Ω
EP = (0.5+j1.2) +
( )
(230V/√ )
EP= 161.75V
ZL

( √ )
%VR= x 100%
IT=IL=
%VR=21.8%
I T=

2. Farad is equivalent to what unit?


A. Volt-coulomb C. Joule/volt
B. Coulomb/volt D. Joule/coulomb

3. A 115 kV three-phase transposed transmission line is composed of 2-ACSR


336,400 cmil, 30/7 Oriole conductors per phase with horizontal configuration:
D12= 7ft, D23= 7ft, and D13= 14ft. The line spacing is measured from the center of
the bundled conductors. The distance between the conductors of the bundle is 6
inches. The conductors have a diameter of 0.741 inch and GMR of 0.02255 ft.
Find the capacitance per phase for 20 km of the line in μF
A.0.1300 B.0.27189 C.0.2889 D.0.2609

4. Three 90 mH inductors are connected in parallel. Find the total inductance.


A.270mH B.30mH C.45mH D.120mH

LT = = 30mH

5. Which of the following is used in power factor correction?


A. Induction motor C. Synchronous motor
B. Long transmission line D. Voltage regulator
REGISTERED ELECTRICAL ENGINEERS PRE-BOARD EXAMINATION
MARCH 2013
PROFESSIONAL ELECTRICAL ENGINEERING SUBJECTS

6. Two equally charged spheres repel each other with a force of 0.1 kg. If their
centers are 20 cm apart, find the change on each.

A. 2.09x10-6coul C. 2.00x10-6coul
B. 2.09x106coul D. 2.00x106coul

F = (0.1kg) (9.81) = 0.981N

( )
0.981 = ( )
; Q = 2.09x10-6 coul

7. A transformer is rated 1KVA, 220/110V, 60 Hz. Because of an emergency this


transformer has to be used on a 50 Hz system. If the flux density in the
transformer core is to be kept the same as at 60 Hz and 220 V, what is the
kilovolt-ampere rating at 50Hz?

A. 0.871 kVA B.0.890 kVA C.0.833 kVA D. 0.909 kVA

= ; S2 = 0.833KVA

8. The lightning load for a dwelling expressed in terms of a unit load in volt-
ampere per square meter must be at least ________.

A. 24 B. 12 C. 16 D. 28

9. A hermetic motor compressor has a full-load current of 25A. What is the


allowable rating for branch disconnecting means?

A. 28.75 A B. 30 A
C. 31.25 A. D. 37.5 A

10. A metal disk of 30 cm radius rotates at an angular velocity of 1,200 rpm


across a uniform vertical magnetic field of flux density 0.5 Wb/m 2. Calculate the
voltage included between the rim and the center of the disk.

A. 4.4 V B. 4.7 V C. 5.1 V D. 5.8 V


REGISTERED ELECTRICAL ENGINEERS PRE-BOARD EXAMINATION
MARCH 2013
PROFESSIONAL ELECTRICAL ENGINEERING SUBJECTS

V = 0.5(1200)* * = 62.83 m/s

Vave = = 31.42m/s

E = BLV = 0.5 * 0.3 * 31.42 = 4.7V


11. A coil draws 1,875 watts when connected to a 150 V DC source. It consumes 3,072
watts when connected on 240 V, 60 Hz AC source. Find the inductance of the coil.

A.0.0255 H B.0.0153 H C.0.0341 H D.0.0240 H

Solution:
Idc = = 12.5 A Z= = 15 Ω

R= = 12 Ω XL = √ =9Ω

Iac = √ = 16 A L= = 0.0240 H
( )

12. What do you call the interior lighting where the roof trusses or ceiling height is
greater than 7.6 m above floor level.

A.HID C. High mast


B. High Bay D. High pressure sodium

13. Coil 1 of a pair of coupled coils has a continuous current of 5 A, and the
corresponding fluxes Φ11 and Φ12 are 0.2 and 0.4 mWb, respectively. Find the coupling
coefficient k.

A.0.5 B.0.333 C.0.667 D.0.2

Solution:

Φ1 = 0.2 + 0.4 = 0.6 mWb K= = = 0.667


REGISTERED ELECTRICAL ENGINEERS PRE-BOARD EXAMINATION
MARCH 2013
PROFESSIONAL ELECTRICAL ENGINEERING SUBJECTS

14. Which of the following is a vector quantity?

A. Magnetic Flux C. Magnetic reluctance


B. Magnetic flux density D. Magnetic field intensity

15. A certain copper wire has a resistance of 0.5 ohms when the length is 10m. What is
the diameter in mils?

A.57.3 B.33 C.29.4 D.26

( )( )
R= 0.5 =

L = 10m ( ) = 32.8ft A = 680.27cm; d =√ = 26mils

= 10 37 Ω -

16. An unshaded lamp is placed at a distance r1 above the ground and produces an
illumination E1 at a point on the ground directly below the lamp. At what angle must the
lamp be aimed on the ground to produce an illumination one-half of E1?

A.45 degrees C.30 degrees


B.37.5 degrees D.27.5 degrees

17. Two equal resistors wen connected in series across a supply draws a total of 20
watts. What is the total power drawn when two resistors are connected in parallel
across the same supply?
A.20 W B.40 W C.60 W D.80

Pp = n2 Ps ; n = 2 ; Ps = 20 watts ; Pp = (2)2 (20) = 80W

18. The Electrical Safety Month of the year is ______?

A. June B. April C. February D. May


REGISTERED ELECTRICAL ENGINEERS PRE-BOARD EXAMINATION
MARCH 2013
PROFESSIONAL ELECTRICAL ENGINEERING SUBJECTS

19. The area of one plate of a two plate mica capacitor is 0.0025 m 2 and the separation
between plates is 0.02 m. If the dielectric constant of mica is 5, what is the capacitance
of the capacitor?

A.5.53 μF B.5.53 pF C 7 74 μF D.7.74 pF

( )( )( )
C= C= (2-1)

= 8.85 pF/m C = 5.53pF

20. Which of the following produces a reactive power?

A. Incandescent lamp C. Flat iron


B. Induction motor D. Heater
21. What capacitance must be placed in series with an inductance of 0.05 H so that
at 100 Hz, the impedance becomes equal to the ohmic resistance?
7μF B 70 7μF C 35 5μF D 87μF
Solution:
Using the formula:

Then derive it to form the equation: 𝐶 = = 7μF
( )( )( )
22. Receptacle outlets shall be located above, but not more than______ mm above
the countertop.
A.300 B.450 C.250 D.600
23. In dwelling units, hallways of __ m or more in length shall have at least one
receptacle outlet.
A. 4 B. 2 C. 5 D. 3
24. A 16-ohm resistor is connected in series with parallel combination of three
resistors, R and 48 ohms. Determine R if the power consumed by the parallel
connected resistor is equal to the power consumed by the 16-ohm resistor.
A. 6 ohms B. 12 ohms C. 24 ohms D. 36 ohms
Solution:
Using the formula: P = I2R
Since: P48R = P16
( )( )
I2 ( = ( )
)
R = 24 Ohms
REGISTERED ELECTRICAL ENGINEERS PRE-BOARD EXAMINATION
MARCH 2013
PROFESSIONAL ELECTRICAL ENGINEERING SUBJECTS

25. The time constant of a series RL circuit Is equal to _____?


A. L/R B.R/L C. RL D. 1/RL

26. A commercial building with an aggregate three-phase load of 170 kVA is to be


served using two single-phase transformers connected in V-V bank. What should
be the minimum size of each transformer in kVA.

A.167 B.75 C.100 D.333

= 100KVA

27. A 240 V to 120 V, 50 Hz step-down transformer is to be operated at 60 Hz. What


is the highest safe input voltage?

A.288 V B.240 V C.200 V D.100 V

V2 = = 288V

28. Heating effect of current has undesirable side effect in

A. Electric oven C. Electric iron


B. Immersion heater D. Vacuum cleaner
29. In an electric system the available short-circuit MVA is 600 MVA. What is the
Thevenin’s equivalent reactance at 34 5 kV?

A.1.14 ohms B.2.29 ohms C.1.98 ohms D.1.32 ohms

( )
X= = 1.98 ohms

30. The net cross section of a transformer is 29 square inches. The maximum flux
density is 11 kilogausses. The frequency is 60 Hz. If there are 1,000 turns in the
primary, determine the primary induced emf.

A.3,780 V B.3,590 V C.3,850 V D.3,945 V

E = (4.44 * 1000 * 60 *11x10-3 * (20(2.54)2)) x10-8


REGISTERED ELECTRICAL ENGINEERS PRE-BOARD EXAMINATION
MARCH 2013
PROFESSIONAL ELECTRICAL ENGINEERING SUBJECTS

E = 3,780V

31. A 15MW, 132 kV, 80%power factor three phase load is to be served by a
transmission line having conductors whose resistance is 0.5 ohm/km. If the
losses on the line shall not exceed 4.5%, how long must this line be?

A. 66 km B. 68 km C. 70 km D. 75 km

( ) ; x= 70 km
( )

32. A change of 5% in the supply voltage of a three-phase induction motor will


produce to its torque a change of approximately _______.

A. 10% B. 5% C. 25% D. 20%

33. In a 3-phase, 4 wire system, the current in line a has I a1=2200 deg A and
Ia2=10060 deg A. the current through the neutral conductor is 300-60 deg.
Find the line current Ib in amperes.

A. 100-300 B. 100120 C. 100-60 D. 300240

34. In two-wattmeter method, the readings of the wattmeter will be identical when

A. Load in one of the phases is zero C. Power factor is 0.5

B. Power Factor in unity D. Neutral is earthed

35. A capacitor of 100 micro farad is connected in series with a resistance of 1000
ohms. The time constant of the circuit is

A. 1 sec B. 0.01 sec C.0.1 sec D. 0.001 sec

T= 1000*100x10-6= 0.1s

36. An electric current can neither be ______nor______.

A. Felt, seen C. Seen, touch any effect

B. Seen, touched D. Produced, felt


REGISTERED ELECTRICAL ENGINEERS PRE-BOARD EXAMINATION
MARCH 2013
PROFESSIONAL ELECTRICAL ENGINEERING SUBJECTS

37. A three-phase 4-pole 50 Hz induction motor has a rotor resistance of 0.020 ohm
per phase and standstill reactance of 0.5 ohm per phase. Calculate the speed at
which the maximum torque is developed.

A.1,500 rpm C. 1,480 rpm

B.1,460 rpm D. 1,440 rpm

38. Two capacitors A and B have capacitances of 50F and 30F, respectively.
When 230V, 50 Hz voltage is applied, find the maximum energy stored assuming
A and B are connected in series.

A. 0.992 J B. 4.232 C. 1,984 J D. 2.116 J


cT= = 18.75F

w = (230)2(18.75x10-6)= 0.92

39. Find the average power in a resistance R=10 ohms, if the current in the Fourier
series form is i=12 sin ὠt + 8 sin ὠt + 3 sin 5 ὠt amperes.

A. 1, 150 W B. 1,027 W C. 1,085 D. 1,203

Irms = √ ( 122+82+32 ) = 10.42A

P= 10.422* 10 = 1085 W

40. In transmission voltage levels, 765kV is categorized as

A. High Voltage C. Ultra High Voltage

B. Extra High Voltage D. Medium Voltage

41. Two parallel long conductors will carry 100 A. If the conductors are separated by
200 m, the force per meter length of each conductor will be_____.

A.10 N B.0.1 N C.1 N D.100 N

Solution:

F = (2x10-7) (100)2 (100) = 0.2/2 = 0.1N


REGISTERED ELECTRICAL ENGINEERS PRE-BOARD EXAMINATION
MARCH 2013
PROFESSIONAL ELECTRICAL ENGINEERING SUBJECTS

42. To a series RLC circuit, a voltage of 10 V is applied. If Q of the coil at resonant


frequency is 20, the voltage across the inductor will be _____.
A.2 V B.50 V C.100 V D.200 V
Solution:
20 = ; VL = 200V

43. For a coil having a span of 2/3 of pole pitch, the coil span factor is

A.0.8 B.0.866 C.0.786 D.0.966

Solution:
Kp = sin (90 * ) = 0.866

44. The potential transformer of the line has a rate of 132,000/66. These are
connected wye-delta. The voltmeter reads 64 V. What is the average of the line?

A.221.7 kV B.112.5 kV C.135.2 kV D.132 kV

Solution:
Vp = 64 ( ) = 128KV * √ = 221.7KV

45. A 50 Hp, 440 V, 3-phase, 60 Hz,6-pole squirrel cage induction motor is operating
at full load and 0.8 p.f. The full load efficiency is 85% and the percentage slip is 5
%. Determine the full load torque.

A.214.55 N-m C. 312.47 N-m


B.206.72 N-m D.323.24 N-m

Solution:
0.85 = = 34875.5W

( )
T= = 292 16 312.47

46. The purposes of a grid system is to

A. Interconnect the power stations


REGISTERED ELECTRICAL ENGINEERS PRE-BOARD EXAMINATION
MARCH 2013
PROFESSIONAL ELECTRICAL ENGINEERING SUBJECTS

B. Supply energy to remote areas


C. Make extra high voltage available to consumers
D. Make distribution of energy at reasonable cost

47. The efficiency of thermal plant is approximately

A.10% B.30% C.60% D.80%

48. What is the half power bandwidth of a parallel resonant circuit which has a
resonant frequency of 3.6 MHz and Q of 218?

A.1.65 kHz B.16.5 MHz C.16.5 kHz D.165 kHz

Solution:
BW = = 16.51kHZ

49. The number of parallel paths in a simplex lap winding is equal to

A. 2 C. Number of pair of poles


B. Number of poles D.1

50. The power consumed by a heating element made of copper wire is 250 W at 220
V and 30 degrees Celsius. What is the power consumed by the same element at
220 V and 100 degrees Celsius? The temperature coefficient of the element at
30 degrees Celsius is 0.004-degree C-1.

A.274 W B.233.33 W C.215.25 W D.196.31 W

Solution:
R1 = = 193.6 ohms

0.004 =

T = 200 degrees Celcius

R2 = 193.6 * = 274.81 ohms


REGISTERED ELECTRICAL ENGINEERS PRE-BOARD EXAMINATION
MARCH 2013
PROFESSIONAL ELECTRICAL ENGINEERING SUBJECTS

51. A generator rated 600 kVA, 2,400 V, 60 Hz, 3-phase, 6-poles and wye-connected
has 10% synchronous reactance. If a three-phase fault occurs at its terminals,
what will be the short-circuit current?

A. 1,428 A B. 1,443 A C. 1,532 A D. 1,435 A

Given: S= 600kVA V= 2400V f= 60Hz P= 6-poles Z1pu= 0.10


Formula: Three Phase Fault Formula, If =

Solution: If = = 1443.38 A
√ ( )( )

52. A 200-Hp 2,200 V, 3-phase star connected synchronous motor has a


synchronous Impedance of 0 3 j3 Ω per phase Determine the induced emf per
phase if the motor works on full-load with and efficiency of 94% and a power
factor of 0.8 leading.

A. 1,354 V B. 1,360 V C. 1,402 V D. 1,522 V

Given: Po= 200Hp x 746W/1Hp Vline = 2200V Z= 0 3 j3 Ω per phase


= 149200W
Pi= (149200) (0.94) p.f. = 0.8 leading
= 140248W
Formula: I= Vp =
√ √

Solution: I= = A
√ ( )( )

Vp = ( )( )=

53. If the load of a wye-connected transformer are:

Ia = 10 cis (-30o) Ib = 12 cis 215o Ic = 15 cis 82o

What is the phase b sequence component?

A.13.4 cis (-32.2o) C. 12.27 cis 208.4o


B. 10.2 cis 240o D. 12.27 cis (-31.6o)

Given: Ia = 10 cis (-30o) Ib = 12 cis 215o Ic = 15 cis 82o


REGISTERED ELECTRICAL ENGINEERS PRE-BOARD EXAMINATION
MARCH 2013
PROFESSIONAL ELECTRICAL ENGINEERING SUBJECTS

Formula: Ia1 = ( ) ( )( ) ( )( )
Ib1 = ( )Ia1
Solution: Ia1 = ( ) ( )( ) ( )( )
Ia1 =
Ib1 =( ) (12.27 )= ( )

54. If the input of the prime mover of an alternator is kept constant but the excitation
is increased then

A. kVA will be leading C. kW will be changed


B. kVA will be lagging D. the power factor of the load remains constant
55. A current of 10 A is flowing in a flexible conductor of length 1.5 m. A force of 15 N
acts on it when it is placed in a uniform field of 2 T. Calculate the angle between
the magnetic field and the direction of the current.

A. 30o B. 45o C. 60o D. 90o

Given: F = 15 N β=2T I = 10A L = 1.5m


Formula: F=
Solution: 15 = (2) (10) (1.5) sin
= sin-1( )
O

56. The useful life of diesel engine in a power plant is expected to be ______ year/s.

A. 5 B. 1 C. 50 D. 15

57. An automatic device that operates at preset values is known as

A. Mercury switch C. relay


B. fuse D. contactor

58. A 240 V dc series motor develops a shaft torque of 200 N-m at 92% efficiency
while running at 600 rpm. Calculate the motor current.

A. 62.09 A B. 52.36 A C. 48.17 A D. 56.91


REGISTERED ELECTRICAL ENGINEERS PRE-BOARD EXAMINATION
MARCH 2013
PROFESSIONAL ELECTRICAL ENGINEERING SUBJECTS

Given: V = 240 V T = 200 N-m n% = .92 N = 600 rpm


Formula: HP = n% = Po/Pi Ia = Po/V
( )( )
Solution: HP =
( ) = 13659.1 W

= 56.91 A

59. A magnetic field copper exists around

A. Copper B. Iron C. Moving charges D. Aluminum

60. A voltage of e = 7.07 sin 50t v is applied to a series circuit containing a resistance
of 4 ohms and an inductance of 60 mH. What is the impedance of the circuit in
ohms?

A. 5 30o Ω B. 5 60o Ω C. 5 53.13o Ω D. 5 36.87o

Given: e = 7.07 sin 50t Ѡ = 50 rad/sec R=4Ω


Formula: XL = ѠL Z = R + j XL
Solution: XL = (50) (60x10-3 = 3 Ω
Z=4+j3Ω =

61. A 230 V, 3-phase, 3-wire system has to balanced 3-phase loads. One, wye
connected has impedance of 6 j8 Ω per phase and the other one, delta
connected has impedance of 24 j18 Ω per phase Find the line current in
amperes.

A. 26.3 B. 23.9 C. 28.2 D. 20.8


Given
V= 230v Zy=6+j8
Z =24+j18
Solution
(3)Z =Z
Z = = 8 + j6
Z = Z // Z
( )( )
Z =
( ) ( )
REGISTERED ELECTRICAL ENGINEERS PRE-BOARD EXAMINATION
MARCH 2013
PROFESSIONAL ELECTRICAL ENGINEERING SUBJECTS

( ) √
` IL = I = =
62. It is desired to deliver 4,000 kW, three-phase at a distance of 25 miles. The load
voltage being 33 kV, 60 Hz and the power factor of the load being 85% lagging.
The reactance of the line per phase is 0.692 ohm per mile. The line loss is 10%
of the power delivered. What is the voltage regulation of the line?

A. 11.2% B. 12.3% C. 13.2% D. 14.8%


Given
P= 4,000kW V= 33kV
f= 60Hz pf=0.8
Solution
Ir=

IR =
√ ( )( )

PLoss = 400, 000 W %VR = )

R=( )( )
= 19.67 Ω % VR = 11.2%
VS = VR + IZ
VS = 21182.4

63. The voltage applied to the stator of a three-phase, 4-pole induction motor has a
frequency of 50 Hz. The frequency of the emf induced in the rotor is 1.5 Hz.
Determine the speed at which motor is running.

A. 1,410 rpm B. 1,425 rpm C. 1,448 rpm D. 1,455 rpm


Given
f=50Hz p=4
Solution
N=
( )
Ns = = 1500 rpm
( )
Fr =
( )
1.5 = Nr = 1455 rpm

64. The applied voltage is increased by 50% and frequency is reduced by 50%. The
maximum core flux density will become
A. 1.5 times B. 3 times C. 5 times D. the same
REGISTERED ELECTRICAL ENGINEERS PRE-BOARD EXAMINATION
MARCH 2013
PROFESSIONAL ELECTRICAL ENGINEERING SUBJECTS

65. Surge impedance of transmission line is given by

A √C/L B √CL √L/ D 1/√LC


66. Which of the following is not a type of the contactor for circuit breakers?

A. Vacuum C. Electro-pneumatic
B. Pneumatic D. Electromagnetic
67. A three-phase motor driven pump at 1,800 rpm is discharging 500 gpm of water
at a head of 25 meters. The pump efficiency is 75%. Determine the horsepower
of the motor.

A. 15 B. 12 C. 14 D. 10
Given
N=1,800rpm L=25m
efficiency=75%

Solution
( )( )( )
Pin = =

Pin = = 14Hp

68. The average weekly energy generated by a thermal plant is 7 x 10 6 kWHR. The
peak load of the plant is 75,000 kW during the week. If the plant has an installed
capacity of 100 MW, solve for the load factor of the plant.

A. 48.92% B. 56.34% C. 50.44% D. 55.55%


Given
W=7x10 kWh P=75,000kW
P=100 MW
Solution
Lf =

Lf = ( )( )

69. A 15 MVA, 34.5 kV / 6.24 kV transformer is connected at an infinite bus. The


percent impedance of the transformer is 2.5%. What is the current at the 34.5 kV
side for a three-phase fault at the 6.24 kV side?
REGISTERED ELECTRICAL ENGINEERS PRE-BOARD EXAMINATION
MARCH 2013
PROFESSIONAL ELECTRICAL ENGINEERING SUBJECTS

A. 25,000 A B. 5,000 A
C. 55,500 A D. 10,000 A
Given
S=15MVA V= 34.5kV/6.24kV
Zpu=0.025
Solution
, If =

If = =
√ ( )( )

70. Charging a lead-acid cell causes the electrolyte to become


A. stronger B. stable C. weaker D. water
71. A coil has a resistance of 5 ohms and inductance of 1 H. At t = 0 it is connected
to a 2 V battery. Find the rate of rise of current when i = 0.2 ampere.

A. 3 A/sec B. 2 A/sec C. 1 A/sec D. 0.5 A/sec


Given: R = 5 ohms
L=1H
t=0
i = 0.2 A

Required:
Solutions:

V = iR + L

2 = (5) (0.2) + 1

72. A shunt motor has a rated armature current of 50 A when connected to 200 V.
The rated speed is 1,000 rpm and armature resistance is 0 1 Ω Find the speed if
the total torque is reduced to 70% of that at rated load and a 3 Ω resistance is
inserted in series with the armature.
REGISTERED ELECTRICAL ENGINEERS PRE-BOARD EXAMINATION
MARCH 2013
PROFESSIONAL ELECTRICAL ENGINEERING SUBJECTS

A. 474 rpm B. 469 rpm C. 464 rpm D. 460 rpm

Given: Ia1 = 50A


E1 = 200V
T1
T2 = 0.7T1
N1 = 1000 rpm

Required: N2

Solutions:

( )2

( )2

Ia2 = 27.32A

( )
= 469 RPM
( )( )

73. If the resistance of the field winding of a dc generator is increased the output
voltage will

A. decrease C. remain the same


B. increase D. fluctuate heavily

74. Wye-wye power transformers are protected by current transformers having


________ connection.

A. delta-delta
B. wye-delta
C. delta-wye
D. wye-wye
REGISTERED ELECTRICAL ENGINEERS PRE-BOARD EXAMINATION
MARCH 2013
PROFESSIONAL ELECTRICAL ENGINEERING SUBJECTS

75. An unbalanced wye-connected load connected to a 3-phase, 4-wire abc system


consists of Za = 10 j5 Ω, Zb = 8 j4 Ω and Zc = 20 + j0 Ω If the line to neutral
voltage is 120 V, find the total power in watts.

A. 4,125 B. 3,312 C. 5,520 D. 2,843

Given: Za = 10 j5 Ω
Zb = 8 j4 Ω
Zc = 20 j0 Ω
VT = 120 V

Required: PT

Solutions:
ST = + +

ST = + +

ST = 3312 – j1296

PT = 3312W

76. A three-phase, 60 Hz, transmission line has its conductors arranged in a


triangular configuration so that the two distances between conductors are 7.62 m
and the third is 12.8 m. The conductors are Dove with GMR of 0.0314 ft. If the
line is 60km long, what is the inductive reactance of the line?
A 38 73 Ω B 37 18 Ω Ω D 33 57 Ω

Given: F = 60Hz
d1 = d2 =7.62 m
d3 =12.8 m
GMR = Ds = 0.0314 ft
L = 60 KM

Required: XL
REGISTERED ELECTRICAL ENGINEERS PRE-BOARD EXAMINATION
MARCH 2013
PROFESSIONAL ELECTRICAL ENGINEERING SUBJECTS

Solutions:

Dm = √( )( )( )

L = 2x10-7 ln( )

XL = 2 ( )( )

77. The luminous intensity of a lamp is expressed in


A. lux B. lumens C. candela D.foot-candle

78. The barrier potential for a silicon diode at 25 oC is approximately

A. 0.4 v B. 0.3 v C. 0.7 v D. 0.5 v

79. Ten oersted is equivalent to

A. 10 Gb/m B. 1 Gb/m C. 10 Gb/cm D. 100 Gb/cm

80. In a 230 V, 3-phase, 3-wire system, two wattmeters are connected to measure
the power. The ratio of the readings of the wattmeters is 2.5 is to 1. What is the
power factor of the system?

A. 80.3% B. 76.6% C. 83.1% D. 86.6%

Given: ratio =

Required: Pf

Solutions:
√ ( )
Φ = tan-1 ( )

Pf = cos (36.59) = 80.3%


REGISTERED ELECTRICAL ENGINEERS PRE-BOARD EXAMINATION
MARCH 2013
PROFESSIONAL ELECTRICAL ENGINEERING SUBJECTS

81. An impedance draws a current i = 10 cos ωt – 30o) from a voltage v = 220 sin
ωt What is the maximum power?

A. 1,100 W B. 2,200 W C. 1,320 W D. 190.5 W

Solution: Vrms = xV

Vrms = (220) (0.707) = 155.54 V

Irms = i x = 7.07 A

Irms = (10) (0.707) = 7.07 A


P = Irms x Vrms

P = (7.07 A) (155.54 V)

P = 1100W

82. Calculate the magnetic field intensity to produce a flux density of 10 x 10 -3 tesla
at the center of a long straight solenoid.

A. 6,423 AT/m B. 7, 958 AT/m C. 8,323 AT/m D.7,869AT/m

Solution: H=

= Free space permeability


= (SI units)
=1 (cgs units)

H=

H= 7958 AT/m

83. A total of 0.8 kg of water at 20 oC is placed in a 1-kW electric kettle. How long a
time in minute is needed to raise the temperature of the water to 100 oC?
REGISTERED ELECTRICAL ENGINEERS PRE-BOARD EXAMINATION
MARCH 2013
PROFESSIONAL ELECTRICAL ENGINEERING SUBJECTS

A. 4.46 min B. 4.56 min C. 5.32 min D. 6.34 min

Solution: Q = mC

Q=(0.89)(4186J/kg-c)(100-20)c

Q = 267904J = W

W = Pt; t =
t=

t= ( )

t= 4.46 min

84. A certain amount of fuel can produce 10 quads of energy. In how many days will
the fuel be totally consumed if it is used to satisfy a demand of 10 13 BTU/day at a
power plant with and overall efficiency of 20 percent?

A. 190 B. 205 C. 195 D. 200

85. What is the equivalent of the constant C in the ABCD constants for the Nominal-
T circuit of a transmission line for which R = 10 Ω, XL = 20 Ω, Y = 400 µs for
each phase?
A. 0.996 0.115o
B. 22.25 63.45o
C. 10 + j20
D. 4 x 10-4 90o

C = 0 +jY
= 0 + j 400x10-6

C = 400x10-4 90

86. Which of the following ammeters is the most sensitive?


REGISTERED ELECTRICAL ENGINEERS PRE-BOARD EXAMINATION
MARCH 2013
PROFESSIONAL ELECTRICAL ENGINEERING SUBJECTS

A. 0-100 micro ammeter C. 0-1 micro ammeter


B. 0-5 milli ammeter D. 0-50 milli ammeter

Note: The smaller the amount of current, the more “sensitive” the
ammeter.

87. A transformer is to be used to match a secondary load impedance of 800 ohms


to a generator line impedance of 128 ohms. What should be the ratio of
transformation of the transformer for maximum power transfer?

A. 1: 6.25 B. 6.25: 1 C. 1: 2.5 D. 2.5: 1

Pmax = √

88. TV Channel 2 has a frequency of 60 MHz What is the wavelength?


A. 50m B. 500m C. 0.5m D. 5m
Solution: Wavelength ( ) =

Wavelength ( ) =

Wavelength ( ) = 5m

89. At a certain point of the system network the positive, negative and zero sequence
impedances are 0.25 p.u., 0.25 p.u. and 0.3 p.u., respectively. The base MVA is
100. The voltage level at that point is 34.5 kV. Determine the zero sequence
current for a one-line ground fault.

A. 8.132 A B. 7,516 A C. 6,275 A D. 2,091 A

Solution:
= =

= = 1.25pu
REGISTERED ELECTRICAL ENGINEERS PRE-BOARD EXAMINATION
MARCH 2013
PROFESSIONAL ELECTRICAL ENGINEERING SUBJECTS

=

= = 1673.48
√ ( )

= ( )

= 1.25 (1673.48)

= 2091.81

90. A 1-phase, 220-V 50-A induction motor draws 10 kW of power. A 1-phase, 8-kVA
synchronous motor is placed parallel with it in order to adjust the power factor to
unity. What must be the power factor of the synchronous motor?

A. 0.80 leading B. 0.82 leading


C. 0.84 leading D.0.86 leading

Solution :

Qc = P (tan )

= 1000 (tan (24.92) – tan (0))

Qc = 4582.58 VAR

= sin-1 ( )

= sin-1 ( )

Pf = cos (34.92) = 0.82 leading


91. Brushes of dc machine are usually made of
A. carbon B. hard copper C. soft copper D. iron
92. Impurities in an electrolyte can cause an internal short circuit condition called
A. Depolarization B. Electrolysis
REGISTERED ELECTRICAL ENGINEERS PRE-BOARD EXAMINATION
MARCH 2013
PROFESSIONAL ELECTRICAL ENGINEERING SUBJECTS

C. Local action D. Polarization


93. Calculate the time taken by the capacitor of 1 µF and in series with a 1 MΩ
resistance to be charged up to 80% of the final value.
A. 0.8 sec B. 1.61 sec C. 2,12 sec D. 1.98 sec
94. Determine the inductance L of a coil of 500 turns wound on an air cored torroidal
ring having a mean diameter of 300mm. The ring has a circular cross section of
diameter 50mm.
A. 6.54 x 10-3 H B. 6.54 x 10-4 H
C. 3.27 x 10-3 H D. 3.27 x 10-3 H
Solution:
N = 500 = 1.963x10-3m2
d = 300x10-3 m L = =
C= = (300x10-3)
= 0.942m ( )( )( )
D = 50mm
L = 6.54x10-4 H
( )
A= =

95. What is the minimum number of wattmeter required to measure unbalanced


power for a three phase system?
A. 2 B. 3 C. 4 D. 1
96. The rotor of a 220 v, 60 Hz, 4-pole induction motor has a power input of 80 kW. It
is observed that the rotor emf makes 90 cycles per minute, Calculate the rotor
copper loss.
A. 2 kW B. 5 kW C. 3 kW D. 4 Kw
Solution:
V = 220V
S= = = 0.025
f = 60hz
P=4 Pcu = SPr
Pin = 80kw = Pr = (0.025)(80kW)
Sf = 90 cpm Pcu = 2kW
= 90/60 cps= 1.5 = fr

97. A certain station in Luzon has 36 kV, 3-phase, is to supply 12 MW load at 30 kV


and 0.8 p.f. lagging over a 3-phase transmission line. Find the resistance of the
line if the length of the transmission line is 4,000 meters and the efficiency is
90%.
A 6 31 Ω B 3 56 Ω C 4 45 Ω D Ω
REGISTERED ELECTRICAL ENGINEERS PRE-BOARD EXAMINATION
MARCH 2013
PROFESSIONAL ELECTRICAL ENGINEERING SUBJECTS

Solution:
V = 36kV Ƞ=
PL = 12MW
VL = 30kV 0.9 =
Pf = 0.8 Lagging Pline = 1.33kW
Ƞ = 90%
PL = √ ILVLpf R=
R = 5.33
IL =
√ ( )( )
IL=288.68A

98. A dc shunt generator has an induced voltage of 220 v on open circuit. When the
machine is on load the voltage is 200 v. Find the load current of the filed
resistance is 100 Ω and armature resistance is 0 2 Ω
A. 100 A B. 102 A C. 98 A D. 96 A
Solution:
Eg = 220V
VL = 200V
RSH = 100
Ra = 0 2
ISH = =
= 2A
Eg = VL+IaRa
220 = 200 + Ia(0.2)
Ia = 100A
Ia = IL + Ish
Ish = 100 – 2
Ish = 98A
REGISTERED ELECTRICAL ENGINEERS PRE-BOARD EXAMINATION
MARCH 2013
PROFESSIONAL ELECTRICAL ENGINEERING SUBJECTS

99. Permeance is analogous to


A. Conductance B. Resistance
C. Reluctance D. Inductance
100. A 100 MW power station delivers 100 MW for 2 hours, 50 MW for 8 hours and is
shut down for the rest of each day. It is also shut down for maintenance for 60
days each year. Calculate its annual load factor.
A. 19% B. 21% C. 23% D. 25%
AUGUST 2013
REGISTERED ELECTRICAL ENGINEERS PRE-BOARD EXAMINATION
AUGUST 2013
PROFESSIONAL ELECTRICAL ENGINEERING SUBJECTS

1. Two Impedances C1 and C2 are connected in series across a 240 v source. If


C1 is 24µF and the total capacitance is 6µF, find the voltage across C 2.
A. 240v B. 60v C. 180v D. 120 v
Solution:
XCT= = =442 0970641 Ω Xc1= = 110 524 Ω
( )( ) ( )( )

I1=VT/XCT= = 0.5429 v
V1= (I1)(X1)=(0.5429)(110.524)= 60 v
V2= VT – V1=240-60
V2= 180 v
2. Estimate the average power output of a wind turbine having a blade diameter
of 10.7 m if the wind velocity ranges from 10 to 30 miles per hour.
A. 40.2kW B. 42.2kW C. 44.2kW D. 46.2kW

Given: Solution:

d= (10.7)(3.2808)=54.1ft P=2.46x10-3(d2)(v3)

v= 10.30 mph P10= 2.46x10-3(35.12)(103) = 3030.7

P30= 2.46x10-3(35.12)(303) = 81830.1

Po=42.2kW

3. A conductor radius of 0.1625 inch. What is the area in circular mils?


A. 26,406.25 B.82,957.68 C.100,000 D. 105,625

Given:

r=0.1625in = 162.5 mils

D= 162.5 mils 2 = 325 mils

A= D2 = 3252

A=105,625 CM

4. If there are eleven resistors of 33kΩ each in parallel, what is the total
resistance?
A. 363kΩ B 3 3kΩ Ω D 16 5kΩ
REGISTERED ELECTRICAL ENGINEERS PRE-BOARD EXAMINATION
AUGUST 2013
PROFESSIONAL ELECTRICAL ENGINEERING SUBJECTS

RT= = Ω
( )( )

5. To reverse the direction of rotation of a three-phase induction motor, it is


necessary to
A. Interchange all the three line connections
B. Reverse the polarity of the rotor circuit
C. Increase the resistance of the rotor circuit
D. Interchange any two of the three line connections
6. An unconventional conductor consists of three strands arranged vertically.
Find the GMR in terms of the radius r of an individual strand.
A. 1.564 r B.1.764 r C. 1.704 r D. 1.89 r
7. Transformers are rated in
A. KW B. KV C. KWHR D. KVA
8. A current is given by the expression i=2.5 + 0.8 + 0.5 .
What is the rms value of the current?
A. 1.54 A B.2.6 A C.2.54 A D. 1.89 A
Given:
i= 2.5 + 0.8 + 0.5
Solution:
IRMS= √
( ) ( ) ( )
=√

IRMS=1.89 A

9. A capacitor is charged to 250 coulombs for 5 seconds. What is the current


through the capacitor.
A. 25 A B. 50 A C. 500 A D. 1,250 A

Given: Solution:

Q=250 C Q=It

T= 5sec I=

I= ? I=50 A

10. In a series RL circuit, the inductor current _________ the resistor current.
A. Lags B. leads C. is equal to D. is negative of
REGISTERED ELECTRICAL ENGINEERS PRE-BOARD EXAMINATION
AUGUST 2013
PROFESSIONAL ELECTRICAL ENGINEERING SUBJECTS

11 A 120v dc shunt motor has an armature resistance of 200 mΩ If the full-load


armature current is 75 A, find the starting resistance needed to limit the armature
starting current to 150% of the full-load value.

A 0 804 Ω B 0 822 Ω 4 Ω D 0 862 Ω

Solution:

Vt=Eb + Ia(Ra+Rst) + Vbc

Eb=0 at starting

Rst=[(Vt-Vbc)/(1.5(Ia))] - Ra

Rst=[(120-2)/(1.5(75))] - 200×10^-3

Rst=0.849 ohm

12. The power factor of incandescent bulb is

A. 0.8 lagging B. 0.8 leading C. unity D. zero

13. The law that induces emf and current always opposes the cause producing
them was discovered by

A. Lenz B. Maxwell C. Faraday D. Ohm

14. An inductor in series with a 5 µF capacitor is connected to supply of 20 v,796


HZ. What is the inductance needed to make the current 5 A leading?

A. 0.002 H B. 0.007 H C. 0.004 H D. 0.008 H

GIVEN:

C = 5µF

V = 20 v

f= 796HZ

I=5
REGISTERED ELECTRICAL ENGINEERS PRE-BOARD EXAMINATION
AUGUST 2013
PROFESSIONAL ELECTRICAL ENGINEERING SUBJECTS

L=?

Solution"

Xc =1/ 2πfc = 1/ 2π 796 5x10-6)

Xc = 39 99 Ω

Z = V/I = 20/5 =4

Z= Xl - Xc XL = 2π f L

4 = XL - 39 99 3599 = 2π 796 L

XL = 35.99 L = 0.007 mH

15. A 60 kJ per minute heat is rejected from refrigerator. What is the power input
to the refrigerator if cop is 1.2?

A. 0.633 kw B. 0.733 kw C.0.833kw D. 0.933 kw

Solution:

COD = pi/ pr

60 kj/min x 1min/x = 1 kj/s = 1000w

1.2 = 1000/x

X = 0.833 kw

16. A 2.400/240 volts transformer is connected Y-∆ What is the base voltage of
the secondary?

A. 415.69 v B. 240 v C. 135.56 v D. 480 v

2,400/240

PRIMARY =2,400

SECONDARY =240

17. Temperature coefficient of a conductor is defined as the

A. increase in resistance per ohm per degree centigrade

B. increase in resistance per degree centigrade


REGISTERED ELECTRICAL ENGINEERS PRE-BOARD EXAMINATION
AUGUST 2013
PROFESSIONAL ELECTRICAL ENGINEERING SUBJECTS

C. increase in resistance per degree absolute

D. decrease in resistance per ohm per degree centigrade

18. A 400 v series motor working with unsaturated field is taking 60 A and
running at 840 rpm. The total resistance of the motor is 0.1 ohm. At what speed
will the motor run when developing half-full load torque.

A. 1,193 rpm B. 1,202 rpm C. 1,167 rpm D. 1,352 rpm

Solution:

(T2/T1) = (Ia2/Ia1)^2

(0.5T1/T1) = (Ia2/60)^2

Ia2 = 30 sq. rt of 2

Eb1 = Vt-Ia1(Ra)

=400-60(0.1)

Eb1 = 394v

Eb2 = Vt-Ia2(Ra)

=400-30 sq.rt of 2(0.1)

Eb2 = 395.757

(Eb1/Eb2) = [(N1(Ia1))/(N2(Ia2))]

N2 = [Eb2(N1)(Ia1)]/[Eb1(Ia2)]

= [395.757(840)(60)]/[394(30 sq.rt of 2)]

N2 = 1193 rpm

19. A capacitor is placed in parallel with two inductive loads, one of 20 A at 30°
lagging and another one of 40 A at 60° lagging. What must be the current in the
capacitor so that the total current from the source shall be at unity power factor?

A. 44.64 A B. 36.84 A C. 48.36 A D. 38.32 A

Solution:
REGISTERED ELECTRICAL ENGINEERS PRE-BOARD EXAMINATION
AUGUST 2013
PROFESSIONAL ELECTRICAL ENGINEERING SUBJECTS

I = (20<30)+(40<60)

= 37.32 + 44.64i

Therefore, the current in the capacitor is 44.64A

20. Open conductors run individually as service drops shall be _______.

I. insulted II. bare III. covered

A. I only B. II only C. III only D. either I or II

21. A three-phase short transmission line has a per phase impedance of 0.3 +
j0 4 Ω The receiving end voltage is 6 351 volts per phase and the voltage
regulations is not exceed 5 percent. Calculate the total line loss when it is
supplying maximum power.

A. 212.08 MW B. 7.69 MW C. 122.46 MW D. 40.81 MW

22. A parallel plate capacitor is made of 350 plates, separated by paraffined


paper 0.001cm thick (k=2.5). The effective size of the plate is 50 cm x 30 cm.
What is the capacitance of this capacitor?

A. 35 µF B. 70 µF C. 115 µF D.140 µF

GIVEN:

n = 350

k = 2.5 =

d=0.001 cm

Σr= 50x10-2x30x10-2)

Solution:

C = ΣoΣr / d n-1)

C=(8.8SAx1018)(2.5)(50x10-2x30x10-2)/0.001x10-2(350-1)

C = 115 µF

23. The percentage overloading of two 20 - KVA transformers when supplying a


balanced 3 ᶲ load of 10 kVA in a V-V bank is ______ percent.
REGISTERED ELECTRICAL ENGINEERS PRE-BOARD EXAMINATION
AUGUST 2013
PROFESSIONAL ELECTRICAL ENGINEERING SUBJECTS

A. 15 B. 57.7 C. 73.2 D. 33.3

24. The power factor of an alternator is determined by its

A. speed B. load C. excitation D. prime mover

25. The pitch factor for 3rd harmonic in an alternator having 18 slots per pole and
coil span 5/6 of pole pitch is

A. 0.707 B. 0.253 C. 0.966 D. 1

Solution:

K3=cos 3α/2 K3=cos(3(30)/2)

α=180O 1-5/6)=30O =0.707

26. A certain power plant has reserve capacity above the peak load of MW. The
annual factors are load factor = 59%, capacity factor = 41% and use factor 46%.
Determine hours per year not in service.

A. 952 hours B. 965 hours

C. 924 hours D. 937 hours

LF = L/ PL

∆L = LE (PL)

= 0.59 (PL) -1

INSTALLED CAPACITY = PL + RESERVED

PL = 1.C -10 -2

SUBSTITUTE EQ. 2 IN EQ. 1

∆l =0.59 (INSTALLED CAPACITY -10)

=0.59 INSTALLED CAPACITY -59- 3

PCF = ∆L / INSTALLED CAPACITY


REGISTERED ELECTRICAL ENGINEERS PRE-BOARD EXAMINATION
AUGUST 2013
PROFESSIONAL ELECTRICAL ENGINEERING SUBJECTS

∆L =0.41 (INSTALLED CAPACITY) -4

EQUATE 3 TO 4

∆L = 0.59 (32.78) -5.9

= 13.44 MW

PUF YEAR = ∆L IN MW - HR / INSTALLED CAPACITY IN MW x N

0.46 = 13.44 x 8768 /32. 78 N

N= 7808 HRS

LET n = 8760 -7808

= 952 HRS

27. The speed of a synchronous motor

A. reduces as load increases

B. increases as load increases

C. adjusts itself to new equilibrium speed whenever load changes

D. always remains constant

28. A heater wire of length 50 cm and 1 mm2 in cross-section carries a current of


2 A when connected across a 2 v battery. What is the resistivity of the wire?

A. 2 x 10- Ω-m B. 2 x 10-5 Ω-m

C. 4 x 10-6 Ω-m D. 4 x 10-5 Ω-m

GIVEN: Find:

L = 50 x 10 -2 m I = 2A P=?

d = 1x10-3 m2 V=2v

Solution:

R = V/ I = 2/2 = 1Ω

R = PL/A
REGISTERED ELECTRICAL ENGINEERS PRE-BOARD EXAMINATION
AUGUST 2013
PROFESSIONAL ELECTRICAL ENGINEERING SUBJECTS

P=RA/L

P= 1Ω 1x10-3m2)/(50 x 10-2m) =2 x 10- Ω-m

29. A string insulator has 4 units. The voltage across the bottom most unit is 30%
of the total voltage. What is the string efficiency?

A. 30% B. 60% C. 75% D. 83.33%

Given: Solution:

Where: n=4 (number of disc) %SE=Vph/n x VDNC

Vph=operating phase voltage %SE=Vph/4 x Vph x 0.30

Voltage disc nearest to conductor=VDNC %SE=(1/4 x 0.30)100


=Vph x 0.30 = 83.33%

30. When a line to ground faults occurs, the current in a faulted phase is 100 A.
What is the zero sequence current in this case?

A. zero B. 33.3 A C. 66.6 A D. 100 A

31. A transmission line has a total series impedance of 40 + j125 ohms and a
total shunt admittance of 10-3 mho. Determine the constant D of the line.

A. j0.001 B. 0.9376 + j0.0196

C. 1 + j0.0656 D. 0.001 + j0.0656

32. A six-pole three-phase 440-v, 60 HZ induction motor develops 10-HP at


1,150 rpm, the power factor being 80% lagging. Stator and core losses amount to
400 w and 350 w, respectively. Frictional losses amount to 0.5-hp. Calculate
motor line current.

A. 12.24 A B. 12.41 A C. 11.42 A D. 14.64 A

33. The efficiency of thermal power plant is approximately

A. 10% B. 30% C. 60% D. 80%

34. A 6.6 kV Y-connected, 3-phase alternator has a synchronous reactance of 6


Ω per phase and a negligible resistance At a certain field current, the armature is
REGISTERED ELECTRICAL ENGINEERS PRE-BOARD EXAMINATION
AUGUST 2013
PROFESSIONAL ELECTRICAL ENGINEERING SUBJECTS

known to induce 5.8 kV per phase. Calculate the maximum power that can be
developed by the alternator at this excitation.

A. 12 MW B. 10 MW C. 9.8 MW D. 11 MW

Given: Find: Max power

V line = 6600 / √3 Z= 0+j6

X= 6 ohms Va= 5800

R= 0

Solution:

I= V / Z Pmax = 3 VI

I= 6600 /√3 / 0 j6 Pmax = 3 (5800) (635.09) `

I= -j635.09 = 635.09A Pmax = 11050566W or 11.05MW

35. A power plant consumes 3,600 tons of coal per day. If the coal has an
average energy content of 10,000 BTU/lb, what is the plant's power output?
Assume an overall efficiency of 15%.

A. 132 MW B. 130 MW C. 128 MW D. 126 MW

36. The positive, negative, and zero sequence reactances of a 20-MVA, 1302 kV
synchronous generator are 0.3 pu, 0.2 pu, nad 0.1 pu, respectively. The
generator is solidly grounded and is not loaded. A line to ground fault occurs on
phase a. Neglecting all the resistances, determine Ia1.

A. -j5 pu B. -j1.67 pu C. -j2.87 pu D. -j3 pu

Solution:

Ia1 (pu) = 1/ Z1 +Z2+Z3

= 1 / j0.3+j0.2+j0.1

= -j1.67

37. The electromagnetic torque developed in a motor is 150 N-m. If the field flux
is decreased by 20% and armature current is increased by 15%, find the new
electromagnetic torque developed.
REGISTERED ELECTRICAL ENGINEERS PRE-BOARD EXAMINATION
AUGUST 2013
PROFESSIONAL ELECTRICAL ENGINEERING SUBJECTS

A. 138 N-m B. 163 N-m C. 159 N-m D. 104 N-m

Given: Find: T2 (new)

T1:50 N-m Ia1= 1

Ø1 =1 Ia2:1+.15=1.15

Ø2:1-0.2=0.8

Solution:

T1/T2=(Ø1) (Ia1)/(Ø2) (Ia2)

150/T2= (1) (1) / (0.8) (1.15).

T2=138 N-m

38. A 220 v, 3-phase , 4-wire abc system supplies unbalanced wye-connected


resistance of Ra = 10 Ω, Rb= 15 Ω and Rc= 20 Ω Determine the total power of
the system in watts.

A. 3,495 B. 3,560 C. 3,385 D. 3,895

Solution:

VØ = 220/√3 = 127 02 v

Pa = VØ/ Ra = 127. 022/ 10= 1,613.40

Pb = VØ/ Rb= 127.02 2 /15 = 1,075.60

Pc = VØ/ Rc= = 127.02 2 /20 = 806.70

Pt= 1,613.40 +1,075.60 +806.70

Pt= 3,495.70

39. A coil of wire of 100 turns enclosing an area of 0.09 m 2 has its plane
perpendicular to a uniform magnetic field of flux density 0.5 T. the coil is turned to
position wherein its plane is parallel to the magnetic field in 0.05 sec. Find the
average emf generated in the coil in volts.

A. 15 B. 10 C. 12 D. 9
REGISTERED ELECTRICAL ENGINEERS PRE-BOARD EXAMINATION
AUGUST 2013
PROFESSIONAL ELECTRICAL ENGINEERING SUBJECTS

e= 0.0045
Solution:
e = N dd/dt
Ø=ß∆
e= 100 (0.045/ 0.5)
= (0.5) (0.09)
e=9v
Ø = 0.0045
40. A 10-hp, 230 v, three-phase Y-connected synchronous motor has a
reactance of 3 ohms per phase a negligible resistance. To what voltage must the
motor be excited to deliver full-load at unity power factor and 90% efficiency?

A. 242 v B. 254 v C. 262 v D. 248 v

41. To a series RLC circuit, a voltage of 10 v is applied. If Q of the coil at


resonant frequency is 20, the voltage across the inductor at resonant frequency
will be.

A. 200 v B. 100 V C. 75 v D. 50 v

Solution:
QFactor = EL / ET
20 = EL/10 EL = 200 V
42. One advantage of distributing the winding in alternator is to

A. Reduce harmonies C. reduce the amount of cooper


B. improves voltage wave fromD. decreases the value of the voltage

43. A dc shunt motor running at 1,200 rpm has armature resistance of 0 15 Ω


The current taken by the armature is 60 A when the applied voltage is 220 v. If
the load is increased by 30%, find the variation in the speed.
A. 1,040 rpm B. 1,162 rpm C. 1,110 rpm D. 1,184 rpm

44. the capacitance between any conductors of a three-phase, three-conductor


cable is 2 µF. The cable operates at 11 kV line voltage and 50 HZ. What is the
charging current through the cable capacitance?

A. 7.98 A B. 3.99 A C. 6.91 A D. 13.82 A

45 The per unit reactance’s of a synchronous generator are X d= 1, Xd1 = 0.35,


and xd11 = 0.25. The generator supplies at 1.0 per unit load at 0.8 power factor
lagging. Calculate the voltage behind the subs transient reactance.

A. 1.79 pu B. 1.24 puC. 1.17 pu D. 1.59 pu


REGISTERED ELECTRICAL ENGINEERS PRE-BOARD EXAMINATION
AUGUST 2013
PROFESSIONAL ELECTRICAL ENGINEERING SUBJECTS

46. A series RLC circuit has a Q of 5 at its resonance frequency of 100 kHZ.
Assuming the power dissipation of the circuit is 100 w when drawing a current of
0.4 A, determine the capacitance C of the circuit.

A. 2.04 nF B. 2.32 nF C. 3.02 nF D. 2.54 nF

GIVEN:

Q=5
F = 100 X 103Hz
P = 10W
I = 0 8∆
C =?
SOLUTION:

Q=

R=
S= ; Xc = 781.25
Xc = ; 782.25

= 𝐶
( )( )
REGISTERED ELECTRICAL ENGINEERS PRE-BOARD EXAMINATION
AUGUST 2013
PROFESSIONAL ELECTRICAL ENGINEERING SUBJECTS

47. An iron ring of circular cross-section of 5 x 10-4 m2 has a mean circumference


of 2 m. It has a saw-cut of 2 x 10-3 m length and is wound with 800 turns of wire.
Determine the exciting current when the flux in the air gap is 0.5 x 10 -3 Wb. Given
µr of Iron = 600 and leakage factor is 1.2. Assume areas of air gap and iron are
same.

A. 2.95 A B. 3.95 A C. 4.95 A D. 5.95 A

48. A 50 hp, 440 v, 60 HZ, 3-phase motor operates at full-load with an efficiency
of 89% and power factor of 0.85 lagging. Determine the capacitance in µf per
phase of wye-connected capacitor bank to be connected across the motor
terminals to improve the power factor to 0.95 lagging.

A. 148 B. 167 C. 188 D. 210

49. Impurities in an electrolyte can cause an internal short circuit condition called

A. depolarization B. electrolysis C. local actionD. polarization

50 A dc constant voltage source feeds a resistance of 2,000 kΩ in series with a


5µF capacitor. Find the time taken for the capacitor when the charge retained will
be decayed to 50% of the initial value, the voltage sourcing being short circuited.

A. 11 B. 12 C. 10 D. 14

51. A resistance of 40 ohms and inductance of 79.6 mH are connected in parallel


across a 240 v, 60 HZ ac supply. Find the total current in amperes.

A. 11 B. 12 C. 10 D. 14

IT= V/Z XL= WL

XL = 2∏ 60 79 6 mH

XL = 30 008 Ω

Z = RXL___
√R2+ XL 2
Z = 40(30.008)

√ 402 + 30.0082
Z=24.004
IT = 240/24.004 = 9.998 or 10 A
REGISTERED ELECTRICAL ENGINEERS PRE-BOARD EXAMINATION
AUGUST 2013
PROFESSIONAL ELECTRICAL ENGINEERING SUBJECTS

52. A 230 v, 3-phase source supplies a balanced delta connected in parallel


across a 240 v, 60 HZ ac supply. Find the total current in amperes.

A. 33.1 B. 39.8 C. 26.5 D. 29.3

SOLUTION:
IL= √3 Vp = √3 230
Z 8 – j6
IL = 39.8

53. Three unbalanced 3-phase currents are given as follows:

Ia= 10 -30° A , Ib= 0 and Ic= 10 -150° A. Find Ic1-

A. 3.34 150° A B. 3.34 -150° A C. 3.34 210° A D. 3.34 -135° A

SOLUTION:
Ia1 = 1 {(Ia+a(Ib)+a2(Ic)}
= 1{(10<-30) + (1<120(0) + (1<240) (10< -150))}
= 3.34<300∆
IC1= a(Ia1)
=(1<120)(3.34<30) =3.34<1500A
54. Two straight parallel wires 2 m long, 3 mm apart carries a current of 8 A in
opposite direction. Calculate the force between these conductors.
A. 8.25 x 10-3 N B. 8.75 x 10-3 N
-3
C. 8.33 x 10 N D. 8.53 x 10-3 N
SOLUTION:
F=(2X10-7) (I1I2) ( )
F=(2X10-7)(8X8)( )
-3
F= 8.53X10 N
55. A three-phase, 6-pole, 50 HZ, induction motor develops a maximum torque of
30 N-m at 960 rpm. Calculate the torque produced by the motor at 6% slip. The
rotor resistance per phase is 0 6 Ω
A. 24.7 N-m B. 25.7 N-m C. 26.7 N-m D. 27.7 N-m

Tout =Tdev. – Tloss


Ns = (120) (50)
6
=1000
Nr = (1000) (1-0.06)
= 940 rpm

Tdev = 9.55 Pm
REGISTERED ELECTRICAL ENGINEERS PRE-BOARD EXAMINATION
AUGUST 2013
PROFESSIONAL ELECTRICAL ENGINEERING SUBJECTS

30 = 9.55Pm
960
Pm = 3015.71 w
Pr = Pm = 3015.71 Pr= 3208.20
(1-s) 1-0.06
2
Pr =I (R/S)
I= 17.91
Ploss= I2R = (17.91)2 (0.6)
=192.46

Tloss = 60 Ploss
2π Nr
Tloss = 60 (192.46) (3)
2π 940
Tloss= 5.87
Tout = 30 – 5.87

=24 13 24.7

56. The turn-on voltage of a Ge junction diode is ______volt.


A. 0.7 B. 0.3 C. 1 D. 0.1
= 0.3 V

57. A 40 miles, 60 cycle single phase line consists of two 000 conductors spaced
5 ft apart. Determine the charging current if the voltage between wires is 33 kV.
The diameter of 000 conductors is 410 mils.
A. 3.92 A B. 2.92 A C. 1.92 A D. 4.92 A

58. A transformer has 4% resistance and 6% reactance drop. Find the voltage
regulation at full load 0.8 p.f lagging.
A. 6.8% B. 5.8% C. 8.8% D. 7.8%
SOLUTION:
%Vr=√ cosθ %IR 2 SINθ %IX 2-1
=√ 0 8 0 04 2+(SIN(36.87)+(0.06))2-1
=0.068X100
=6.8%

59. A dc motor takes an armature current of 100 A at 230 v. The armature


resistance is 0 03 Ω The total number of lap connected armature conductors is
500 and the number of poles is 4. The flux per pole is 0.03 Wb. What is the
torque developed by the motor?
A. 242.86 N-m B. 218.93 N-m
REGISTERED ELECTRICAL ENGINEERS PRE-BOARD EXAMINATION
AUGUST 2013
PROFESSIONAL ELECTRICAL ENGINEERING SUBJECTS

C. 238.73 N-m D. 227.54 N-m


GIVEN: SOLUTION:
I=100A P=4 T=? T=
( )( )( )( )
V=230V T= ( )
R=0 03Ω T=238.73N-M
Z=500Ω

60. A 0.5 µF capacitor, an 80 mH inductance and a 500-ohm resistor are


connected in parallel across a voltage source of V = 12 -j6 v. Find the current as
a phasor if the angular velocity is 500 rad/sec.
A. 0.333 92° A B. 0.333 -92° A
C. 0.333 122° A D. 0.333 -122° A

IT = V/Z
XL= WL = 500 rad/sec (80x10-3)= 40
XC= 1 = 1/500(0.5 µF) = 4000
WC
Z= RXLXC/√XL2XC2+R2 (XC-XL)2
Z= 500(40) (4000) _________
√ 40 2(4000)2+ (500)2(4000-40)2

Z= 40.27
Ѳ=cos-1(Z/R) = (40.27/500)
=85.38

IT = 12-j6
40.27< 85.38

IT = 0.333 < - 122 A

61. What is the fourth harmonic of 800 KHZ?

A. 200 KHZ B. 400 KHZ C. 3.20 MHZ D. 4.00 MHZ

F4= F(n) = 800KHZ(4) = 3.2MHZ

62. A mode of communications in which either party can hear while taking, thus
allowing one party to instantly interrupt the other at any time, is called

A. Half simplex B. Half duplex


C. Full simplex D. Dull duplex
REGISTERED ELECTRICAL ENGINEERS PRE-BOARD EXAMINATION
AUGUST 2013
PROFESSIONAL ELECTRICAL ENGINEERING SUBJECTS

63. suppose a cell of 1.5 v delivers 100 mA for 7 hours and 20 minutes, and the it
is replaced. How much energy is supplied during this time?

A. 0.49 Whr B. 1.1 Whr C. 7.33 Whr D. 733 mWhr

W(energy) = Pt =(1.5v)(100x10-3)(7+ ) = 1.1Whr

64. A device that repeatedly reverse the polarity of a magnetic field in order to
keep a dc motor rotating is known as

A. a solenoid B. an armature coil

C. a commutator D. a field coil

65. Find the cross-sectional area of the core of a 10 turns transformer for a
voltage of 50 v at 50 HZ. The flux density is 0.9 Wb/m2.

A. 0.025 m2 B. 0.046 m2 C. 0.065 m2 D. 0.085 m2

SOLUTION:

GIVEN:

N=10, V=50v, f=50Hz, Ø=0.9Wb/m2

A= = = 0.025m2
( )( )

66. The rotor resistance of an 8-pole, 50 HZ, wound rotor induction motor is 0.5
Ω per phase The speed of the motor is 720 rpm at full load Determine the
external resistance to be connected with the rotor circuit to reduce the speed to
680 rpm for full-load torque.

2 Ω B 1 1625 Ω C 0 5225Ω D 0 8625 Ω

SOLUTION:

P=8, f=50Hz, ZØ=0 5Ω, N1=720rpm, N2=680rpm

Ns1=(120(50)/8) = 750rpm

s=(750-680/750) = 0.04

S2/S1 = R2/R1
REGISTERED ELECTRICAL ENGINEERS PRE-BOARD EXAMINATION
AUGUST 2013
PROFESSIONAL ELECTRICAL ENGINEERING SUBJECTS

S2=750-680/750 = 0.093

0.093/0.04 = R2/0.5

R2 = 1 1625Ω

Re = 1.11625 – 0.5 = 0.6625

67. 87 is the device function number assigned by ANSI to this relay;

A. lock out relay C. directional relay


B. undervoltage relay D. differential relay

68. A series RL circuit is connected across a dc source through a switch. Three


milliseconds after the switch is closed, the voltage across the inductance is 20 v
and drops to 5 v after ms. If R = 100 ohms, find L. Assume at t = 0, i = 0.

A. 1.59 H B. 1.30 H C. 1.22 H D. 1.25 H

SOLUTION:

R=100Ω, V=20V, V 3 =5V, t=0, i=0

E=Ee-(R/L)t

20=Ee-(100/L)(.003) – Eq. 1

5=Ee-(100/L)(.025) – Eq. 2

Divide Eq.1 by Eq. 2

20 = Ee-(100/L)(.003)

5 Ee-(100/L)(.025)

4 = e-(.3/L)

e-(2.5/L)

4e-2.5/L=e-.3/L

Ln4 – (-2.5/L) = (-0.3/L)

L = 1.59 H
REGISTERED ELECTRICAL ENGINEERS PRE-BOARD EXAMINATION
AUGUST 2013
PROFESSIONAL ELECTRICAL ENGINEERING SUBJECTS

69. A generator rated 600 KVA, 2,400 v, 60 HZ , 3-phase, 6-poles and wye-
connected has 10% synchronous reactance. If a three-phase fault occurs, what
will be the short circuit current?

A. 1, 443 A B. 1,532 A C. 1,435 A D. 1,426 A

GIVEN: SOLUTION:

Sb=600KVA I F= ( )

( )
V=2,400V IF=√ ( )( )

f=60HZ IF=1443 A

P=6

IF=?

70. A coil of inductance 8 mH, produces 80 µWb of magnetic flux. Of this total
flux, 60 µWb are linked to a second coil of inductance 2 mH. How much is the
mutual inductance between coils?

A. 2 mH B. 3 mH C. 4 mH D. 5 mH

SOLUTION:

K= =

K=0.75

M=K√L1L2

=0 75√ 8X10-3)(2X10-3)

=3X10-3H = 3MH

71. Wind mils in addition to proper gearing is the best driver for

A. dc generator C. inductance generators


B. synchronous generators D. rotary rectifiers

72. A rectangular bus bar is made of aluminum and is 90 cm long, 10 cm wide


and 1 cm thick. If the bus bar current flows along its length, find the bus bar
conductance provided the bus bar conductivity is 3.6 x 108 s/m.
REGISTERED ELECTRICAL ENGINEERS PRE-BOARD EXAMINATION
AUGUST 2013
PROFESSIONAL ELECTRICAL ENGINEERING SUBJECTS

A. 0.4 x 106 s B. 0.4 x 105 s C. 0.5 x 106 s D. 0.5 x 105


s

( )( )( )
G= = = 0.4 x 106 s

73. An energy source supplies 500 J of energy at 100 v for a certain period of
time. Determine the quantity of charge passed through.

A. 0.2 c B. 50 kc C. 20 c
D. 5 c

SOLUTION:

W=Pt

W=Ivt Q= = = 5C

74. A 220 v, 3-phase supply has a balanced delta connected load with
impedance of 15 + j20 ohms/phase. What is the total real power of the load in
watts?

A. 5,133.3 B. 4,122.5 C. 2,965.6 D. 3,484.8

GIVEN:
V=220
ZØ=15tj20Ω=25<53 13
SOLUTION:

P=3( COSØ)

P=3( 𝐶 ( ))

P=3,484.8W

75. In a 3-phase, 4-wire system, the current in line A has Ia1 = 200 0° A Ia2
=100 60° A, the current through the neutral conductor is 300 -60° A. Find the
line current Ibin amperes.

A. 300 120° B. 300 -60° C. 300 240° D. 100 -30°

GIVEN: SOLUTION:
REGISTERED ELECTRICAL ENGINEERS PRE-BOARD EXAMINATION
AUGUST 2013
PROFESSIONAL ELECTRICAL ENGINEERING SUBJECTS

Ia1=200<0 In= BIao


Ia2=100<60 Iao=100<-60
In+300<-60 Ib=100<-60+(1<240)(2)(1<120)(100)
Ib=? Ib=300<-120+360
Ib= 300<240

76. Calculate the no-load current taken by a 100 kW, 460 v, shunt motor
assuming the armature and field resistance to remain constant and equal to 0.03
Ω and 46 Ω, respectively The efficiency at full load is 88%

A. 23.5 A B. 24.5 A C. 26.0 A D. 28.0 A

Pʟ = 100 kw Since Ra and Rs are constant

Vʟ = 460 V SPʟ = Pʟ - Po = 100 x 103 – 0.88 (100x103) = 12,000


watts

Ra = 0 03 , Rs = 46 Ω SPʟ = Vʟ Ia-Ia2 Ra

12,000 = 460 (Ia) – ( Ia2 (0.03)

Ia = 26.13 A = Iʟ

77. A 100 MW hydroelectric station is supplying full load for 10 hrs in a day .
Calculate the volume of water which has been used. Assume effective head of
the water as 200 m and the overall efficiency of the station as 80%. Assume 1 m 3
of water = 1,000 kg.

A. 2,232,435 m3 B. 2,345,456 m3

C. 2,293,560 m3 D. 2,371,402 m3

Pt = Q8hn

( )( )( )
Q= ( )(
= 2,293,577.98 m3
)

78. The dc generator used as pilot exciter is

A. series B. Shunt

C. over compound D. flat compound

79. A lead storage battery is rated at 12 volts. If the internal resistance is 0.01
ohm, what is the maximum power that can be delivered to the load?
REGISTERED ELECTRICAL ENGINEERS PRE-BOARD EXAMINATION
AUGUST 2013
PROFESSIONAL ELECTRICAL ENGINEERING SUBJECTS

A. 1,200 w B. 1,800 w C. 3,600 w D. 7,200 w

V = 12 volts P= = = 3600 W
( )

R = 0 01 Ω

80. Bundled conductors in EHZ transmission lines results to

A. reduce capacitance C. increase capacitance


B. increase inductance D. increase resistance

81.The per unit impedance of a circuit element is 0.30. If the base KV and base
MVA are halved, then the new value of the per unit impedance of the circuit
element will be,

A. 0.30 B. 0.60 C. 0.0030 D. 0.0060

Xpu2=Xpu1( )2( )

Xpu2=0.3 ( )2 ( ) ; xpu2=0.60

82. What should be the turns ratio of a three-phase transformer to transform


10,000 KVA from 230 KV to 4,160 V if the transformer is to be connected ∆-Y?

A. 94 B. 96 C. 97 D. 98

(√ )
a= =

a = 96

83. The input power to a 600-v synchronous motor is measured by means of a


two-method. One wattmeter reads 64 kW and the other reads 36.5 kW. The
motor is known to be operating at leading p.f. Determine the line current drawn.

A. 110 A B. 104 A C. 112 A D. 107 A


REGISTERED ELECTRICAL ENGINEERS PRE-BOARD EXAMINATION
AUGUST 2013
PROFESSIONAL ELECTRICAL ENGINEERING SUBJECTS

SOLUTION:

√ ( )
Ø=tan-1( )
( )

Ø=25.360

W=VLILCOS(30-Ø)

64X103=(600)(IL)COS(30-25.36)

IL=107

84. A power plant has an overall efficiency of 30%. If this plant can consume
4,200 kilograms of coal per hour, estimate the total electric energy produced in
one day. Assume the calorific value of the coal being used is 8,000 kcal per
kilogram.

A. 183 MWHR B. 168 MWHR

C. 176 MWHR D. 155 MWHR

Win = 4200 kg/hrs x 5000 kcal/kg x 24hrs

Win = 504 x 106kcal x = 586.05 MWHR

Wouth = n (Win) = 0.3 (586.05 MWHR) = 175.81 MWHR

85. Four lamps are suspended 6 m above the ground at the corners of a lawn 4
m on each side. If each lamp emits 250 cd, calculate the illumination at the
center of the lawn.

A. 20.56 lux B. 21.62 lux C. 19.85 lux D. 22.34 lux

d2=(2)2+(2)2+(6)2=44

d=6.63

Et= 4 EACH LAMP

= 4 ( )COSØ=4( )( )

=20.56LUX
REGISTERED ELECTRICAL ENGINEERS PRE-BOARD EXAMINATION
AUGUST 2013
PROFESSIONAL ELECTRICAL ENGINEERING SUBJECTS

86. A 170-mile, 230-kV, 60-HZ, three-phase single-circuit transmission line uses,


a triangular arrangement with 20 ft. 20 ft, and 36 ft spacing respectively. If the
line conductors have a GMR of 0.0217 ft, determine the capacitive susceptance
to neutral per mile.

A. 4.81 x 10-5 mho per mile C. 7.92 x 10-5 mho per mile
B. 12.74 x 10-6 mho per mile D. 2.47 x 10-6 mho per mile

Dm = √ = 24.33 ft

( )
Cn = = 7.922 x 10-12 F/m x = 1.275x10-8 F/mile
( )

Xc = = ( )( )

Xc = 208091.59 Ω-mile
= 4.81 x 10-5

87. The surge impedance of an underground cable is around

A 50 Ω B 2 Ω C 400Ω D. 100 Ω

88. A process equipment contains 100 gallons of water at 25°C. It is required to


bring it to boiling in 10 minutes. The heat loss is estimated to be 5%. What is the
kW rating of the heater?

A. 50.5 kW B. 252 kW C. 125 kW D. 207 kW

V = 100 gallons Pt eff = mCΔT

( )( )( )
T = 25°C P= ( )( )

T = 100°C @ 10 minutes

(eff)= 0.95 P = 208.47 KW

89. A 500 MCM ACSR cable has 37 strands. Determine the diameter in mils of
each strand.

A. 116.25 B. 118.34 C. 110.35 D. 120.24


REGISTERED ELECTRICAL ENGINEERS PRE-BOARD EXAMINATION
AUGUST 2013
PROFESSIONAL ELECTRICAL ENGINEERING SUBJECTS

Aт = 500 MCM Astrands = = 13513.51 CM

N = 37 strands

d=√ = √ = 116.25 mils

90. A contact device installed at the outlet for the connection of a single
attachment plug.

A. pendant B. switch C. receptacle D. plug

91. An 8-pole lap wound dc generator has 1,000 armature conductors, flux of 20
mWb per pole and emf generated is 400 v. What is the speed of the machine?
A. 1,000 rpm B. 1,800 rpm C. 1,200 rpm D. 1,500 rpm
N = E(60)a / PZ0
N = 400(60)(8) / 8 (1000) (20x10-3)
N = 1200rpm
92. An air cored coil has 500 turns. The mean length of magnetic flux path is 50
cm and the area of cross-section is 5 x 10-4 m2. If the exciting current is 5 A,
determine the flux density.
A. 3.1415 x 10-3 T C. 6.283 x 10-3 T
B. 3.1415 x 10-4 T D. 6.283 x 10-4 T

Given: Solution:

N = 500 H = NI / L = 500 (5) / 0.5 = 5000


L = 50 cm to 0.5 m B = uH = 4π x 10-7 x 5000
A = 5 x 10-4 m2 B = 6.283 x 10-3T
I = 5A

93. In a balanced, 3-phase, wye-connected voltage source with phase sequence


abc, Vab= 10° v. Find the phase voltage Von.

A. 230 -230°v B. 132.8 -80°v

C. 132.8 -140°v D. 132.8 -260°v.


Vab = <10°
Assume 230v
REGISTERED ELECTRICAL ENGINEERS PRE-BOARD EXAMINATION
AUGUST 2013
PROFESSIONAL ELECTRICAL ENGINEERING SUBJECTS

230 / √3 <10
Von = 132.8<10°-270°
Von = 132.8<-260°v
94. When carrier wave is modulated at 100%, its power is increased by ____
percent.
A. 100 B. 150 C. 50 D. 0
95. A certain amount of fuel can produce 10 quads of energy. In how many days
will the fuel be totally consumed if it is used to satisfy a demand of 10 13 BTU/day
at a power plant with an overall efficiency of 20 percent?
A. 170 B. 180 C. 190 D. 200
96. Differential relays are installed to protect the equipment against.
A. reverse current B. internal faults C. overcurrent D. no voltage
97. A short 3-phase, 3-wire transmission line has an impedance of 3 + j4 ohms
per wire. At the receiving end are connected a 3-phase inductive load drawing
3,500 kW at 0.8 power factor and line to line voltage of 13, 200-volt and a
capacitor bank drawing 150 amperes line current. Calculate the voltage at the
sending end.
A. 13, 808 v B. 13, 718 v C. 13, 758 v D. 13, 658 v
98. A 15 KVA, single-phase distribution transformer has a core and copper
losses at rated condition of 90 w and 300 w, respectively. Determine its
maximum efficiency at 0.707 power factor lagging.
A. 95% B. 96% C. 97% D. 98%
Given: Solution:
S = 15KVa Pout = (S) (p.f) = (15KVa)(0.707) = 10605W
Pcu= 300W nmax= Pout / Pout + Pcu+Pco = (10605 / 10605 + 90 + 300) x
100%
Pco= 90W nmax = 97%
p.f = 0.707
nmax = ?
REGISTERED ELECTRICAL ENGINEERS PRE-BOARD EXAMINATION
AUGUST 2013
PROFESSIONAL ELECTRICAL ENGINEERING SUBJECTS

99. A three-phase, 400 v 50 HZ induction motor has a speed of 900 rpm on full-
load. The motor has six poles. How many complete alternations will the rotor
voltage takes per minute?
A. 30 B. 60 C. 300 D. 600
Given: Solution:
N = 900rpm Ns = 120f / p = 120 x 50 / 6 = 1000 rpm
f = 50 Hz s = Ns - N / Ns = 1000 - 900 / 1000 = 0.1 or 10%
p=6 f’ = s x f = 0 1 x 50 = 0 5 sec x 60 min = 30 min
100. This is the porcelain insulator for the end poles;
A. pin B. post C. suspension D. akimbo.
MARCH 2014
REGISTERED ELECTRICAL ENGINEERS PRE-BOARD EXAMINATION
MARCH 2014
PROFESSIONAL ELECTRICAL ENGINEERING SUBJECTS

1. In television system, channels 2 to 13 are classified as


A. VHF B. UHF C. SHF D. EHF

Sol’n
VHF 2-13 channel

2. Two inductors 20 mH and 10 mH are connected in series carrying current of 1


A. Find the stored energy.
A. 15 mJ B. 20 mJ C. 30 mJ D. 60 Mj

Sol’n

L1=20mH
L2=10mH
LT=30mh

WL= I2 L
WL= (1)(30mH)
WL=15mJ

3. Three capacitors are connected in parallel. C1 = 12 µF, C2 = 10 µF, and C3 =


40 µF. What is the total capacitance?
A. 45 µF B. 5 µF C. 10 µF D. 62 µF

Sol’n
C1= 12µF C3=40µF
C2=10µF

CT=C1 + C2 + C3
CT= 12µF + 10µF + 40µF
CT= 62 µF

4. For an ac motor, the frequency is 60 Hz and the speed is 300 rpm. What is
the number of poles?
A. 26 B. 24 C. 28 D. 30
Sol’n

F= P=

( )( )
P=
REGISTERED ELECTRICAL ENGINEERS PRE-BOARD EXAMINATION
MARCH 2014
PROFESSIONAL ELECTRICAL ENGINEERING SUBJECTS

P=24

5. A 240 v balanced 3ɸ source supplies a purely resistive impedance of 11.52


ohms per phase. What is the total power drawn if it is Y-connected?
A. 5 kW B. 15 kW C. 20 kW D. 30 kW

Sol’n

I= √
P= √3 VIcosθ

I=12 028 P= √3 12 028 240 cos0


P= 5KW

6. Power in a three-phase delta system with balanced load is equal to


√ LIL (p.f.) B √3 VɸIɸ(p.f.) C. 3Vɸ IL (p.f.) D. 3VLIL (p.f.)

Answer
√ LIL (p.f.)

7. What is the minimum number of wattmeters required to measure unbalanced


power for a three-phase system?
A. 2 B. 4 C. 3 D. 1

Answer:
2 wattmeter

8. Form factor is defined as


A. rms value/peak value C. rms value/average value
B. maximum value/rms value D. effective value/rms value

Answer:
rms value/average value

9. The value of form factor for a pure sine wave is


A. 1.414 B. 0.707 C. 0.637 D. 1.11
Sol’n

Formfactor = =

Formfactor= 1.11
REGISTERED ELECTRICAL ENGINEERS PRE-BOARD EXAMINATION
MARCH 2014
PROFESSIONAL ELECTRICAL ENGINEERING SUBJECTS

10. Find the horsepower rating of a motor running at 1,500 rpm and developing a
torque of 105 lb-ft.
A. 22 B. 28 C. 30 D. 36

Sol’n
P=

( )( )
P=

P= 30.925 Hp
11. There are two wired A and B. If the resistance of B is R and the diameter of A
is twice of B, the length and the nature of the material is the same, find
resistance of A.
A. R/2 B. R/4 C. 4R D. 2R

12. The breakdown voltage of the dielectric of a cable is 800 volts/mil. A factor of
safety of 4 is used. Find the maximum voltage at 3.5 mils.
A. 2,800 v B. 700 v C. 3,600 v D. 900v
Soln:

13. A wire has a resistance of 1 Ω If the length is increased four times and the
area is doubled the original, find the new resistance of the wire.
A 5Ω B 2Ω C 1Ω D. 0.5

Soln: ( )

( *

( *
REGISTERED ELECTRICAL ENGINEERS PRE-BOARD EXAMINATION
MARCH 2014
PROFESSIONAL ELECTRICAL ENGINEERING SUBJECTS

14. The basic function of a circuit breaker is to


A. produce the arc C. transmit voltage by arcing
B. ionize the surrounding air D. extinguish the arc

15. Air used in air blast circuit breaker


A. must have least carbon dioxides C. must have oil mist
B. must be ionized D. must be free from moisture

16. Two 30 Ω resistances are connected in series. When an unknown resistance


R is connected in parallel with one, the new total resistance is 40 Ω What is
the value of R?
5Ω B Ω C 20 Ω D 25 Ω

Soln:

17. A 10:1 single-phase transformer is rated 100 kVA 2,400/240 v. When the low
voltage side of the transformer is maintained at 240 v, what is the secondary
impedance?
0.676 B. 0.776 C. 0.576 D.0.876
Soln:

18. What is the counterpart of magnetic flux in an electric circuit?


A. voltage B. current
C. resistance D. conductance

19. In a parallel circuit the potential difference across the resistance


A. varies C. is sometimes constant
B. is different from the applied voltage D. is always constant

20. The rms value of a sinusoidal wave is equal to ______ of maximum value.
A. 0.637 B. 0.707 C. 0.506 D. 1.414
REGISTERED ELECTRICAL ENGINEERS PRE-BOARD EXAMINATION
MARCH 2014
PROFESSIONAL ELECTRICAL ENGINEERING SUBJECTS

21. When one coil of a magnetically coupled pair has a current of 5 A, the
resulting fluxes ɸ11 and ɸ12 are 0.4 and 0.8 mWb, respectively. If the turns are
N1 = 500 and N2 = 1,500, find L2.
A. 530 mH B. 540 mH C. 1,060 mH D. 1,080 mH

22. In two-wattmeter method, the readings of the wattmeters will be identical


when
A. load in one of the phases is zero C. power factor is 0.5
B. power factor is unity D. neutral is earthed

Answer: B. Power Factor is Unity

23. The positive, negative, and zero-sequence reactance of a 20 MVA, 31.2 kV


synchronous generator are 0.3 pu, 0.2 pu, and 0.1 pu, respectively. The
generator is solidly grounded and is not loaded. A line to ground fault occurs
on phase a. Neglecting all the resistances, determine the fault current.
A. 2,916 A B. 4,374 C. 2,525 D. 5,050
24. What is the maximum distance between two receptacles in a single family
dwelling unit?
A. 1.5 m B. 1.6 m C. 1.8 m D. 2.0 m

Answer: C. 1.8m

25. What is the minimum clearance between the recessed incandescent fixture
and the nearest point of a storage space of a closet?
A. 100 mm B. 150 mm C. 200 mm D. 250 mm

Answer: B. 150mm

26. What is the derating factor in each of 7 to 9 conductors in a raceway?


A. 80% B. 85% C. 70% D. 75%

Answer: C 70% is the derating factor in each 7 to 9 conductors

27. What is the maximum number of conductors in a raceway that does not need
to be derated?
A. 2 B. 3 C. 4 D. 5

Answer: B.3

28. Two electrons have a force of 3.46 x 106 N. What is the distance between
them?
A. 8.1 x 10-7 m B. 1.8 x 10-7 m
REGISTERED ELECTRICAL ENGINEERS PRE-BOARD EXAMINATION
MARCH 2014
PROFESSIONAL ELECTRICAL ENGINEERING SUBJECTS

C. 8.1 x 10-18 m D. 1.8 x 10-18 m

Solution.
F=3.46x109
k=9x109
Q= 1.6x10-9

( )

m C.

29. A wire 36 inches long is carrying a current of 1,200 A. Find the flux density in
gauss at a point 12 inches from the wire.
A. 2.40 B. 7.87 C. 6.31 D. 5.52

Solution

u = 4pix10-7
I = 1200A
r = 12inch x x = .3048m

( )
= 7.87x10-4
( )

30. Four resistances of 10, 20, 30 and 40 ohms are connected in parallel. What is
the equivalent resistance?
A. 100 Ω B 96Ω C 36Ω D. 4.8

Solution

[10-1 + 20-1 + 30-1 + 40-1]-1


REGISTERED ELECTRICAL ENGINEERS PRE-BOARD EXAMINATION
MARCH 2014
PROFESSIONAL ELECTRICAL ENGINEERING SUBJECTS

4.8 ohms

31. A factory works at 400-kW, 0.7 pf lagging. A 200 HP synchronous motor is


connected to the plant to correct the pc to 75% lagging. What should be the pf
of the synchronous motor?
A. 80% B.89% C. 91% D. 95%

Sol’n

Po = 400KW Psync = 200HP = 149200W


Pfold = 0.7 lag
Pfnew = 0.75 lag

( )
( )
( )

( ) ( )
149200 ( )
( )

27.09
( )

32. A transformer has an impedance of 6%. The X/R ratio is 16. Find the per unit
admittance G+jB.
A. 10.3963 + j1.6634 B. 1.0396 + j16.6342
C. 10.3963 – j1.6634 D. 1.0396 – j16.6342

Sol’n

Z = 6% = 0.06 pu
X = 16R
Z2 = R2 + X2
0.062 = R2 + (16R)2

( )
REGISTERED ELECTRICAL ENGINEERS PRE-BOARD EXAMINATION
MARCH 2014
PROFESSIONAL ELECTRICAL ENGINEERING SUBJECTS

33. A 120 v dc motors draws 225 A from the supply at a distance of 350 m using
two copper conductors of diameter 0.5 inch and resistivity 10.4 ohm-cmil/ft.
Find the voltage of the supply.
A. 126 v B. 133 v C. 138 v D. 142 v

Sol’n

D= 5 in x 1000mils/1in=500mils

( )

( )

34. A balanced delta connected load impedance of 15 j18 Ω per phase is


connected across a 230 v, three-phase source. Find the total power
consumed.
A. 10,580 W B. 1,445 W C. 4,336 W D. 3,527 W

Sol’n

( )

35. 34.5 kV to 69 kV is classified as


A. low voltage B. medium voltage
C. high voltage D. extra high voltage

Ans: High Voltage

36. A circuit has a resonant frequency of 4,000 rad/sec and bandwidth of 100
rad/sec. If the resistance is 5 ohms, find the values of L and C.
REGISTERED ELECTRICAL ENGINEERS PRE-BOARD EXAMINATION
MARCH 2014
PROFESSIONAL ELECTRICAL ENGINEERING SUBJECTS

A. 50 mH, 1.25 µF B. 1.25 mH, 50 µF


C. 5 mH, 12.5 µF D. 12.5 mH, 5 µF

Sol’n

BW = 100 rad/sec
Fr = 4000 rad/sec
R = 5 ohms

( )( )

𝐶 ( )( )( )

37. A 10 HP, 230 v, 60 Hz three-phase motor has an efficiency of 90% and pf of


0.85 lagging. What is the capacitance per phase in microfarads to be
connected in parallel with the motor correct the power factor to unity?
A. 258 µF B. 155 µF C. 25.8 µF D. 15.5 µF

Sol’n

V = 230V
Po = 10Hp = 7460W
f = 60Hz
eff = 90%
PFold = 0.85% ( )
PFnew = 1 ( )
Pin = Po/eff
Pin = 7460/0.90 = 8288.89W
C=?
( )
( ( ) ( )).89(tan31.799-tan0)

𝐶 1/2πfXc= 1/2π 60 11 44
( )( )

38. Three impedances are in parallel. The currents drawn by the impedances are
Ia= 10 A, Ib = 12 A and Ic = 15 A, respectively. If Iais the reference, Ibleads Ia
by 30º, and Icleads Ia by 45º, what is the total current?
A. 10 0º + 12 30º - 15 45º C. 10 0º - 12 30º + 15 45º
REGISTERED ELECTRICAL ENGINEERS PRE-BOARD EXAMINATION
MARCH 2014
PROFESSIONAL ELECTRICAL ENGINEERING SUBJECTS

B. 10 0º - 12 30º - 15 45º D. 10 0º - 12 30º - 15 45º

Ans: D. 10 0º - 12 30º - 15 45º

39. A 99 MHz radio station has a 150 µH inductor, what is the capacitance?
A. 0.0172 pF B. 17.2 pF C. 1.72 µF D. 17.2 µF

Sol’n

L = 150 micro henry


Fr = 99 MHz

√ √( )( )

40. An aluminum cable has 10 identical strands, a length of 1,500 m and a


diameter of 2.50 mm each. What is the resistance of the cable?
A. 5.63 Ω B 0 563 Ω C 8 63 Ω D. 0.863

Sol’n

L = 1500m
D = 2.50mm
A= 84in x 1000mils/1in=98.4mils
𝐶

41. If the speed of an Induction motor increases, what happens to the rotor
frequency?
A. Increases C. Does not change
B. Decreases D. frequency has no relation to the
speed

42. A 230 v shunt motor has a no-load line current of 7 A and a full-load line
current of 46 A. The armature resistance is 0.05 Ω and the shunt field is 75 Ω
Determine the motor efficiency at full load.
A. 81% B. 84% C. 87% D. 85%
REGISTERED ELECTRICAL ENGINEERS PRE-BOARD EXAMINATION
MARCH 2014
PROFESSIONAL ELECTRICAL ENGINEERING SUBJECTS

Sol’n:

SPL = VNL – IaNL - ( )

( )( ) ( ) ( )

( ) ( )

= 84%

43. A 40-m metallic conductor of a cross-sectional area 1 mm2 has a resistance


of 12 Ω Calculate the conductivity of the metal
A. 3.33 MS/m B. 0.333 MS/m
C. 6.67 MS/m D. 0.667 MS/m

Sol’n:

( )

44. The voltage across a 50 µF capacitor rises at constant rate of 10 V/ms.


Calculate the current through the capacitor.
A. 2 mA B. 5 mA C. 50 mA D.500 mA

Sol’n:
REGISTERED ELECTRICAL ENGINEERS PRE-BOARD EXAMINATION
MARCH 2014
PROFESSIONAL ELECTRICAL ENGINEERING SUBJECTS

𝐶 𝐶 ( )

( )

45. Express the current phasor 25 -30º as instantaneous current.


A. 17 68 sin ωt-30º) B. 35.35 sin ωt 30º)
C. 35.35 sin ωt-30º) D. 25 sin ωt-30º)

Sol’n:

46. A 7.5 HP, 500 v three-phase induction motor is developing its full-load output
at 960 rpm. The starting torque is equal to the full-load torque. What will be
the starting torque if the supply voltage fall to 400 volts?
A. 35.62 N-m B. 32.34 N-m
C. 28.27 N-m D. 32.82 N-m
Sol’n:

47. A light bulb having a tungsten filament, draws 0.5 A at 110 v. the cold
resistance of the filament is 20 Ω at 20 ºC At this temperature the
temperature coefficient of resistance for tungsten is 0.005 ºC -1. Determine the
operating temperature of the bulb.
A. 1,980 ºC B. 2,080 ºC C. 2,120 ºC D. 2,020 ºC

48. One advantage of distributing the winding in alternator is to


A. reduce harmonics C. reduce the amount of copper
B. improve voltage waveform D. decrease the value of the voltage

49. A single-circuit three-phase line operated at 60 Hz is arranged in a triangular


configuration with distances of 20 ft., 20ft. and 38 ft. between conductors, the
conductors are ACSR Drake with GMR of 0.0373 ft. Find the inductive
reactance per mile per phase.
A. 0.478 Ω B 0 578 Ω 7 Ω D 0 678 Ω

50. A 120-v shunt motor has an armature equivalent of 0.5 Ω and a field
resistance of 60 Ω At full-load, the motor takes 10 A and the speed is 1,000
REGISTERED ELECTRICAL ENGINEERS PRE-BOARD EXAMINATION
MARCH 2014
PROFESSIONAL ELECTRICAL ENGINEERING SUBJECTS

rpm. At what speed must this motor may be driven as generator to deliver 10
A to an external load at 120 v?
A. 1,158 rpm B. 1,086 rpm C. 1,262 rpm D. 1,045 rpm

Sol’n:

( ) ( )( )

( )( )

( )( )

51. The terminal voltage of a Y-connected load consisting of three equal


impedances of 20 30º Ω is 4 4 kV line to line The impedance in each of the
three lines connecting the load to a bus at a substation is Z L = 1.4 75º Ω
Find the lie to line voltage at the substation bus.
A. 4.52 kV B. 4.62 kV C. 4.48 kV D. 4.58

Sol’n:

kV
, V =4.4kV/&3=2540.34V

( ) ( )

( ) ( )( )

52. The reactance of a generator designated “X” is given as 0 25 per unit based
on the generator’s nameplate rating 18 kV, 500 MVA The base for calculation
is 20 kV, 100 MVA Find “X” on the new base
A. 0.0405 pu B. 0.0504 pu C. 0.0450 pu D. 0.0540 pu
Sol’n:
REGISTERED ELECTRICAL ENGINEERS PRE-BOARD EXAMINATION
MARCH 2014
PROFESSIONAL ELECTRICAL ENGINEERING SUBJECTS

( ) ( )u,oldV1/V2^2Sb2/Sb1

( ) ( )

53. If the voltage is doubled, the ampacity of the conductor ___________.


A. increases B. decreases
C. doubles D. remains the same

54. A 20kW, 200 v shunt generator is operated at rated load. If the driving engine
is developing 30 bhp, determine the iron and friction losses. Armature
resistance is 0.05 ohm and shunt field resistance is 50 ohms.
A. 1,340 W B. 1,039 W C. 954 W D. 1,034 W
Sol’n:

Ia = Ia + Ish = 100 + 4 = 104 A

6=22,380 W

+ Ploss

=Pout+Ia^2Ra+Ish^2Rsh+SPL

( ) ( ) ( ) ( )

( ) ( ) ( ) ( )

55. A three-phase alternator has 6 slots per pole per phase. Determine the
distribution factor.
A. 0.977 B. 0.956 C.0.963 D. 0.948
Sol’n:
REGISTERED ELECTRICAL ENGINEERS PRE-BOARD EXAMINATION
MARCH 2014
PROFESSIONAL ELECTRICAL ENGINEERING SUBJECTS

slots=6 slots/poles x phase=


( )

56. The purpose of a grid system is to


A. interconnect the power stations
B. supply energy to remote areas
C. make extra high voltage available to consumers
D. make distribution of energy at reasonable cost

57. As the load is increased the speed of a dc shunt motor will


A. remain constant C. increase proportionally
B. increase slightly D. reduce slightly

58. Power in a balanced 3-phase system is measured by the two-wattmeter


method and it is found that the ratio of the two wattmeter readings is 2 is to 1.
What is the power factor of the system?
A. 0.9 B. 0.866 C. 0.82 D. 0.707

59. A 50 HP, 440 v, 3-phase, 60 Hz, 6-pole squirrel cage induction motor is
operating at full-load and 0.8 power factor. The full-load efficiency is 85% and
the percentage slip is 5%. Determine the full-load torque.
A. 214.55 N-m B. 206.72 N-m
C. 312.47 N-m D. 323.24 N-m

Sol’n:

( )

( )

( )( )

60. A 170-mile, 230 kV, 60 Hz, three-phase single-circuit transmission line uses a
triangular arrangement with 20 ft, 20 ft and 36 ft spacing, respectively. If the
line conductors have a GMR of 0.0217 ft, determine the capacitive
susceptance to neutral per mile.
A. 4.81 x 10-6 mho per mile C. 7.92 x 10-6 mho per mile
REGISTERED ELECTRICAL ENGINEERS PRE-BOARD EXAMINATION
MARCH 2014
PROFESSIONAL ELECTRICAL ENGINEERING SUBJECTS

B. 12.74 x 10-6 mho per mile D. 2.47 x 10-6 mho per mile

61. A series RLC circuit has R = 100 ohms, L = 100 µH and C = 100 pF. What is
the frequency in MHz at unity power factor?
A. 1.59 B. 1.36 C. 15.9 D. 13.6
Sol’n:
R=100 ohms, L=100 µH and C=100 pF

1
f=
2π √ 𝐶

1
f=
2π √ ( )

f = 1.5915 Mhz

62. An impedance draws a current of i = 10 cos ωt - 30º) A from a supply with


voltage e = 220 sin ωt v What is the impedance in ohms?
A. 11 + j19.05 B. 11 – j19.05
C. 19.05 + j11 D. 19.05 – j11
Sol’n:

i = 10 cos ωt - 30º = 10 sin ωt 60º

e = 220 sin ωt

Z= = ( )

Z = 11 – j19.05

63. A balanced wye – connected load with resistance of 8 ohms per phase and
no reactance is supplied by a 240 v, 3 – phase source. Find the real power of
the load in watts.
A. 6,400 B. 7,200 C. 6,800 D.7,600
Sol’n:

P=( )2 = 7,200 w

64. If the field of a synchronous motor is under – excited the power factor will be
A. lagging B. leading C. unity D. more than unity

65. Find the efficiency of a 150 kVA transformer at 25% full-load at 0.8 p.f.
lagging if copper losses are 1,600 w at full load and iron losses are 1,400 w.
REGISTERED ELECTRICAL ENGINEERS PRE-BOARD EXAMINATION
MARCH 2014
PROFESSIONAL ELECTRICAL ENGINEERING SUBJECTS

A. 96.24% B. 94.24% C. 97.24% D. 95.24%


Sol’n:

Convert kVA to watts:


kVA x p.f.
150 kVA (.8) = 120 kW

Pin = Pout + 2Pcu + 2Pcore


= 120 kW + 2(1600) + 2(1400)
= 126000

η=

η= x 100 = 95.24%

66. An air gap 1.1 mm long and 40 sq. cm in cross-section exists in a magnetic
circuit. Determine the mmf required to create a flux of 10 x 10-4Wb in the air
gap.
A. 2.1885 AT B. 21.885 AT
C. 218.85 AT D. 2,188.5 AT
Sol’n:

R= = 218.84k
( )

T = RØ = 218.84k (10x10-4)
T = 218.84 AT

67. A three-phase, 440 v, 50 Hz, 6-pole induction motor running at 950 rpm takes
50 kW at a certain load. The friction and windage loss is 1.5 kW and stator
losses = 1.2 kW. Determine the output from the rotor.
A. 42.68 kW B. 46.48 kW C. 48.80 kW D. 44.86 kW
Sol’n:
( )
Ns = = 1000
S= = 0.05
Pr = Pin – Pcus = 50 kW – 1.2 kW
Pr = 48.8 kw
Pm = Pr (1-S) = 48.8k (1-0.05)
Pm = 46,360 W
Po = Pm - Pfw
Po = 46360 – 1.5k = 44.86 kW
REGISTERED ELECTRICAL ENGINEERS PRE-BOARD EXAMINATION
MARCH 2014
PROFESSIONAL ELECTRICAL ENGINEERING SUBJECTS

68. In DC machines, lap winding is used for


A. high voltage, low current C. high voltage, high current
B. low voltage, low current D. low voltage, high current

69. Wye-wye power transformers are protected by current transformers having


______ connection.
Δ–Δ B. Y – Δ C Δ–Y D. Y – Y

70. What is the overall efficiency of the turbine if the power developed is 12 MW
at a head of 150 meters and the nozzle discharges 10 cubic meters each
second?
A. 70.65% B. 78.99% C. 81.55% D. 89.23%

71. If supply voltage is changed by 5% for a 3ɸ induction motor, its torque will
have a change of
A. 5% B. 25% C. 10% D. 20%
Sol’n
2 (0.05) = 10 %

72. A single phase transmission line has a total impedance of 5 60º ohms and
supplies a total load of 120 A, 3.3 kW and 0.8 power factor lagging. Calculate
the sending end voltage.
A. 3.93 kV B. 3.62 kV C. 3.54 kV D. 3.86 kV
Sol’n

Vs=Vr+Ir Z
Vs=3.3 k+ (120 cis-36.87)(5cis60)
Vs=3858.97 cis 3.5
Vs=3.86kV
73. A 2.3 kV three-phase system supplies 120 – kW, 0.6 p.f. lagging balanced
load. Determine the capacitance in each phase of wye – connected capacitor
bank to adjust the power factor to 0.90 lagging.
A. 51.1 µF B. 52.2 µF C. 62.5 µF D. 78.2 µF
Sol’n
( )
Cy =
( )( )
( ) ( )
y μF

74. An impedance of 5 – j3 ohms is connected in parallel with an ideal inductor L


across a 110 v, 60 Hz mains. Determine L if the total voltage supplied the
current will be in the phase with the total current drawn.
A. 16.6 mH B. 19.2 mH C. 22.1 mH D. 26.8 mH
REGISTERED ELECTRICAL ENGINEERS PRE-BOARD EXAMINATION
MARCH 2014
PROFESSIONAL ELECTRICAL ENGINEERING SUBJECTS

Sol’n

IRC = = 22cis 35.87

L= = 22.1 mH
75. The sequence components of the line A current I aare as follows: Ia0 = 14.13
17.34º A, Ia1 = 708.26 -31º A and Ia2 = 2.98 10.06 º A. Find the line current
Ibin amperes.
A. 695 205º A B. 695 -155º A
C. 695 151º A D. 695 209º A
Sol’n
Ib = Iao + Ia1 (a2) + Ia2 (a)
Ib = 695 cis -155 A

76. Which of the following equipment is not a part of meggar?


A. hand crank C. a coil of high inductance
B. moving coil meter D. gear box generator

77. An industrial plant has an aggregate load of 100 kW. The demand factor is
60%. The average total energy consumption of the plant in one year is 186
MWHR. What is the yearly load factor of the plant?
A. 32.45% B. 42.25% C. 38.67% D. 35.39%
Sol’n
P=100kW D.F=60%
Pave =186MWHR @ 1year
PL=CL x DF
PL = (1000kW)(0.60)
PL = 60,000
L.F =
L.F =0.35388 x 100 = 35.39 %
REGISTERED ELECTRICAL ENGINEERS PRE-BOARD EXAMINATION
MARCH 2014
PROFESSIONAL ELECTRICAL ENGINEERING SUBJECTS

78. A series RL circuit in which R = 5 and L = 20 mH has an applied voltage e


= 100 50sin ωt 25sin 3ωt, with ω = 500 radians per sec Determine the
power dissipated in the resistor of the circuit.
A. 2510 w B. 2234 w C. 2302 w D. 2052w

79. Two parallel connected loads A and B are supplied by a 440 v, 3 – phase, 60
Hz generator. Load A draws and apparent power of 100 kVA at 0.80 power
factor lagging and load B draws an apparent power of 70 kVA at unity power
factor. Determine the feeder current.
A. 202 A B. 214 A C. 212 A D. 208 A
Sol’n
SA = 100kVa SB = 70 kVA
Pf = 0.8 pf = 1
P T= cis35.87
PT = 185.81 cis23.81
PT = VL IL cos
ST=( ) ( ) ( )
ST = 161.55 cis21.81
IL= = 212 cis21.8
√ ( )

80. The armature current drawn by any dc motor is proportional to


A. the voltage applied to the motor C. the flux required in the motor
B. the torque required D. the speed of the motor
Sol’n

81. Surge impedance of transmission line is given by


A.√𝐶 B. √𝐶 C. D. √

82. A solenoid 500 cm long and 10 cm in diameter is wound with 1,500 turn. Find
the energy stored in the magnetic field when a current of a 5 A flows.
A. 0.045 J B. 0.055 J C. 0.046 J D. 0.072 J
Sol’n
N1=1500 turns d=10cm L=500cm I=5A
L=
W = (4.44x10-3)(5) 2 = 0.055 J
REGISTERED ELECTRICAL ENGINEERS PRE-BOARD EXAMINATION
MARCH 2014
PROFESSIONAL ELECTRICAL ENGINEERING SUBJECTS

83. A coil of inductance 8 m H, produce 80 µWb of magnetic flux. Of this total flux
60 µWb are linked to a second coil of inductance 2mH. How much is the
mutual inductance between the coils?
A. 3 mH B. 4 mH C. 5 mH D. 2 mH

84. Calculate the capacitance between two parallel plates each of which is 100
cm2 and 2 mm apart in air.
A. 0.443 pF B. 0.515 pF C. 0.452 pF D. 0.502 pF
Sol’n
A =100cm2 , d=2mm x 1cm/10mm=0.2cm
( )( )( )( )
C=

C = 0.4427x10-12 or 0.443 pF Sol’n


A =100cm2 , d=2mm x 1cm/10mm=0.2cm
( )( )( )( )
C=

C = 0.4427x10-12 or 0.443 pF

85. The potential at the terminals of the battery falls from 9 v on open circuit to 6
volts when a resistor of 10 ohms is connected across its terminals. What is
the internal resistance of the battery?
B 4 C 3 D 2
Sol’n
I= = 0.6A
Vr = E = Vr
Vr = 9 – 6 = 3V
r= = = Ω

86. An unshaded lamp is 6 m above the table. It is lowered down by 2 m. By how


much is the illumination on the table increased?
A. 2 times B. 2.25 times C. 2.5 times D. 2.75 times
Sol’n
d =6m
E0 = K x (I/d2)
( )

( )
D2 = 3.79 m
REGISTERED ELECTRICAL ENGINEERS PRE-BOARD EXAMINATION
MARCH 2014
PROFESSIONAL ELECTRICAL ENGINEERING SUBJECTS

87. A three-phase motor driven pump at 1,800 rpm is discharging 500 gpm of
water at a head of 25 meters. The pump efficiency is 75%. Determine the HP
of the motor.
A. 15 hp B. 12 hp C. 14 hp D. 10 hp

88. The reactance of a three – phase alternator is 8%. If the alternator is rated 25
MVA at 13.25 kV output voltage, line to phase, solve for the magnitude of the
fault current generated when a short circuit occurs between 2 phases at the
terminals.
A. 6,704 A B. 6,135 A C. 6,808 A D. 6,458 A
Sol’n
Srated = 25MVA
V rated = 13.25 kV = V flux
I actual = ( )( )
=628.93

I fault = √ ( ) = 6808 A

89. For an unbalanced load which connection is suitable?


A. 3 – wire open delta connection C. 3 – wire delta connection
B. 4 – wire wye connection D. 3 – wire wye connection

90. An inductor has a value of 44 mH at 60 Hz. What is the inductive


susceptance, stated as an imaginary number?
A. –j0.06 B. j0.06 C. –j17 D. j17
Sol’n
XL=2πfL
XL= 2π 60 44x10-3 = 16 588Ω
Inductive = j = -j 0.06

91. The meter movement in an illumination meter directly measures


A. current B. voltage C. power D. energy

ANS.: A. current

92. A span of wire 1 km long has a conductance of 0.6 s. What is the


conductance of a span of this same wire that is 3 km long?
A. 1.8 s B. 0.6 s C. 0.2 s D. 0.36 s
Sol’n
dx
( ) = x= 0.2s
REGISTERED ELECTRICAL ENGINEERS PRE-BOARD EXAMINATION
MARCH 2014
PROFESSIONAL ELECTRICAL ENGINEERING SUBJECTS

93. A short 230 kV transmission line has an impedance of 5 cis 78º ohms. The
sending end power is 100 MW at 23 kV and 85% power factor. What is the
voltage at the other end?
A. 225.4 kV B. 226.3 kV C. 223.2 kV D. 228.2 kV

Sol’n
Vs = Vr + Is Z
Is = = 295.32 A
√ √ ( )( )
Er = Es – Is Z
= 230 – 295.32(5cis78)
Er = 228.9 = 228.2kV

94. A single – phase load on 220 v takes 5 kW at 0.6 lagging power factor. Find
the kVAR size of capacitor, which may be connected in parallel with this
motor to bring the resultant power factor to unity.
A. 6.64 B. 7.32 C. 8.66 D. 6.26

Sol’n
θ = cos-1 (0.6)
θ = 53 13֯
Q= ( )
= 5000 [tan (53.13) – tan (0)]
Q = 6.67 kVAR

95. A 5 kVA, 2,400 – 120/240 volt distribution transformer when given a short
circuit test had 94.2 volts applied with rated current flowing in the short
circuited wiring. What is the per unit impedance of the transformer?
A. 0.0392 B. 0.0386 C. 0.0415 D. 0.0435

Sol’n
Isc Ip = = = 2.083A
Z= = = 45 22
( )
Zpu = =
Zpu = 0.0392pu

96. A six – pole, three – phase squirrel – cage induction motor is connected to a
50 – cps supply. At full – load, the rotor’s inducted emf makes 72 complete
cycles in 1 minute. Find the rotor speed.
A. 1,200 rpm B. 1,124 rpm C. 1,176 rpm D. 1,054 rpm
REGISTERED ELECTRICAL ENGINEERS PRE-BOARD EXAMINATION
MARCH 2014
PROFESSIONAL ELECTRICAL ENGINEERING SUBJECTS

Sol’n
( )
Nr = = 1200
72 cycle /min x = 12 cycles/sec
( )
Fr =
( )
4320 =
Nr = 1176 rpm

97. A 100 µF capacitor initially charged to 24 v is discharged across a series


combination of 1 k resistor and a 200 µF capacitor Find the current after 1
sec.
A. 7.24 nA B. 7.34 nA C. 8.43 nA D. 8.84 nA

Sol’n

98. An electrical insulator can be made a conductor


A. by heating it B. by cooling it
C. by ionizing it D. by oxidizing it

ANS.: C. by ionizing it

99. A power plant consumes 100,000 pounds of coal per hour. The heating value
of the coal is 12,000 BTU per pound. The overall plant efficiency is 30%.
What is the kW output of the plant?
A. 205,000 kW B. 105,500 kW
C. 142,500 kW D. 175,000 kW

Sol’n
100,000 lb x = 1.2x10^2BTU
1.2x10 ^9BTU= = 351.579 MW
Po = (351.579)(0.3)
Po = 105,500kW

100. In a short circuit analysis, the positive, negative, and zero sequence
impedances are 0.20 pu, 0.20 pu, and 0.25 pu, respectively, using a base
MVA of 50. Estimate the fault current on the faulted lines if a line to line fault
at the 138 kV level occurs.
A. 1,204 A B. 2,718 A C. 1,569 A D. 906 A
REGISTERED ELECTRICAL ENGINEERS PRE-BOARD EXAMINATION
MARCH 2014
PROFESSIONAL ELECTRICAL ENGINEERING SUBJECTS

Sol’n

Iactual = = 209.185
(√ )

I=3( ) = 906 A
AUGUST 2014
REGISTERED ELECTRICAL ENGINEERS PRE-BOARD EXAMINATION
AUGUST 2014
PROFESSIONAL ELECTRICAL ENGINEERING SUBJECTS

1. One MWHR is equivalent to___________.


A. 8, 160 calories B. 1,000 KWHR C. 4, 186 joules D. 3,412 BTU

2. The current following in L and C at parallel resonance are


A. zero B. equal C. infinite D. different

3. What is the equivalent of acis2 + a + j?

A.1+j B. 1-j C.-1+j D. -1-j

4. In a series RLC circuit , i.e resonance is

A.maximum B. minimum C. infinity D. zero

5. An inductance of 8 mH is in series with the parallel combination of 3 mH and 6


mH. What is the equivalent inductance ?

A.4 mH B. 6 mH C. 8 mH D. 10 mH

Solution:

( )( )
+( =
) ( )

6. The current in RLC series circuit, i.e at resonance is

A. maximum B. minimum C. infinity D. zero

7. What is the maximum distance between two receptacles in a single family


dwelling unit?
A.1.5 m B. 1.6 m C. 1.8 m D. 2.0 m

8. Which of the following plants will take least time in starting from cold conditions to
full-load operation?

A. gas turbine C. steam power plant


B. Nuclear power plant D. hydroelectric

9. The Value of mutual inductance in terms of self-inductance of the two coils L1


and L2 is proportional to

A. L1L2 B. L1/L2 C. D. √
Solution:
REGISTERED ELECTRICAL ENGINEERS PRE-BOARD EXAMINATION
AUGUST 2014
PROFESSIONAL ELECTRICAL ENGINEERING SUBJECTS

10. When one coil of a magnetically coupled pair has a current of 5 A, the result
fluxes and are 0.4 and o.8 mWb, respectively. If the turns are N1= 500
and N2= 1,500, find M.
A. 120 mH B. 20 mH C.240 mH D. 40 mH

Solution:
( )( )

11. Which of the following relays is used in transformers?


A. Buchholz relay B. Mho relay C. Merz- Price relay D. Translay relay

12. The heat required to raise 1 gram of water at 1 degree Celsius.


A. Calorie B. BTU C. joule D. KWHR

13. In how many years will the energy consumption of the Philippines be quadrupled
at 6.9 percent growth rate?
A. 5 B. 10 C. 15 D. 20

14. Heating value of coal largely depends on


A. Ash content C. Volatile matter
B. Moisture content D. Size of coal particles

15. An inductance of 40 mH is connected in series with 300 ohms resistance, At t=0,


the energy stored is 0.96 microjoule. Find the current at t=0.
A. 0.44 x 10^-3 A C. 13.86 x 10^-3 A
B. 0.22 x 10^-3 A D. 6.93 x 10^-3 A

Given:
L=40mH
R=300Ω
W=0.96 J
Sol’n.
W=
0.96= ( )( )
I = 6.93x A
REGISTERED ELECTRICAL ENGINEERS PRE-BOARD EXAMINATION
AUGUST 2014
PROFESSIONAL ELECTRICAL ENGINEERING SUBJECTS

16. For open circuited condition of Thevenin’s theorem, all sources of emf in the
network are replaced by
A. their internal impendance C. as a total a big source of emf
B. their internal resistance D. their internal reactance

17. A hydroelectric generating station is supplied from a reservoir of capacity 5 x


10^6 m at a head of 200 meters. Find the total energy available in KWHR if the
overall efficiency is 75 .
A. 1.044 x 10^6 C. 3.044 x 10^6
B. 2.044 x 10^6 D. 4.044 x 10^6

o ’n:
Weight of water available is W= x Density
=
=
Electrical Energy Available= WH

18. A direct lighting fixture directs at least


A. 90 of the light downward C. 85 of the light upward
B. 85 of the light downward D.80 of the light downward

19. One decibel represents a power ratio of


A. 10:1 B. 20:1 C. 2:1 D. 1:26:1

20. A generating station to supply four regions of load whose peak loads are 10 MW,
5 MW, 8 MW, 7 MW. The diversity factor at the station is 1.5 and the average
annual load factor is 60 . Calculate the annual energy supplied by the station.
A. 150.12 x 10^6 KWHR C. 15.012 x 10^6 KWHR
B. 10.512 x 10^6 KWHR D. 105. 12 x 10^6 KWHR

Given:
REGISTERED ELECTRICAL ENGINEERS PRE-BOARD EXAMINATION
AUGUST 2014
PROFESSIONAL ELECTRICAL ENGINEERING SUBJECTS

Diversity = 1.5
Load Factor = 60%=0.6
Annual energy supplied = ?

Sum of individual max. demands = = 30,000


Max. Demand
=
Annual energy supplied =
=
=

21. Calculate the distance from a normally placed screen a 30 candela lamp be
placed in order that the illumination be 5 lux.
A. 2.45 m B. 4.45 m C. 5.25 m D. 3.77 m

Solution:

d=√ = 2.45 m

22. Semiconductors like silicon and germanium are


A. Tetravalent B. Monovalent C. Trivalent D. Pentavalent

23. In a 13.8 KV, 10 MVA base three phase system a phase to phase fault occurs. If
the Thevenin’s equivalent impendance of the system is 2 63 , what is the
magnitude of the fault current?
A. 13,800 A B. 15,900 A C. 27,600 A D. 7,970 A

Solution:

)
( )
√ ( )(

24. A 50- KVA single phase transformer has a full-load a copper loss of 560 w and
iron loss of 500 w. Calculate the efficiency at 25 of full-load for a power factor
of 0.8 lagging.
A. 93.9 B. 94.9 C. 95.9 D. 96.9
REGISTERED ELECTRICAL ENGINEERS PRE-BOARD EXAMINATION
AUGUST 2014
PROFESSIONAL ELECTRICAL ENGINEERING SUBJECTS

25. A three- phase induction motor is taking 40 kW, the slip being 4 , stator losses
amount to 800 watts, calculate the total power developed.
A. 39.200 KW B. 40.800 KW
C. 37.632 KW D. 36.732 KW

Solution:
Pm = Pr-Pcu = 40000-800 = 39,200 (1-0.04
Pm = 37,632 KW

26. A 500 volts 1 synchronous motor is taking 50 amperes at a p.f. of 0.8 leading.
The output is 25 Hp and the effective resistance of the armature is 0.25 ohm.
Find the iron and friction losses.
A. 375 w B. 650 w C. 725 w D. 175 w

Solution:
Pm= VLIL cos theta-(Ip2Ra+Pstray)
25(746) = 500(50)(.8)-(50)2(0.25)-Pstray
Pstray = 725 W

27. What is the copper loss at full-load of a transformer if its copper loss at 7/8th full-
load is 4,900 w?
A. 5,600 w B. 3,750 w C. 6,400 w D. 375 w

Solution:
Pcu =4900/(7/8)2 = 6400W

28. If three transformers in - are delivering rated load and one transformer is
removed, then overload on each of the remaining transformers is ________
percent.
A. 66.6 B. 173.2 C. 57.7 D. 73.2

29. The output of a 200-volt 1 synchronous motor, taking a current of 20 amperes is


4 HP, its effective armature resistance is 0.5 ohm. The iron and friction losses
amount to 400 watts. What is the motor power factor?
A. 0.896 B. 0. 846 C. 0.796 D.0.916

Solution:
REGISTERED ELECTRICAL ENGINEERS PRE-BOARD EXAMINATION
AUGUST 2014
PROFESSIONAL ELECTRICAL ENGINEERING SUBJECTS

Pm = VLIL cos theta-(Ip2Ra+Pstray)


4(746) = 200(20) cos theta- 202(.5)-400
Cos theta = p.f. = 0.896

30. In a half-wave rectifier, if ac supply is 60 Hz, the ac ripple is__________.


A.120 Hz B. 60 Hz C. 30 Hz D. 15 Hz

31. The three unbalanced currents are : Ia= 10cis(-30 ), Ib=0, Ic=10cis150 . Find the
phase C positive sequence current.
A. 5.77 A B. 5.77cis120
C. 8.66cis120 D. 5.77cis240

Ans. Ia1 = 1/3 [IA+ a IB + a^2Ic]


=1/3 [(10 angle -30) + (1 angle 120) (0) + (1 angle 240) (10 angle 150)]
= 5.77
Ic1 =aIa1 = (1 angle 120) (5.77)
Ic1 = 5.77 angle 240 A

32. A coil of inductance 10 H and resistance 2 ohms is supplied at 20 v, Find the


average emf induced when the circuit is opened if the current fails to zero in
0.5sec.
A. 20V B. 10V C. 100V D. 200V

Ans. L equals 10H R equals 2 E equals 20 V


I= 0 @ t equals 0.5
i= E/R [1-e ^–R/L (t) answer 200 Volts

33. A 200V, three-phase source is connected to two equal impedance of 7.07 + j7.07
ohms in parallel and with both connected in wye configuration. What is the
current per phase of the combination?
A. -16.26 – j16.26 A C. 16.26 – j16.26 A
B. 16.26 + j16.26 A D. -16.26 + j16.26 A

34. A dc machine has 8 poles and a rated current of 100A. How much current will
flow in each conductor at rated condition if the armature is simplex lap wound?
REGISTERED ELECTRICAL ENGINEERS PRE-BOARD EXAMINATION
AUGUST 2014
PROFESSIONAL ELECTRICAL ENGINEERING SUBJECTS

A. 6.25A B. 50A C. 25A D. 12.5A

Ans. A = mP = 1(8) = 8 Ic = 100/8 = 12.5A

35. A 2-μF capacitor, with initial charge of 100μC is connected across a 100Ω
resistor at t=0. Calculate the time in which the transient voltage across the
resistor drops from 40 to 10 volts.
A. 27.7 ms B. 0.277ms C. 2.77ms D. 277ms

Ans. E0 =Q0/c = 100MC/2MF = 50V


i = E/R =40/100 = 6.4
i 10/100 = 0.1 ERE e^t/RC
0.1 = 0.4 (e ^t/100 (2 x 10^6) )
Answer T = 0.277 ms

36. In parallel RC circuit as the value of frequency increases the power factor _____.
A. Remains the same C. decreases
B. Becomes unity D. increases

37. A 400-volt, 50Hz star connected induction motor takes a line current of 50
amperes at a p.f of 0.8 lagging. Three mesh connected capacitors are connected
to raise the p.f to 0.9. Find the value of each capacitor in microfarads.
A. 146.7 B. 48.9 C. 16.3 D. 24.5

Solution:

Cy = Pt(tanteta1-Tanteta1)/squaroot3(VL^2)W\
Cy=(400)(50)(0.8)(Tan(cos^-1(0.8))-Tan(cos^1(0.9)/squareroot3(400)(314)
Cy = 48.9 uF

38. Two capacitors C1 and C2 are placed in series. If C1= 100 F, C2= 50 F, find
the maximum energy stored when a 220 v dc supply is applied across the
combination.
A. 0.107 J B. 0.507 J C. 0.807 D. 1.107 J

Solution:

CT = c1(C2)/C1(C2)
= 100(50)/100+50 = 33.33 uF
REGISTERED ELECTRICAL ENGINEERS PRE-BOARD EXAMINATION
AUGUST 2014
PROFESSIONAL ELECTRICAL ENGINEERING SUBJECTS

W = ½ CE^2
W = ½ (33.33 uF)(200) Answer W = 0.807J

39. A 2- element series circuit consumes 700 w and has a p.f of 0.707 leading. If the
applied voltage is v= 141 sin(314t + 30 ), find the circuits constants.
A. R=7.14 , C= 44.6 F C. R= 71.4 , C= 44.6
B. R= 7.14 , C= 446 D. R= 71.4 C= 446 F

Solution:
P = 700w p.f= 0.707
V =141 sin (314t +30)
W =314
F =50 Hz
Vrms = 141(0.707) = 99.7V
S =700/0.707 angle negative cos^-1(0.707) S = 700-j700,21
R = (141/squaroot2)^2/700.21 =14.2/2 = 7.1
Xc =(141/square root 2)^2/70.21 = 14.2/2 = 7.1
Xc = / 2pi(50)(7.1) =448X10^-6
Answer: R = 7.1 ohms C =448X10^-4

40. Two electrons in a vacuum experience a force of 2 x 10^-15 N. How apart are the
electrons?
A. 2.84 x 10^-12 m C. 9.6 x 10^-7 m
B. 3.4 x 10^-7 m D. 5.68 x 10^-12 m

Solution:

Q electron = 1.61X10^-19
F = KQ^2/d^2
d = square roots (1.61X10^-19)(9X10^9)/2X10^-15
Answer d = 3.41X10^-7

41. What is the current per phase in the primary windings of a 3 phase / Y
connected 6,600/400 – V 500- KVA, 50 Hz transformer. Neglect no load current.
A. 43.74 A C. 25.25 A
B. 75. 76 A D. 14.58 A
REGISTERED ELECTRICAL ENGINEERS PRE-BOARD EXAMINATION
AUGUST 2014
PROFESSIONAL ELECTRICAL ENGINEERING SUBJECTS

42. It is desired to correct the 2,400 – KVA 0.65 Lagging power factor load in a plant
unity by the installation of a synchronous condenser. Calculate the KVA rating of
the latter.
A. 2,400 B. 1,825 C. 1,560 D. 1,960

43. For a 36-slot 4-pole three phase full-pitch distributed winding, what is the percent
reduction in the generated voltage?
A. 4 B. 3 C.6 D. 5

44. Three 10:1 transformers are connected / Y for stepping up the 2,300 – volt
three-phase source. Calculate the secondary line voltage.
A. 23,000 v B. 69,000 v C. 46,000 v D. 40,000 v

Solution:

2300(5.77)(3)=3918 = 40,000

45. A transformer is rated at 550 / 220 v. With the low voltage winding shorted, it
requires 25 v across the 550 v winding to permit rated current through both
windings of the transformer. Calculate the percent impedance.
A. 4.55 B. 6.55 C. 3.55 D. 5.55

46. A 3- phase overhead transmission line is being supported by three disc


insulators. The potentials across top unit and middle unit are 8 KV and 11 KV,
respectively. Calculate the ratio of capacitance between pin and earth to the self-
capacitance between pin and earth to the self-capacitance of each unit.
A. 0.475 B. 0.775 C. 0.375 D. 0.275

47. This device function is used to reflect back surgest into the line.
A. Choke coil B. Arc horns
C. Surge depressors D. Delon gaps

48. A 50 KW carrier is to be modulated to a level of 85 . What is the carrier power


after modulation?
A. 58.8 KW B. 18 KW C. 50 KW D. 42.5 KW

49. What is the device function number of overvoltage relay?


A. 59 B. 76 C. 27 D. 67
REGISTERED ELECTRICAL ENGINEERS PRE-BOARD EXAMINATION
AUGUST 2014
PROFESSIONAL ELECTRICAL ENGINEERING SUBJECTS

50. A short 3 line with an impedance of 6 + j8 ohms per line has sending and
receiving end line voltages of 120 and 110 KV respectively for some receiving end
load at a p.f. 0.9 lagging. The active power at the receiving end is nearest to
A. 100,200 KW B. 200,000 KW
C. 120,000 KW D. 110,200KW

51. A farmer has a small stream on his property which he thinks might supply
enough power to light his building. The stream is gauged and found to be flowing
0.03m2 per second. The available fall is 2.1m. A small hydraulic turbine and
generator will have a combined efficiency of 70%. Taking the transmission line
efficiency as 80%, how many 50W lamps can be connected to the generator
simultaneously?
A. 5 B. 6 C. 7 D. 8

52. Two long straight conductors each carrying an electric current of 5A are kept
parallel to each other at a separation of 2.5cm. Calculate the magnitude of magnetic
force experienced by 10cm of a conductor.
A. 2X10-3N B. 2X10-4N C. 2X10-5N D. 2X10-6N

Solution:
( )( ) ( )( )

53. A 400V, 10KW series motor drives a fan when running at 800 rpm. The motor
draws 50A from the supply The resistance of the armature and series field are 0 2Ω
and 0 1Ω, respectively Determine the electromagnetic torque developed by the
motor.
A. 225N-m B. 230N-m C. 235N-m D. 240N-m

Solution:
: ( ) ( )
( )
( ) ( )
( )
REGISTERED ELECTRICAL ENGINEERS PRE-BOARD EXAMINATION
AUGUST 2014
PROFESSIONAL ELECTRICAL ENGINEERING SUBJECTS

54. The load of a certain transformer is reduced to 50%, the copper loss would
become
A. 0.25 times B. 0.50 times
C. 2 times D. same as before

55. At a slip of 4% the maximum possible speed of 3Ø, 60HZ squirrel cage induction
motor is
A. 3,600 rpm B. 3,456 rpm C. 3,200 rpm D. 3,000 rpm

Solution:

( )
( ) ( )

56. An electric heater develops a power of 1KW when run on 230V supply. If the
resistance wire of the heater has a diameter of 0.5mm and a resistivity of 60x10 -8Ω-
m, find the minimum length of the wire required.
A. 11.33 m B. 13.3 m C.15.3 m D. 17.3 m

Solution:

( )
( *

57. Each conductor of a horizontally arranged two- conductor bundle line is ACSR,
1,272 MCM Pheasant. The spacing between bundle centers is 8m, 8m and 16m and
between bundle conductors is 45 cm. Find the inductive reactance of the line in
ohms per km per phase if GMR of each conductor is 0.0466 ft.
A. 0.165 B. 0.265 C. 0.365 D. 0.465

58. Shunt generators are used where the main requirement is constant
A. current and voltage C. voltage over a narrow load range
B. current D. voltage over a wide load range
REGISTERED ELECTRICAL ENGINEERS PRE-BOARD EXAMINATION
AUGUST 2014
PROFESSIONAL ELECTRICAL ENGINEERING SUBJECTS

59. A 4-pole lap-wound armature has 480 conductors and a flux per pole of 25mWb.
What is the emf generated, when running at 600 rpm?
A. 240V B. 120V C. 60V D.30V

Solution:
( )( )( )

60. Transmission voltage level of 1,000kV is categorized as


A. Low Voltage C. Extra High Voltage
B. High Voltage D. Ultra High Voltage

61. The efficiency of thermal plant is approximately


A.10% B. 30% C. 60% D. 80%

62. When a line to ground fault occurs, the current in a faulted phase is 100A. The
zero sequence current in the case will be
A. zero B. 66.6 A C. 33.3 A D. 100A

Solution:

63. In which type of faults given below all the 3 components Ia0, Ia1 and Ia2 are equal?
A. Single line to ground fault C. Double line to ground fault
B. Line to line fault D. Three phase fault

64. A 220V series motor takes 10A and runs at 600 rpm. The total resistance is
0 8Ω At what speed will it run when a 5Ω resistance is connected in series, the
motor taking the same current at the same supply voltage
A. 456 rpm B.458 rpm C. 460 rpm D. 462 rpm

65. An electric heater takes 1.2 kWhr in 30 minutes at 120V. What is the current
input to the heater?
A. 20 A B. 15 A C. 10 A D. 5 A

Solution:
REGISTERED ELECTRICAL ENGINEERS PRE-BOARD EXAMINATION
AUGUST 2014
PROFESSIONAL ELECTRICAL ENGINEERING SUBJECTS

( )( )( )( )

66. When the relative permeability of a material is slightly less than 1, it is


called_____ material.
A. non-magnetic B. paramagnetic C. ferromagnetic D. diamagnetic

67. A coil of 50 turns having a mean diameter of 3cm is placed coaxially at the
center of a solenoid 60cm long wound with 2,500 turns and carrying a current of 2A.
What is the mutual inductance of the arrangement?
A. 18.5 mH B. 1.85 mH C. 0.185 mH D. 185 mH

68. The rms value of sinusoidal ac current is equal to its value at an angle of ______
degrees.
A. 45 B. 30 C. 90 D. 60

69 Two capacitors of 0 1μF, 250V and 0 2μF, 500V are connected in series What is
the maximum voltage that can be applied across the combination?
A. 375V B. 750V C. 250V D. 500V

Solution:
( ) ( )( )

70. A current of 20A flows through two ammeters A and B joined in series. Across A,
the potential difference is 0.2V and across B, it is 0.3V. Find how the same current
will be divided between the two when they are joined in parallel.
A. 10 A, 10 A B. 12 A, 8 A C. 14 A, 6 A D. 16 A, 4 A

Solution:
( )( )

( )( )
REGISTERED ELECTRICAL ENGINEERS PRE-BOARD EXAMINATION
AUGUST 2014
PROFESSIONAL ELECTRICAL ENGINEERING SUBJECTS

71. An office 17m by 8m wide, is lighted by 8 lamps has a luminous intensity of 200
Cd. Allowing an absorption loss of 4,000 lm for the reflectors, walls and ceiling,
Calculate the average illuminations on the working plane.
A. 147.9 lm/m2 B. 132.6 lm/m2 C. 118.4 lm/m2 D. 112.8 lm/m2

72. One of the advantages of distributing the winding in alternator is to


A. reduce harmonics C. reduce the amount of copper
B. improve voltage waveform D. decrease the value of the voltage

73. A circuit breaker will normally operate


A. when the switch is put on C. when the power is to be supplied
B. when the line is to be checked D. whenever the fault in line occurs

74. Two-wattmeters can be used to measure three-phase power for a


A. balanced and unbalanced load C. balanced load only
B. unbalanced load only D. unity power factor only

75. A shunt motor runs at 500 rpm on a 200V circuit Its armature resistance is 0 5Ω
and the current taken is 30A in addition to field current. What resistance must be
placed in series in order that the speed may be reduced to 300 rpm, the current in
the armature remaining the same.
A 4 37 Ω B. 3 37 Ω 2 47 Ω D 1 47 Ω

Solution:

Eb1/N1 = Eb2/N2
Eb1= 200- (30) (0.5)
= 165V
Eb2 = 185 (300) / 500
= 111V
Ia2 = 111-200/-30
2 7 2 47

76. A transformer has 4% resistance and 6% reactance drop. Find the voltage
regulation at full-load unity power factor
A. 2% B. 3% C. 4% D. 5%

77. What is the nature of the current flowing in the armature of dc machine?
A. pulsating dc B. pure dc
REGISTERED ELECTRICAL ENGINEERS PRE-BOARD EXAMINATION
AUGUST 2014
PROFESSIONAL ELECTRICAL ENGINEERING SUBJECTS

C. rectified ac D. alternating current

78. The current in the circuit is given by i=50sinωt If the frequency be 25Hz, how
long will it take for the current to rise to 25 amp?
A. 0.02 sec B. 3.33x10-3 sec C. 0.05 sec D. 0.033 sec

79. A single phase transformer when supplied from 220V, 50 Hz has eddy current
loss of 50W. If the transformer is connected to a voltage of 330V, 50Hz, the eddy
current loss will be
A. 168.75W B. 112.5W C. 75W D. 50W

Solution:
Peddy = ke(Eg) = (1.033) (330)² = 112.494W

80 The resistance of the field coil of a motor is 200Ω at 15 . After the motor worked
for a few hours on full load, the resistance increases to 240Ω Calculate the
temperature rise of the load coil assuming the temperature coefficient of resistance
is 0.0042 per at 0 .
A. 20.6 B. 30.6 C. 40.6 D. 50.6

Solution:

t1 = 1/t0+T = = 3 951X10ˉ³ ; R = R1 1 (X))


( )

240 = 200 (1 3 951X10ˉ³X ;

X = 50.62 °C

81. A magnetic field exist around


A. copper B. moving charges C. iron D. aluminum

82. Inductance affects the direct current flow at the time of


A. turning on and off C. turning on
B. operation D. turning off

83. A power plant is said to have had a use factor of 48.5% and a capacity factor of
a 42.4%. How many hours did it operate during the year?
A. 7,660 B. 8,660 C. 5,660 D. 6,660
REGISTERED ELECTRICAL ENGINEERS PRE-BOARD EXAMINATION
AUGUST 2014
PROFESSIONAL ELECTRICAL ENGINEERING SUBJECTS

Solution:

84. Surge impedance of transmission line is given by


A √C/L B √CL C 1√L/C D √L/

85. In a radio a gang condenser is a type of


A. air capacitor B. paper capacitor
C. ceramic capacitor D. electrolytic capacitor

86. A single- phase transmission line has a total resistance of 1 5Ω and a loop
reactance of 0 3Ω It delivers 2MW at 11kV at a power factor of 0 8 lagging.
Calculate the line voltage at the sending end.
A. 11.21kV B. 11.11kV C. 11.41kV D. 11.31kV

87 Three 40Ω non-inductive resistances are connected in delta across 200V, 3-


phase lines. Calculate the power taken from the mains if one of the three resistors is
disconnected.
A. 500W B. 1,000W C. 1,500W D. 2,000W

Solution:
REGISTERED ELECTRICAL ENGINEERS PRE-BOARD EXAMINATION
AUGUST 2014
PROFESSIONAL ELECTRICAL ENGINEERING SUBJECTS

88. Power in a balanced three-phase system is measured by the two wattmeter


method and it is found that the ratio of the two readings is 2:1. What is the power
factor of the system?
A. 0.5 B. 0.8 C. 0.866 D. 1

Solution:

= √

= √

89. A material commonly used for shielding or screening magnetism is


A. brass B. aluminum C. copper D. soft iron

90. One advantage of secondary cell is that it


A. can be recharged C. it is compact, easy to carry
B. can be used for portable equipment D. cannot be recharged

91. Calculate the capacitance of a parallel plate capacitor having 20cm x 20cm
square plates separated by a distance of 1mm. Assume the dielectric medium to be
air.
A. 3.54pF B. 35.4pF C. 354pF D. 3,540pF

Solution:

( )( )

92. Calculate the self-inductance of an air-cored solenoid, 40cm long having an area
of cross-section 20cm2 and 800 turns.
A. 2.42mH B. 4.02mH C. 5.20mH D. 6.40mH

Solution:
REGISTERED ELECTRICAL ENGINEERS PRE-BOARD EXAMINATION
AUGUST 2014
PROFESSIONAL ELECTRICAL ENGINEERING SUBJECTS

( )( )( )

93. A three-phase induction motor having an input power of 30KW running at a slip
of 4%. The stator loss is 1KW and friction and windage loss is 1.5KW. Determine the
motor efficiency.
A. 86.8% B. 87.8% C. 88.8% D. 89.8%

94. An iron core has a cross-section of 500mm2and having a length of 100cm. A


magnetizing force of 500AT in it produces a magnetic flux of 400µWb. Determine the
relative permeability of the material.
A. 1,275 B. 1,250 C. 1,225 D. 1,200

Solution:

( )( )( )( )

95. Find the capacitive susceptance per mile of single-phase line operating at 60Hz.
The conductor is partridge with an outside diameter of 0.642in and spacing is 20ft.
between centers.
A. 4.10x10-6 mho/mile to neutral C. 5.10x10-6 mho/mile to neutral
-6
B. 4.90x10 mho/mile to neutral D. 5.90x10-6 mho/mile to neutral

96. The reactance generator having a subtransient reactance of 0.15pu and


operating at 5 percent above its rated voltage supplies a synchronous motor having
a 0.2pu subtransient reactance. The motor is connected to the generator by a
transmission line and a transformer of total reactance 0.305pu. A sudden three-
phase short circuit occurs at the generator terminals. Determine the per unit
subtransient fault current.
A. –j6.443pu B. –j7.443pu
C. –j9.079pu D. –j10.079pu
REGISTERED ELECTRICAL ENGINEERS PRE-BOARD EXAMINATION
AUGUST 2014
PROFESSIONAL ELECTRICAL ENGINEERING SUBJECTS

97. A synchronous generator having a subtransient reactance of 0.15 pu and


operating at 5 percent above its rated voltage supplies a synchronous motor having
a 0.2 pu subtransient reactance. The motor is connected to the generator by a
transmission line and a transformer of total reactance 0.305 pu. A sudden three-
phase short circuit occurs at the generator by a transmission line and a transformer
of total reactance of 0.305 pu. A sudden three phase short circuit at the generator
terminals. Determine the per unit subtransient fault current.
A. –j6.443 pu B. –j7.443 pu C. –j9.079 pu D. –j10.079 pu

98. What type of core is used for a high frequency transformer?


A. Air core B. Aluminum core
C. Closed iron core D. Open iron core

99. For a single line to ground fault the zero sequence current is given by j3.0pu.
The current carried by the neutral during the fault is
A. j1.0pu B. j3.0pu C. j9.0pu D. j6.0pu

Solution:

( )

100. An inductor supplied with 100V ac with a frequency of 10kHz passes a current
of 10mA. The value of inductor is
A. 1.7 mH B. 16 mH C. 1 mH D. 160 Mh

Solution:
MARCH 2015
REGISTERED ELECTRICAL ENGINEERS PRE-BOARD EXAMINATION
MARCH 2015
PROFESSIONAL ELECTRICAL ENGINEERING SUBJECTS

1. A material best suited for manufacturing of fuse wire is


A. Aluminum B. Silver C. Lead D. Copper

2. In the Fleming’s left hand rule, thumb always represents


A. voltage B. current
C. direction of force on conductor C. magnetic field

3. The net reactance in a series RLC circuit is


A. XL B. XC C. XC + XL D. XL – XC

4. Two resistors R1 and R2 are connected in parallel. R1 is twice of R2. If


the current in R1 is I, what is the current in R2?
A. I B. 2I C 4I D. I/2
SOLUTION:
R1 = 2 R2 I2 = I (2R2) / R2
If R1 = I I2 = 2I
V1 = V2
I1 R1 = I2 R2

5. The ampere-hour capacity of battery depends on


A. The thickness of the plates B. the strength pf the electrolytes
C. The area of the plates D. the distance between the
plates

6. A Y-∆ connected transformer has a turns ratio of 1 If the primary line


current is 5 amperes, what is the secondary line current?
A. 5 A B. 15 A C. 8.66 A D. 2.89 A
SOLUTION:
ILA = sqrt(3) x (5 A) = 8.66 A
REGISTERED ELECTRICAL ENGINEERS PRE-BOARD EXAMINATION
MARCH 2015
PROFESSIONAL ELECTRICAL ENGINEERING SUBJECTS

7. In a certain country the energy consumption is expected to double in


10 years. Calculate the growth rate.
A. 4% B. 3.96% C. 13.86% D.6.93%
SOLUTION:
Q = Q0 ekt
2 = I ek(10)
k= 6.93%

8. The condition in Ohm’s Law is that


A. ratio V/I should be constant
B. current should be proportional to voltage
C. the temperature should remain constant
D. the temperature should vary

9. Two single phase transformers with equal ratings and turns ration are
operated in parallel to supply a load of 300 KVA, 0.80 lagging power
factor. Transformer A has a resistance of 2.5% and reactance of 5.5%.
Transformer B has a resistance of 2.0% and a reactance of 5.5%. Find
the KVA load of each transformer.
A. 147.7, 152.2 B. 148.7, 151.5
C. 149.7, 150.5 D. 150.7, 149.5
SOLUTION:
Z1 = sqrt.( 0.0252 + 0.0552 ) = 0.060415
Z2 = sqrt.( 0.022 + 0.0552 ) = 0.058523
S1 = (300)(0.060415 / 0.060415 + 0.058523) = 152.39 kVA
S2 = (300)(0.058523 / 0.060415 + 0.058523) = 147.61 kVA

10. If an electrical network having one or more than one voltage source, is
transformed into equivalent electrical network with a single voltage
source (which is short circuit current of previous circuit) with parallel
REGISTERED ELECTRICAL ENGINEERS PRE-BOARD EXAMINATION
MARCH 2015
PROFESSIONAL ELECTRICAL ENGINEERING SUBJECTS

internal resistance of the network with all current source replaced by


their internal resistance. The above illustration is called
A. Thevenin’s theorem B. Reciprocity Theorem
o ton’s th o m D. Superposition theorem

11. What is the power given V = 4 sin3t and I = 4 sin3t?


A. 16 sin(3t)2 B. 16 sin2(3t)
C. 8(1-6cos3t) D. 16(1-6cos3t)
SOLUTION:
P = (4 Sin 3t)( 4 Sin 3t) = 16 Sin2 3t

12. A 40µF capacitor is supplied with a voltage of v(t) = 100sin377t. Find


the current equation.
A. 1.51sin(377t - 90º) B. 1.51sin377t
C. 1.51cos(377t+90º) D. 1.51cos377t
SOLUTION:
40x10-3 ( (100) ( 377 cos 377t) + ( sin 377t) (0)) = 1.51 cos 377t

13. A 20mH inductor draws a current of i(t) = 100sin100t. Find the voltage
equation.
A. 200sin100t B. 200cos100t
C. 2000sin100t D. 2000cos100t
SOLUTION:
20x10-3 ( 100(100 cos 100t) + (sin 100t) (0)) = 200 cos 100t

14. For a three phase unbalanced load


A. The power factor of each phase will be proportional to the load
B. The power factor each phase will be the same
C. The power factor of at least one of the phases must be leading
D. The power factor of each phase may be different
REGISTERED ELECTRICAL ENGINEERS PRE-BOARD EXAMINATION
MARCH 2015
PROFESSIONAL ELECTRICAL ENGINEERING SUBJECTS

15. Cellular phones operate in the frequency range of


A. EHF B. VHF C. UHF D. SHF

16. A medium unshaded lamp hangs 8 m directly above the table. To what
distance should it be lowered to increase the illumination to 2 times its
former value?
A. 2.34 m B. 5.66 m C. 4 m D. 4.64 m
SOLUTION:
E1 / 2 E1 = (d2 / 8)2 d2 = 8 ( sqrt.(1/2)) = 5.66 m

17. The resistance of a copper wire at 30ºC is 50Ω If the temperature


coefficient of copper at 0ºC is 0.00427, what is the resistance at
100ºC?
A. 72 26 Ω B 24 Ω C 54 25 Ω D 58 15 Ω
SOLUTION:
T = 1 / ɑ = 1 / 0 00427 = 234 19 deg C
R2 = ((100 + 234.19) / (30 + 234.19)) (50) = 2 Ω

18. What is the maximum allowable voltage drop for lighting circuit?
A. 3% B. 5% C. 2% D. 10%

19. What is the maximum allowable voltage drop for feeder?


A. 1% B. 2% C. 3% D. 5%

20. Fiber optic operates on the principle of


A. Refraction B. Reflection
C. Diffraction D. Dispersion
21. color is reserved for grounding equipment wire only.
A. white B. green C. yellow D. grey
REGISTERED ELECTRICAL ENGINEERS PRE-BOARD EXAMINATION
MARCH 2015
PROFESSIONAL ELECTRICAL ENGINEERING SUBJECTS

22. Overhead lines are protected from lighting by the use of


A. I only B. II only
C. either I or II D. both I and II

23. An inductance of 20 mH is connected in parallel with a capacitance of


80 µF. What is the frequency when XL = XC?
A. 100 HZ B. 150 HZ C. 128.5 HZ D. 125.8 HZ
SOLUTION:

√( )( )

24. A horizontal powerline carries a current of 90 A. Compute the magnetic


field at a point 1.5 m below the line.
A. 1.2 x10-5 T B. 1.2 x10-4 T C. 1.2 x10-3 T D. 1.2 x10-2T
SOLUTION:

( )
( )( )
( )

25. The syncroscope is an instrument used to indicate the


A. Speed of motor
B. Phase relationship between the current and voltage
C. Phase relationship between two voltages
D. Phase relationship between the true power and the reactive power
REGISTERED ELECTRICAL ENGINEERS PRE-BOARD EXAMINATION
MARCH 2015
PROFESSIONAL ELECTRICAL ENGINEERING SUBJECTS

26. A three-phase transmission line of a two-conductor bundle is arranged


horizontally with a spacing of 8 m between bundle centers. Each
conductor is ACSR, 1,272,000 cmils Pheasant with GMR of 0.0466 ft.
the spacing between conductors of bundle is 45 cm. Find the per unit
series reactance of the line if its length is 160 km and base is 100
MVA, 345 KV.
A. 0.069 pu B. 0.059 pu C. 0.049 pu D. 0.039 pu
SOLUTION:
√( )( )

( )
( )( )
( ⁄ ) ( )
( )( )

27. The current flowing in the armature conductors of a dc motor is


A. ac B. dc C. ac as well as dc D. transients

28. A string insulator has 5 units. The voltage across the bottom most until
is 30% of the total voltage. What is the string efficiency?
A. 60% B. 66.67% C. 75% D. 83.33%

SOLUTION:

( ) ( )

29. For a single line to ground fault the zero sequence current is given by
j3.0 pu. The current carried by the neutral during the fault is
A. j 1 pu B. j 3 pu C. j 9 pu D. j 6 pu
SOLUTION:
REGISTERED ELECTRICAL ENGINEERS PRE-BOARD EXAMINATION
MARCH 2015
PROFESSIONAL ELECTRICAL ENGINEERING SUBJECTS

( )

30. Two single-phase transformer are each rated 75 KVA are connected in
V or open delta to serve a 3-phase load of 120 KW at 0.80 power
factor lagging. Determine the size in KVAR of the capacitor needed to
prevent overloading of the transformers.
A. 40.25 B. 41.28 C. 45.24 D. 43.50
SOLUTION:
√ √

√ √ ( )

31. A time of 10 milliseconds is required for the current on a series RL dc


circuit to reach 90% of its final steady state value. What is the time
constant in seconds for the circuit?
A. 3.39 ms B. 4.34 ms C. 3.86 ms D. 4.25 mS

32. The armature current drawn by any dc motor is proportional to


A. The voltage applied to the motor B. the torque required
C. The flux required in the motor D. The speed of the motor
33. Fault level means
A. Voltage at the point fault B. Fault current
C. Fault power factor D. Fault MVA
REGISTERED ELECTRICAL ENGINEERS PRE-BOARD EXAMINATION
MARCH 2015
PROFESSIONAL ELECTRICAL ENGINEERING SUBJECTS

34. A bulb having a rating of 220 volts, 75 watts is to be connected to a


110 volts source. What power will it draw if connected across 110 volts
source?
A. 37.5 watts B. 9.375 watts C. 75 watts D.18.75 watts
SOLUTION:
75 watts = 2202 / R R = 2202 / 75 = 645 33 Ω
P = 1102 / 645.33 = 18.75 watts

35. A capacitor is charged 0.23 watt-second of energy at a voltage of 48


volts. What is its capacitance?
A. 180 µF B. 240 µF C. 200 µF D. 220 µF
SOLUTION:
C = (0.23 (2)) / 482 = 199.65 x 10-6 F = 200 µF

36. A 20-HP, 110 v dc shunt motor has an efficiency of 88% and an


exciting current of 4 amperes and an armature resistance of 0.04 ohm.
What is the starting resistance required for full-load torque?
A. Ω B 0 96Ω C 0 92Ω D 0 46Ω
SOLUTION:
Po = 20 HP (746 watts) / 1 HP = 14920 watts
Pi = 14920 / 0.88 = 16954.55 watts

37. A coil of 50-ohm resistance and 150 mH inductance is connected in


parallel with a 50 µF capacitor. What is the power factor of the circuit?
A. 70% lagging B. 70% leading
B. C. 80% lagging D. 80% leading

SOLUTION:
A = 50 + 2 pi (60) (150 x 10-3)I = 50 + 18 pi i
B = 1 / 2 pi (60) ( 50 x 10-6) (-i) = -53.05 i
REGISTERED ELECTRICAL ENGINEERS PRE-BOARD EXAMINATION
MARCH 2015
PROFESSIONAL ELECTRICAL ENGINEERING SUBJECTS

((50 + 18 pi) (-53.05i)) / (50 + 18 pi) + (-53.05 i) = 79.895 < -45.48


P.F. = cos (45.48) = 0.707 x 100 = 70 % leading

38. A 220-volt dc shunt motor runs at 500 rpm with an armature current of
50 amperes and an armature resistance of 0 1 Ω Calculate the speed
if the torque is halved?
A. 510.8 rpm B. 508.6 rpm C. 505.8 rpm D. 502.8 rpm
SOLUTION:
N2 = (217.5 (500)) / 215 = 505.8 rpm

39. A 150-MVA, 13.8-KV, Y-connected, 3ɸ, 60-HZ alternator is operating


at rated voltage and at no-load when a three-phase faults occurs at its
terminals. If the substransient reactance of the alternator is j0.2 pu,
calculate the current the instant fault occurs.
A. 10,870 amps B. 6,276 amps
B. C. 32,611 amps D. 31,380amps
SOLUTION:
I” = 150 x 106 / (sqrt (3)) (13.5 x 103) (0.2) = 31,377.73 A

40. The capacitance reactance of a transmission line is 90,000 ohms per


kilometer. Find the total capacitance reactance if the transmission line
is 50 km long.
A. 90,000 Ω B. 4.5 x 106 Ω C 180,000 Ω D , Ω
SOLUTION:
Xc = 90000 / 50 = Ω

41. A generating station supplies peak load of 12,000 KW, 9,000 KW, and
6,000 KW. Calculate the maximum demand of the station and the
energy supplies in a year if it its diversity factor and its average annual
load factor is 1.2 and 0.7, respectively.
REGISTERED ELECTRICAL ENGINEERS PRE-BOARD EXAMINATION
MARCH 2015
PROFESSIONAL ELECTRICAL ENGINEERING SUBJECTS

A. 27,000 KW,137.97 x 106 KWHR


B. B. 22,500 KW, 137.97 x 106 KWHR
C. 27,000 KW, 5.75 x 106 KWHR
D. 22,500 KW, 5.75 x 106 KWHR
SOLUTION:
PL = (12000 + 9000 + 6000) / 1.2 =22,500,000
0.70 = Annual Energy / 22,500 (8760) = 137.97 x 106 Kw-hr

42. If the input of the prime mover of an alternator is kept constant but
excitation is increased, then
A. KVA will be leading B. KVA will be lagging
C.KW will be changed D. The p.f. of the load remains
constant
43. A load of 10 ohms was connected to a 12-volt battery. The current
drawn was 1.18 amperes. What is the internal resistance of the
battery?
A. 0 35 Ω B 7Ω C 0 25 Ω D 0 30 Ω
SOLUTION:
( )( )
( )( )

44. In electroplating, the positive electrode is called the


REGISTERED ELECTRICAL ENGINEERS PRE-BOARD EXAMINATION
MARCH 2015
PROFESSIONAL ELECTRICAL ENGINEERING SUBJECTS

A. cathode B. anode C. terminal D. iontrap

45. The per unit value of a 2-ohm resistor at 100 MVA base and 10 KV
base voltage is
A. 4 pu B. 0.5 pu C. 0.2 pu D. 2 pu
SOLUTION:

( )( )

( )( )

46. Calculate the mean spherical candle power of a lamp emitting a flux of
1,500 lumens.
A. 119.37 cd B. 1,500 cd C. 375 cd D. 477.46 cd
SOLUTION:

47. A piece of conductor 10 cm long moves across a magnetic field of


10,000 gauss at a velocity of 120 cm/sec. What voltage appears
across the conductor?
A. 0.24 v B. 60 v C. 0.12 v D. 120 v
SOLUTION:

( )( )( )( )
REGISTERED ELECTRICAL ENGINEERS PRE-BOARD EXAMINATION
MARCH 2015
PROFESSIONAL ELECTRICAL ENGINEERING SUBJECTS

48. A 220 volt, 10 HP, 60 HZ, three-phase induction motor having four
poles is operating at full-load slip of 3%. What is the frequency of the
rotor current at full-load?
A. 60 HZ B. 18 HZ C. 1.8 HZ D. 180 HZ
SOLUTION:

( )( )

49. A load of 10 MVA, 0.8 p.f. lagging, 22 KV is served by a transmission


line which has a line resistance of 3 ohms and a line reactance of 10
ohms. Solve for the sending end voltage.
A. 25,972 v B. 28,721 v C. 22,433 v D. 24,214 v
SOLUTION:

( )

( )
( )( )

50. The core loss of a 5 KVA single phase transformer with normal voltage
applied to the primary is 75 watts. The maximum efficiency occurs 60%
of full load KVA. What is the full load efficiency of the transformer at
0.80 power factor?
A. 95.16% B. 93.38% C. 89.52% D. 91.36%
SOLUTION:
REGISTERED ELECTRICAL ENGINEERS PRE-BOARD EXAMINATION
MARCH 2015
PROFESSIONAL ELECTRICAL ENGINEERING SUBJECTS

( )
( )

51. The reactance of a generator designated X” is given as 0 25 per unit


based on the generator’s nameplate rating of 18 KV, 500 MVA The
base for calculations is 20 KV, 100 MVA Find X” on the new base
A. 0.0617 pu B. 0.0405 pu C. 0.0081 pu D. 0.0450 pu
SOLUTION:
X”pu new = 0 25 18 / 20 2 (100 / 500) = 0.0405 pu

52. A constant current of 2 A flows through a circuit element. In 10


seconds, how much net charge passes through the element?
A. 0.2 C B. 2 C C. 20 C D. 200 C
SOLUTION:
REGISTERED ELECTRICAL ENGINEERS PRE-BOARD EXAMINATION
MARCH 2015
PROFESSIONAL ELECTRICAL ENGINEERING SUBJECTS

Q = 2 (10) = 20 C

53. Compute the resistance of a copper wire having a diameter of 2.05 mm


and a length of 10 m.
A. 52 Ω B 52Ω C 0 52 Ω D 2Ω
SOLUTION:
A = 2.059 (1 / 10) (1 / 2.54) = (0.0807 in x 1000)2 = 6514.10 CM
R = 10.37 (10 (3.28) / 6514.10) = 2Ω

54. What is the capacitance of a parallel-plate capacitor having rectangular


plates 10 cm by 20 cm separated by a distance of 0.1 mm? the
dielectric is air.
A. 17.7 pF B. 177 pF C. 1,770 pF D. 17,700 pF
SOLUTION:
C = (1 (0.02) (8.854 x 1012)) / 1 x 10-4 = 1770.9 pF

55. A wire length 1 m carries a current of 10 A perpendicular to a field of


flux density 0.5 T. Determine the magnitude of the force on the wire.
A. 5 N B. 50 N C. 10 N D. 100 N
SOLUTION:
F = 0.5 (10) (1) = 5 N

56. The apparent power supplied to a load in an ac circuit is 2,000 volt-


amperes with a power factor of 0.6 lagging. The reactive power is most
nearly:
A. 1,200 VAR B. 3,333 VAR C. 1,600 VAR D. 2,500
VAR
SOLUTION:
S= 2000<-cos(0.60) = 2000<-53.13 = 1200 – j 1599.998 VA
Or 1,600 VAR
REGISTERED ELECTRICAL ENGINEERS PRE-BOARD EXAMINATION
MARCH 2015
PROFESSIONAL ELECTRICAL ENGINEERING SUBJECTS

57. Barrier potential of silicon diode is:


A. 0.1 v B. 0.3 v C. 0.4 v D. 0.7 v

58. Which of the following is an active element of a circuit?


A. Ideal current source B. Resistance
C. Inductance D. Capacitive

59. A 200 µH coil has a Q of 250 at resonance frequency of 800 KHZ.


What is the effective resistance of the coil?
A. 1 Ω B 2Ω 4Ω D 8Ω
SOLUTION:
800 x 103 = 1 / 2 pi (sqrt. (200 x 10 -6 (C)) C = 1.98 x 10-10 F
250 = (1 / R) (sqrt.(200 x 10-6 /1.98 x 10-10) R=4 2Ω

60. Find the capacitive susceptance per mile of a single-phase line


operating at 60 HZ. The conductor is Partridge with an outside
diameter of 0.642 in, and spacing is 20 ff between centers.

A. 5.10 x 10-4 mho/mi to neutral B. 0.1961 x 104 mho/mi to neutral


C.5.10 x10-6 mho/mi to neutral D.0.1961x 10-4 mho/mi to neutral

61. Which wqave form will have the minimum rms value if they have the
same peak value of voltage?
A. Square wave B. Sine wave
C. Triangular Wave D. Full wave rectified sine wave
REGISTERED ELECTRICAL ENGINEERS PRE-BOARD EXAMINATION
MARCH 2015
PROFESSIONAL ELECTRICAL ENGINEERING SUBJECTS

62. The line currents in amperes in phases a, b and c respectively are 500
+ j150, 100 – j600, and -300 + j600. Determine the zero sequence
component of phase a current
A. -100 – j50 amps C. 100 – j50 amps
B. -100 + j50 amps D. 100 + j50 amps
SOLUTION:
Iao =1/3 (Ia+Ib+Ic)

Iao =1/3 { (500+j150) + (100-j600) + (-300+j600) }

Answer: 100 + j50 amps

63. The minimum armature current of a synchronous motor corresponds to


operation at
A. 0.8 p.f. lagging B. unity p.f.
B. C. 0 p.f. leading D. 0 p.f. lagging

64. A 10 KW, 110 volts dc series generator has a 2 volts brush drops, an
armature resistance Ra of 0 1 Ω and a series field resistance Rs of 0.05
Ω Calculate the generated emf when delivering rated current at rated
speed.
A. 110 v B. 130.25 v C. 115.56 v D. 125.64 v
SOLUTION:
Eg = Vt + Ia(Ra+Rs) + Vb
Eg = 110 + (10X10^3/110)(0.1 + 0.05) +2
Eg = 125.64 V

65. Lap winding is best siuited for voltage, current dc


generators.
A. High, Low B. Low, High C. High, High D. Low, Low
REGISTERED ELECTRICAL ENGINEERS PRE-BOARD EXAMINATION
MARCH 2015
PROFESSIONAL ELECTRICAL ENGINEERING SUBJECTS

66. A three-phase motor is rated 50 HP, 440 volts and 85% power factor.
What is the rated current if the motor efficiency is 89%?
A. 57.5 A B. 64.7 A C. 55 A D. 59 A
SOLUTION:
P = squareroot of 3 x V x I x pf
(50 x 746)/0.89 = squareroot of 3 (440)(I)(0.85)
I = 64.7 A

67. The phase b line voltage and the phase a line current of a balanced
three phase system are V = 220 sin ωt 210º and I = 10 sin ωt - 30º)
respectively. What is the power of the system?
A. 1,905 W B. 5,716 W C. 3,300 W D. 3,810 W
SOLUTION:
Z = 220 sin ωt 210º / 10 sin ωt 180º
Z = 22 cis 30
P = squareroot of 3 (220)(10)cos30
P = 3,300 W

68. A 200:5 current transformer is used with a 10 amps ammeter. If the


meter reads 4.5 amps, determine the line current.
A. 400 A B. 20 A C. 45 A D. 180 A
SOLUTION:
I = I ratio X I reading
I = (200/5)(4.5)
I = 180 A

69. What is the relative permittivity of vacuum?


A. Unity B. 8.854 x 10-12 C. 8.854 D. zero
REGISTERED ELECTRICAL ENGINEERS PRE-BOARD EXAMINATION
MARCH 2015
PROFESSIONAL ELECTRICAL ENGINEERING SUBJECTS

70. On triling the number of turns of a coil the inductance


A. Becomes 9 times as large B. 3 times
C.1/3 C Will remain the same

71. A 1 5 MΩ resistor is connected in series with a 20 µF capacitor. The


voltage source is 120 volts dc. Determine the current at t = 10 seconds
after switch is closed.
A. 80 µA B. 63.85 µA C. 32,64 µA D. 57.32 µA

SOLUTION:
T=RC=20micro x 1.5mega=30 sec

Q=CV=20micro(120)=2.4x10^-3 F

q=Q(1-e^(-t/RC))=2.4x10^-3(1-e^(-10/30))

q=6.803x10^4 F

q=cv; v=q/c= 6.803x10^-4 F/20micro

v=34.016; voltage resistor is equal 120-34.016

voltage resistor = 85.984; i=v/r

i=85.984/1.5 mega= 57.323 micro ampere


72. A dc ammeter has an internal resistance of 0.1 ohm. A shunt of 1.01 x
10-3 ohm is connected to the ammeter. What is the multiplier of the set
up?
A. 80 B. 50 C. 100 D. 10
SOLUTION:
Multiplier: (Rg+Rsh)/Rsh

= (0.1+0.00101)/0.00101
REGISTERED ELECTRICAL ENGINEERS PRE-BOARD EXAMINATION
MARCH 2015
PROFESSIONAL ELECTRICAL ENGINEERING SUBJECTS

=100

73. The field circuit of a 200 KW, 230 volts shunt generator is 8 A when
running full-load at rated terminal voltage. If the combined brush and
armature resistance is 0 03 Ω, solve for the electrical efficiency of the
generator.
A. 88.9% B. 85.7% C. 93.5% D. 81.7%
SOLUTION:
IL=PL/VL=200000/230=869.56 A

Ia=IL+ISH=877.56

E=VL+IaRa=230+877.56(0.03)=256.32V

Pg=EIa=256.32(877.56)=224.936 KW

n=PL/Pg x 100 = 88.91%

74. A parking lot is to be illuminated by four floodlights placed on each


corner of the lot mounted on 7 meters high poles. The parking lot
measures 25 m x 20 m. if each lamp has an output of 500 candelas,
calculate the illumination at the center of the lot.
A. 51.44 lux B. 25.72 lux C. 6.43 lux D. 12.86 lux
SOLUTION:
d2=12.52+102+72

d2=305.25

Ep=(I/d2)cos theta

Ep=(500/305.25)(7/17.471)

Ep=25.62 lux
REGISTERED ELECTRICAL ENGINEERS PRE-BOARD EXAMINATION
MARCH 2015
PROFESSIONAL ELECTRICAL ENGINEERING SUBJECTS

75. An external resistance is added in series with the field of a dc shunt


motor. When the motor runs, the effect of resistance is
A. To reduce the speed of the motor
B. To increase the speed of the motor
C. To reduce the armature current drawn by the motor
D. To reduce the losses

76. The PTs of a 220 KV transmission line have a ratio of 132.76 KV/
66.375 v and are connected wye-wye. A voltmeter connected line to
ground reads 66 volts, what is the transmission line voltage?
A. 228.65 KV B. 220.10 KV C. 223.15 KV D. 225.25 KV
SOLUTION:
VL=V x PT ratio

=114.316(132.76 KV/66.375) =228.65 KV A

77. What is the speed of an induction motor of four poles if the percent slip
is 3%?
A. 1,764 rpm B. 1,756 rpm C. 1,746 rpm D. 1,730 rpm
SOLUTION:
f=PN/120; N=120f/P=(120 x 60)/4

N=1800 rpm

Ns=1800(0.03)=54

N-Ns=756 rpm

78. A 3 phase synchronous motor runs at full load of 1,200 rpm. If the load
is reduced to one-half of rated, calculate its speed regulation.
A. 100% B. 50% C. 0% D. 86.6%
SOLUTION:
REGISTERED ELECTRICAL ENGINEERS PRE-BOARD EXAMINATION
MARCH 2015
PROFESSIONAL ELECTRICAL ENGINEERING SUBJECTS

SR%=(NR-NS)/NS

NR=1200 rpm

NS=f(120)/P=60(120)/3

NS=2400/2=1200

SR%=(1200-1200)/1200=0

79. Whicj of the following is not the method of arc dispersion?


A. Oil immersion contacts B. Magnetic blow out of
arc
C. Use of rectifiers D. Deionization of arc path

80. During load shedding


A. System voltage is reduced B. Some loads are switch off
C. System power factor is changed D. System frequency is reduced

81. The capacitor in a series RC circuit at steady state is:


A. Open circuit
B. Short circuit
C. Transient circuit
D. Coupled circuit

82. A lead storage battery is rated at 12 volts. If the internal resistance is


0 01 Ω, what is the maximum power that can be delivered to the load?
A. 1,200 watts B. 7,200 watts
B. C. 1,800 watts D.3,600watts
SOLUTION:
P = 122 / 4(0.01) = 3600 watts
REGISTERED ELECTRICAL ENGINEERS PRE-BOARD EXAMINATION
MARCH 2015
PROFESSIONAL ELECTRICAL ENGINEERING SUBJECTS

83. Two No. 8 copper conductors are placed 15 cm apart with a GMR of
1.27 x 10-3 m. if the lenth of the line is 4 km, find the value of the line
inductance.
A. 9.65 mH B. 6.85 mH C. 5.78 mH D. 7.63 mH
SOLUTION:
d= 15cm× =0.15m

-7
L= 4 ln( )

=4× 10-7 ln( )

L=1.9086×10-6

@4 km

4km×

L= 1.9086×10-6 ⁄ ×4000m

L= 7.63 mH

84. There is no zero sequence component of the fault current for


A. Line to line fault B. Single line to ground fault
C. Symmetrical fault D. Double line to ground fault

85. A diesel generator set burns fuel with a heating value of 18,000 BTU
per lb. the diesel engine has an efficiency of 30% and the alternator
has an efficiency of 95%. Determine the fuel cost component of
producing one KWHR if the diesel costs P 2.80 per lb
A. P 0.15 B. P 2.15 C. P 3.28 D. P 1.86
SOLUTION:
REGISTERED ELECTRICAL ENGINEERS PRE-BOARD EXAMINATION
MARCH 2015
PROFESSIONAL ELECTRICAL ENGINEERING SUBJECTS

Let ɳeng=efficiency of the diesel engine

ɳalt=efficiency of the alternator

Win= ( )( )

Lbs.of coal=

= 0.66548 lbs.

Fuel cost = 0.66548 lbs. × (2.80) = 1.8633

Fuel cost = 1.86

86. The ratio of lumens reaching the work plane divided by the total
lumens generated by the light is:
A. Room ratio B. Coefficient of utilization
C. Illumination factor D. Maintenance factor

87. A 50 Ω resistance is connected in series with a coil having 25 Ω


resistance and 150 mH inductance. The circuit is connected to a
voltage source of 200 sinωt Calculate the instantaneous current
A. 2 9 sinωt B 2 sin ωt-37º)
C 1 7 sin ωt-37º) D 5 11 sin ωt-37º)

SOLUTION:
Square root (2) x 200 = 163.30
Square root (3)
200-163.30=37
200 = 2.1
25+j150
2 sin ωt-37)

88. What is 1013 ergs/sec in KW?


REGISTERED ELECTRICAL ENGINEERS PRE-BOARD EXAMINATION
MARCH 2015
PROFESSIONAL ELECTRICAL ENGINEERING SUBJECTS

A. 1,000 KW B. 2,000 KW C. 1,800 KW D. 3,000 KW


SOLUTION:
1kW=1000W, 1 Watt= 1 , ergs= 10-7

1013ergs/sec×10-7 joule=1×106

=1000 kW

89. Find the rotor frequency of an induction motor having 4 poles if the
rotor speed is 1,746 rpm.
A. 60 HZ B. 1.8 HZ C. 3 HZ D. 50 HZ
SOLUTION:
( )
Ns= = =1800 rpm

S= = =0.03

Fr=sf= 0.03(60)

=1.8Hz

90. The conductors of a 30 km long transmission line have a triangular


configuration. The conductors are placed 4 ft apart. Each conductor
has 336.4 MCM ACSR and its diameter is 0.75 inch. Find the
capacitive reactance per phase?
A. 5,389 Ω/phase
B 1,354 Ω/phase
7,7 2 Ω/ph s
D. 8,261 Ω/phase
SOLUTION:
Deq= √( )( )( ) = 4
REGISTERED ELECTRICAL ENGINEERS PRE-BOARD EXAMINATION
MARCH 2015
PROFESSIONAL ELECTRICAL ENGINEERING SUBJECTS

(∑ ) (
C= = = 11.4604
( ) ( )

Xc= ( )(
=231.456×106

=231.456×106m×106 231.456×103km

= = 7715.2

=7715.2 Ω / phase

91. The secondary current of a 20 KV/7.79 KV autotransformer is 100


amperes. Determine the current that flows through the common
winding.
A. 60 amps B. 100 amps C. 79 amps D. 50 amps
SOLUTION:

therefore, for step down autotransformer:

Ic=100-38.5 Ic=60 A

92. A winding is short pitched by 60 degrees electrical. Its pitch factor is:
A. 0.5 B. 0.707 C. 0.966 D. 0.866
SOLUTION:
Sin (60) = 0.866

93. When the excitation of an unloaded salient pole synchronous motor


suddenly gets disconnected, then
A. The motor stops B. It runs at the same speed
C. It runs at lower speed D. It runs at very high speed

94. Sensitivity of a voltmeter is expressed as


A. Volts/Ohm B. Ohms/Volt
REGISTERED ELECTRICAL ENGINEERS PRE-BOARD EXAMINATION
MARCH 2015
PROFESSIONAL ELECTRICAL ENGINEERING SUBJECTS

B. C. Ohm-Volt D.1/Ohm-Volt

95. Two conductors carrying 50 amperes and 75 amperes respectively are


placed 10 cm apart. Calculate the force between them per meter.
A. 0.075 N/m B. 100 N/m
C. 0.0075 N/m D. 10 N/m
SOLUTION:

( )( )( )

F=0.0075 N/M

96. A coil has a resistance of 3 Ω and an inductance of 0 01 H is


connected in series with a capacitor across a voltage source of 220
volts. What must be the capacitance of the capacitor in order that
maximum current occurs at 60 HZ?
A. 3.77 µF B. 703.6 µF C. 0.03 µF D. 512.45 µF
SOLUTION:
60 = 1 / 2 pi (sqrt (0.01 (C))
C = 703.62 µF

97. In a T-T connected transformer, if the terminal voltage in the primary


side is 220 volts, what is the voltage across the teaser transformer?
A. 127 volts B. 190 volts C. 220 volts D. 110 volts

SOLUTION:
220(.866)= 190 V

Note that the in T-T bank primary is 50% and the teaser is 86.6%

98. At a certain point of the system network the positive, negative and zero
sequence impedances are 0.25 pu, 0.25 pu, 0.3 pu, respectively. The
REGISTERED ELECTRICAL ENGINEERS PRE-BOARD EXAMINATION
MARCH 2015
PROFESSIONAL ELECTRICAL ENGINEERING SUBJECTS

base MVA is 100. The level at that point is 34.5 KV. Determine the
zero sequence current for a one line to ground fault.
A. 6,275 A B. 7,516 A C. 8,132 A D. 2,091A
SOLUTION:
Ifpu = 3 (1 / 0.25 + 0.25 + 0.3) = 3.57 pu
If = 3.75 ( 100 x 106 / 34.5 x 103 (sqrt (3))) = 6275.55 A

99. The dc armature winding in which the coil sides are placed a pole pitch
apart is called winding.
A. Multiplex B. Full-pitch C. Fractional-pitch D. Pole-pitch

100. A power plant gets water from a dam from a height of 100 m at the
rate of 1,000 cubic meters per minute. If the overall efficiency of the
plant is 75%, what is the KW output of the plant?
A. 12,250 KW B. 5,250 KW
C. 7,550 KW D. 15,250 KW
SOLUTION:
( )( )( )
AUGUST 2015
REGISTERED ELECTRICAL ENGINEERS PRE-BOARD EXAMINATION
AUGUST 2015
PROFESSIONAL ELECTRICAL ENGINEERING SUBJECTS

1. One kwhr is equal to_____ BTU.


A. 960 B. 3,413 C. 3600 D. 1000

2. Which of the following is the basis of standards of electrical wiring installation


in the Philippines?
A. Philippine Distribution Code C. Philippine Electrical Code
B. Philippine Grid Code D. Fire Code of the Philippines

3. In dwelling units with a hallway of 3m or more in length, at least how many


receptacle outlet/s shall be installed?
A. 3 B. 2 C. 1 D. none

4. When installing two grounding electrodes, the minimum distance between


them is
A. 1800 mm B. 2000 mm C. 1500 mm D. 2400 mm

5. A lamp produces an initial illumination of 800 lux over a normally placed


screen. If the distance of the lamp from the screen is doubled, what is the
new illumination on the screen?
A. 100 lux B. 200 lux C. 300 lux D. 400 lux

6. What is the minimum required ampacity of the feeder conductors supplying a


5A, single-phase motor and a 10A lighting load?
A. 14.25A B. 15.5A C. 17.5A D. 16.25A

7. When the length of a wire is doubled and all other factors remained the same,
what would the wire resistance become?
A. Remained the same C. decreased
B. Doubled D. could not be determined
8. A 3 phase, 60Hz, 60-pole induction motor delivers 25 HP at 1,150rpm. If the
rotor copper loss is 300 watts, what is the rotor power input?
A. 7,194 w B. 6,194 w C. 8,194 w D. 5,194 w

9. An aluminum wire has a diameter of 2 mm. What must be the diameter of


copper wire of the same resistance and length as aluminum?
A. 1.33 mm B. 1.45 mm C. 1.56 mm D. 1.75mm

10. A customer owns an old air conditioning unit rated ½ HP at 70% efficiency but
decide to buy a new one of the same horse power rating with 85% efficiency
REGISTERED ELECTRICAL ENGINEERS PRE-BOARD EXAMINATION
AUGUST 2015
PROFESSIONAL ELECTRICAL ENGINEERING SUBJECTS

and with 2 years warranty. If the usage of new unit is 8 hours per day and 150
days per year, what is the differential cost of energy in one year when the unit
cost of energy is P8.99/KWHR?
A. P9,870 B. P8,070 C. P1,014 D. P2,029

11. Three impedances Z1= 2.2+j2.2 , Z2= 10+j10 and Z3=-j5 are connected in
series. The voltage across Z1 is 26. 59-j4.69 v. What is the total voltage?
A. P93+j62.4 v B. 93-j62.4 v C. 112+j51 v D. 112-j51 v

12. A series circuit consists of three resistors R1, R2 and R3 which are connected
to a 240 v source. If the voltage drop across R1 is 60 v when the current is 4
A and R2= 1.5 R3, what is the ohmic resistance R3?
A. 15 B. 18 C. 27 D. 28

13. Two exactly similar turbo-alternators are rated 5 MW each. They are running
in parallel. The speed load characteristics of the driving turbines are such that
the frequency of alternator one drops uniformly from 60 Hz on no-load to
57.5Hz on full-load, that of alternator two from 60 Hz to 58 Hz. How will two
machines share a load of 8,000 kW?
A. 3,556 kW, 4,444 kW C. 3,500 kW, 4500 kW
B. 4,000 kW, 4,000 kW D. 3,636 kW, 4,364 kW

14. Two coupled coils have self-inductances L1= 2 H and L2= 0.5 H, and a
coefficient of coupling k= 0.9. Determine the turns N1/N2 of the two coils.
A. 2 B. 0.2 C. 0.5 D. 0.9
15. A 220-v bulb draws a current of 10 A. How much change in kilocoulombs will
pass through the bulb in two hours period?
A. 7.2 B. 3.6 C. 36 D. 72

16. A 36 resistor is connected in parallel with an unknown resistor R. Their


combination is then connected in series with a 12 resistor. Find the value of
R such that power drawn by the parallel combination of 36 and R is equal to
the power in the 12 resistor.
A. 48 B. 24 C. 18 D. 6

17. A 3 phase, 60 Hz transposed transmission line is arranged horizontally with


distances D12= 5 ft., D23= 5 ft. and D13= 10 ft. The outside diameter of the
conductors is 0.721 inch and with GMR of 0.0243 ft. determines the
capacitance per phase of the line in microfarad per kilometer.
REGISTERED ELECTRICAL ENGINEERS PRE-BOARD EXAMINATION
AUGUST 2015
PROFESSIONAL ELECTRICAL ENGINEERING SUBJECTS

A. 0.0404 B. 0.0304 C. 0.0204 D. 0.0104

18. An act to ensure and accelerate the total electrification of the Philippines, also
known as the EPIRA law of 2001.
A. RA 7920 B. RA 8189 C. RA 9136 D. RA 1081

19. Which of the following is not included in the evaluation of the Technical
Performance of a Distribution System according to Philippine Distribution
Code (PDC)?
A. Power quality C. Collecting Efficiency
B. Reliability D. System Efficiency

20. A 13.8 kV/480 v, 30-MVA three- phase transformer has a 0.03 pu resistance
and 0.004 pu reactance. What is the transformer actual impedance referred to
the primary?
A. 0.3174 B. 0.6348 C. 0.1587 D. 0.2749

21. Three capacitance of 4µF, 6 µF, and 5 µF are connected in series across a
110-volt dc source. Find the charge passing through each capacitor.
A. 1,650 µC B. 69 µC C. 178 µC D. 7.33 µC

22. 7.1x10^10 BTU is equal to______ Gcal.


A. 17.829 B. 178.92 C. 1,789.2 D.17,892
23. Calculate the force that will be exerted on the scale in pony-brake test when a
20-HP, 1,400 rpm motor is operating at full-load. The length of the brake arm
is 3-ft, and the tare weight of the brake is 3.75lb.
A. 26.86 lb B. 62.76 lb C. 36.70 lb D. 28.76 lb

24. The core of a certain magnetic circuit has a length of 5 cm and the inductance
of the coil is 10 mH. What is the inductance of the same if the number of turns
is reduced to 1/2 of the original? All other factors are unchanged.
A. 20 mH B. 10 mH C. 2.5 mH D. 5.0 mH

25. A piece of conductor 20 cm long moves across a magnetic field of 10,000


gauss at a velocity of 120 cm/sec. What voltage appears across the
conductor?
A. 0.24 v B. 0.12 v C. 120 v D. 60 v
REGISTERED ELECTRICAL ENGINEERS PRE-BOARD EXAMINATION
AUGUST 2015
PROFESSIONAL ELECTRICAL ENGINEERING SUBJECTS

26. A certain electric generating station has an efficiency of 25%. If coal


consumption is 4,500 kg/hr and calorific value of coal is 6,000kcal/kg. Find
the total energy produced per day.
A. 753.6 MWHR B. 188.4 MWHR C. 376.8 MWHR D.565.2 MHWR

27. Three 40 non-inductive resistances are connected in wye across 200 v, 3-


phase lines. If one of the resistors is disconnected, what would be the power
taken from the mains?
A. 3,000 w B. 2,000 w C. 1,000 w D. 500 w

28. A 220 v, 3-phase supply has a balanced delta connected load with
impedance of 15+j20 /phase. What is the total real power of the load in
watts?
A. 5,133.3 B. 2,965.6 C. 4,122.5 D. 3,484.8

29. Sparking occurs when a load is switched off because the circuit has high
A. Capacitance B. inductance C. resistance D. impedance

30. A 120-volt battery having an internal resistance of 0.5 is connected through


line resistance of 9.5 to a variable load resistor. What maximum power will
the battery deliver to load resistor?
A. 36 watts B. 63 watts C. 630 watts D. 360 watts

31. A sis pole induction motor is connected to a 60 Hz supply and full-load the
rotor emf makes 105 complete cycles in 2 minutes. Find the rotor speed.
A. 1,150 rpm B. 1,182.5 rpm C. 1,140.5 rpm D. 1,170 rpm

32. A. 20 kVA, 2000/220 v, single –phase transformer has a primary resistance of


2.1 and a secondary resistance of 0.026 . The corresponding leakage
reactances are 2.5 and 0.03 . Determine the transformer per unit
impedance.
A. 0.065 pu B. 0.043 pu C. 0.033 pu D. 0.055 pu

33. Slip ring are made of


A. Steel B. copper C. bakelite D. mica
REGISTERED ELECTRICAL ENGINEERS PRE-BOARD EXAMINATION
AUGUST 2015
PROFESSIONAL ELECTRICAL ENGINEERING SUBJECTS

34. The resistance of a coil of wire increases from 40 at 10 degrees Celsius to


48.25 at 60 degrees Celsius. Find the temperature coefficient at 0 degrees
Celsius of the conductor material.
A. 0.0043 C-1 B. 0.0039 C-1 C. 0.0027 C-1 D. 0.0013 C-1

35. The primary reason for using carbon brushes over copper brushes in DC
motors is
A. They have low loss C. They produce less arcing
B. They are stronger D. all of these

36. A capacitor consists of two metal plates each plates 40 cm x 40 cm, spaced
6 mm apart. The space between the metal plates is filled with a glass plate
5mm thick and layer of paper 1mm thick. The relative permittivities of glass
and paper are 8 and 2, respectively. Calculate the capacitance.
A. 118 X 10-11 F B. 122X10-11 F C. 126X10-11 F D.130X10-11 F

37. A single-phase transformer is rated 110/440 v, 2.5 kVA. Leakage reactance


measured from the low- tension, side is 0.06 . Determine leakage reactance
on the high-tension side in per unit.
A. 0.0124 B. 0.1984 C. 0.0496 D.0.3963

38. The capacitance per kilometer of a 3-phase belted cable is 0.3µF between the
two cores with the third core connected to the lead sheath. Calculate the
charging current taken by the five kilometers of this cable when connected to
a 3-phase 50 Hz, 11kV supply.
A. 5.98A B. 10.37A C. 3A D. 4.18A

39. Power developed by dc motor is maximum when the ration of back emf and
applied voltage is
A. Double B. Zero C. Unity D. Half

40. An office 15m by 8m wide is lighted by 8 lamps and each lamp has luminous
intensity of 200 sd, allowing an absorption loss of 4,000 lm for the reflectors,
walls, and ceiling, calculate the average illumination on the working plane.
A. 167.6 lux B. 134.2 lux C. 20.9 lux D. 106.7 lux

41. Two coils having inductances of 20 mH and 50 mH are wound on top of each
other. Calculate the mutual inductive between the two coils if the coupling
coefficient k= 0.98
REGISTERED ELECTRICAL ENGINEERS PRE-BOARD EXAMINATION
AUGUST 2015
PROFESSIONAL ELECTRICAL ENGINEERING SUBJECTS

A. 29 mH B. 30 mH C. 31 mH D. 32 mH

42. A lead storage battery is rated at volts. If the internal resistance is 0.01 ,
what is the maximum power that can be delivered to the load?
A. 1,200 watts B. 7,200 watts C. 3,600 watts D. 1,800 watts

43. Two charges of 2 x 10-5 C and 3 x 10-6 C are placed 10cm apart. Find the
force in kg on each charge.
A. 5.5kg B. 7.7kg C. 10kg D. 2.25kg

44. Three units of 1:10 Transformers are connected in ∆/Y to supply a 3-phase
load from a 400-v, 3-phase source. What is the line voltage on the load side?
A. 6,928 v B. 3,464 v C. √3(40)v D. 4,000 v

45. The starting winding of single-phase motor is placed in


A. Rotor B. stator C. armature D. field

46. A 50-MVA. 11-kV, 3-phase alternator supplies full-load at a lagging power


factor of 0.7. What would be the percentage increase in earning capacity if
the power factor is increase in earning capacity if the power factor is
increased to 0.95?
A. 26.32 B. 6.92 C. 12.76 D. 35.71

47. In a synchronous motor, the rotor copper losses are met by


A. Motor input
B. armature input
C. supply lines
D. DC source

48. A string insulator has 5 units. The voltage across the bottom most unit is 25%
of the total voltage. What is the string efficiency?
A. 80% B. 75% C. 67% D. 86%

49. A delta connected load of 9 +j12 /phase is supplied by a 230 v, 3-phase


source through a line with an impedance of 0.06+j0.12 /conductor. Find the
load voltage in volts.
A. 220 B. 222 C. 224 D. 226
REGISTERED ELECTRICAL ENGINEERS PRE-BOARD EXAMINATION
AUGUST 2015
PROFESSIONAL ELECTRICAL ENGINEERING SUBJECTS

50. A substance that contains atoms with several bands of electrons but with only
one valence electron is
A. Insulator B. Conductor C. Semiconductor D. Resistor

51. What is the resistance of 2,000 m of copper wire that has a resistivity of 1.72X
10-8 -m and a diameter of 4-mm?
A. 0.086 B. 2.74 C. 1.37 D. 3,24
52. The charge on the plates of a 1µF capacitor is 50 µC. What is the energy
stored by the capacitor?
A. 1.25 mJ B. 25mJ C. 5.5mJ D. 10.5Mj

53. What is the ac current in a circuit if the impedance is 5 j4 Ω and the voltage
source, Vrms is 10v?
A. 2.04 38.7 A B. 1.56 -38.7 A C. 1.25 54.3 A D. 1.11 -42.6 A

54. If the capacitance in a series RLC ac circuit increase by a factor of 4, what


happens to the resonant frequency?
A. It doubles C. It decrease by 50%
B. It stays the same D. It increases by 50%

55. A 10:1 ideal transformer has a load of 0.1µF on the secondary. What is the
load capacitance, as referred to the primary?
A. 1µF B. 10µF C. 0.01µF D. 0.001µF

56. A 110-volt shunt motor has an armature resistance of 0 8Ω and a field


resistance of 220Ω At full load the motor takes 10 amperes, and the speed is
1,200 rpm. At what speed must this motor run as a generator to deliver 10
amperes to an external circuit at 110v?
A. 1,237.8 rpm B. 1,337.5 rpm C. 1,037.8 rpm D. 1,387.5 rpm

57. When a conductor is stationary and the magnetic field is moving or changing,
the emf is called:
A. Self-induced emf C. Mutually induced emf
B. Statically induced emf D. Dynamically induced emf

58. The per unit impedance of a circuit element is 0.30. If the base kV and base
MVA are halved, then the new value of the per unit impedance of the circuit
element will be
A. 0.30 B. 0.003 C. 0.60 D. 0.006
REGISTERED ELECTRICAL ENGINEERS PRE-BOARD EXAMINATION
AUGUST 2015
PROFESSIONAL ELECTRICAL ENGINEERING SUBJECTS

59. In a series RL circuit, R=20Ω and L= 0 06H The current lags the voltage by
80. Determine .
A. 1,890 rad/sec B. 328 rad/sec C. 1,980 rad/sec D. 823 rad/sec
60. A 12-pole, 3- phase alternator driven at a speed of 500 rpm supplies power to
an 8-pole, 3-phase induction motor. If the slip of the motor at full-load is 3%,
calculate the full-load speed of the motor.
A. 750 rpm B. 734.5 rpm C. 727.5 rpm D. 742 rpm

61. A coulomb
A. Represent a current of 1 ampere
B. Flows through a 100-watt light bulb
C. Is equivalent to 1 ampere per second
D. is an extremely large number of charge carriers

62. A balanced wye-connected load with resistance of 8 Ω/phase and no


reactance is supplied by a 240v, 3-phase source. Find the real power of the
load in watts.
A. 6,400 B. 7,200 C. 6,800 D. 7,600

63. Given three unbalanced line currents Ia= 0+j12 A, Ib=5-j10A and Ic=-10-j5A.
Find the negative sequence component of Ic in amperes.
A. 3.12 -14.3 B. 3.12 14.3 C. 2.25 -14.3 D. 2.25 14.3

64. A hollow sphere is charged to 12 µC of electricity. Find the potential inside the
sphere. The radius of the sphere is 0.1m.
A. 1.08x107v B. 1.08x106v C. 1.08x104v D. 1.08x 102v

65. An electromagnet has an air gap of 3 mm and the flux density in the gap is
1.257 Wb/m2. Calculate the ampere-turns required for the gap.
A. 2000 B. 2,500 C. 3,000 D. 4,000

66. An alternating current is represented by i=70.7sin 520t. Determine the value


of the current 0.0015 second after passing through zero and increasing
positively.
A. 49.7 A B. 0.96 A C. 24.8 A D. 74.9 A
REGISTERED ELECTRICAL ENGINEERS PRE-BOARD EXAMINATION
AUGUST 2015
PROFESSIONAL ELECTRICAL ENGINEERING SUBJECTS

67. A 50-kVA single-phase transformer has on full-load a copper loss of 560


watts and iron loss of 500 watts. Calculate the efficiency at 25% of full-load
for a power factor of 0.8 lagging.
A. 92.94% B. 94.92% C. 93.94% D. 95.92%
68. Fault level means
A. Voltage ate the point of fault C. Fault power factor
B. Fault current D. Fault MV

69. Which plant can never have 100% load factor?


A. Hydro B. Peak C. Nuclear D. Base load plant

70. A lamp produces an illumination of 10 lux at point 10 m directly below it. What
is luminous intensity of the lamp?
A. 100 B. 1000 C. 10 D. 1

71. Find the capacitive susceptance per mile to neutral of a single-phase line
operating at 60 Hz. The conductor is Partidge with an outside diameter of
0.642 inch, and spacing is 20ft between centers.
A. 2.55x10^-6 mho/mi to neutral C. 5.10x10^-6 mho/mi to neutral
B. 2.55x10^-4 mho/mi to neutral D. 5.10x10^-4 mho/mi to neutral

72. A 100 mw power station delivers 100 MW for 2hrs, 50 MW for 8hrs and is
shut down for the rest of each day. It is also shut down for maintenance for 60
days each year. Calculate its annual load factor.
A. 18% B. 19% C. 20% D. 21%

73. When a line to ground fault occurs, the current in a faulted phase is 300 A.
What is the zero sequence current in this case?
A. Zero B. 100A C. 200A D. 300A

74. Two 100-watt lamps connected in series across a 220-v line draw 1 A. What
is the total power consumed?
A. 200w B.220w C. 100w D. 50w

75. Three capacitors connected in series across a 135-v supply. The voltage
across them are 30,45 and 60v and charge on each is 4,500 µC. Find the
total capacitance in microfarads.
A. 33.3 B. 100 C. 75 D. 150
REGISTERED ELECTRICAL ENGINEERS PRE-BOARD EXAMINATION
AUGUST 2015
PROFESSIONAL ELECTRICAL ENGINEERING SUBJECTS

76. A potential lethal electric current is on the order of


A. 0.01 mA B. 0.1mA C. 1mA D. 0.1A

77. The unit elastance is


A. Farad B. daraf C. siemen D. henry

78. A complex current wave is represented by the equation i= 5+ 5sin(120pi)t A.


Find its average value in amperes.
A. 4.5 B. 6 C. 5 D. 4

79. The core loss of a 5-kVA single-phase transformer with normal voltage
applied to the primary is 75 watts. The maximum efficiency occurs 60% of full-
load kVA. What is the full-load efficiency of the transformer at 0.80 power
factor?
A. 95.16% B. 93.38% C. 89.52% D. 91.36%

80. A 34.5 kV feeder line is 5 miles long. The conductors are spaced 4-feet
horizontally. The conductor is 4/0 copper with GMR of 0.01688 ft. What is the
reactance?
A. 2.36 Ω B. 6.72 Ω C. 3.46 Ω D. 5.16 Ω

81. A power plant gets water from a dam from a height of 122.45 meters at the
rate of 1,000 cubic meters per minute. If the output of the plant is 15,000kW,
what is the plant efficiency?
A. 65% B. 70% C. 75% D. 80%

82. If resonant frequency is 10kHz and quality factor is 50, then


A. Bandwidth is 200 Hz C. R= 50 Ω
B. XL=50,000 Ω D. Xc= 50,000 Ω

83. All magnetic fields originates from


A. Moving electric charges C. magnetic domain
B. Iron atoms D. permanent magnets

84. A capacitor is charged by a dc source through a resistor of one megohm. If in


one second, the potential difference across capacitor reaches 80% of its final
value, calculate its capacitance.
REGISTERED ELECTRICAL ENGINEERS PRE-BOARD EXAMINATION
AUGUST 2015
PROFESSIONAL ELECTRICAL ENGINEERING SUBJECTS

A. 0.321µF B. 0.421µF C. 0.512µF D. 0.621µF

85. The input power to a 3-phase induction motor running without load is
measured by two wattmeter method. One wattmeter indicates-220 watts and
the other one indicates 940 watts. What is the no –load power factor of the
motor?
A. 0.483 B. 0.253 C. 0.425 D. 0.337

86. The power required to produce a gain of 10dB over 4 w of original power is
A. 80w B. 40w C. 0.4w D. 8w

87. In a 12-Pole duplex lap wound dc armature, each conductor can carry a
current of 50 amperes. If the total armature conductor is 600, what is the
rated current of the armature?
A. 30,000A B. 1,200A C. 600A D. 2,500A

88. A 5-MVA, 13.8 kV/480 V, 5% impedance transformer is tapped at 13.8 kV line


where the Thevenin’s equivalent impedance is 0 5Ω Determine the fault
current at the primary for a three-phase fault at the secondary.
A. 10,500A B. 4,200A C. 6,050 D. 3,300A

89. If stator voltage of a SCIM is reduced to80% of its rated value, torque
developed is reduced by____ percent of its full-load value.
A. 64 B. 36 C. 20 D. 40

90. An ion
A. Is electrically neutral
B. Has positive electric charge
C. has negative electric charge
D. can have either a positive or negative charge

91. Two long straight conductors each carrying an electric current of 5A are kept
parallel to each other at a separation of 2.5cm. Calculate the magnitude of
magnetic force experienced by 10cm of a conductor.
A. 5X10^-5N B. 4x 10^-5N C. 3x 10^-5N D. 2x 10^-N
92. The current in the coil of a large electromagnet falls from 6A to 2A in 10ms.
The induced emf across the coil is 100v. What is the self- inductance of the
coil?
A. 10H B. 5H C. 0.50H D. 0.25H
REGISTERED ELECTRICAL ENGINEERS PRE-BOARD EXAMINATION
AUGUST 2015
PROFESSIONAL ELECTRICAL ENGINEERING SUBJECTS

93. An induction motor is analogous to


A. Two winding transformer with open circuit secondary
B. Two winding transformer with short circuit secondary
C. 3 winding transformer
D. Autotransformer

94. A three-phase,4-pole,50 Hz induction motor supplies a useful torque of 160


N-m at 4% slip. If friction and windage losses are 500 watts, determine the
rotor input.
A. 25,653w B. 26,653w C. 24,615w D. 23,615w

95. In a balanced 3-phase voltage source with phase sequence abc, phase A
voltage is 127cis 0 volts. Find the line voltage Vbc
A. 220 cis(-150)v B. 220 cis(-120)v C. 220 cis(-60)v D. 127 cis(-120)v

96. If the line frequency is 60 Hz, the output frequency of a bridge rectifier is
A. 30 Hz B. 60 Hz C. 120 Hz D. 240 Hz

97. A diesel generator set burns fuel with a heating value of 18,000 BTU per lb.
The diesel engine has an efficiency of 30% and the alternator has an
efficiency of 95%. Determine the fuel cost component of producing one
KWHR if the diesel cost P12.80 per lb.
A. P8.51 B. P7.51 C. P6.51 D. P5.51

98. The capacitance reactance of a transmission line is 90,000 ohms per


kilometer. Find the total capacitive reactance of a transmission line is 60-km
long.
A. 5.4x10^6 ohms B. 1500 ohms C. 90,000 ohms D. 180,000 ohms

99. Estimate the distance over which a load of 150 MW at a power factor of 0.8
lagging can be delivered by a three phase transmission line having a
conductors each of the resistance 1 ohm per kilometer. The voltage at the
receiving end is to be 132 kV and the loss in the transmission is to be 5% of
the power delivered.
A. 37.18 km B. 55.77 km C. 42.93 km D. 74.36 km
100. A power transformer rated 50MVA, 34.5kV/13.8 kV is connected wye-wye.
What is the rated primary line current?
A. 636.7 A B. 1,449.3A C. 2,091.8 D. 1,207.7A

SOLUTION:
REGISTERED ELECTRICAL ENGINEERS PRE-BOARD EXAMINATION
AUGUST 2015
PROFESSIONAL ELECTRICAL ENGINEERING SUBJECTS

1. 1Kwhr = 3413 BTU


2. C. Philippine Electrical Code
3. C. Outlet
4. A. 1800 mm
5. B. 200 Lux
Solution :

( *

( *

6. D. 16.25
Solution:
( )( )

7. B. doubled
Solution:

8. A. 7, 194 W
Solution:
( )( )

9. C. 156 mm
Solution:

( ) ( )

10.
REGISTERED ELECTRICAL ENGINEERS PRE-BOARD EXAMINATION
AUGUST 2015
PROFESSIONAL ELECTRICAL ENGINEERING SUBJECTS

11. D. 112-j51 V
Solution :

( )

(( ) ( ) ( ))

12. B. 18
Solution :

13. A. 3556 kW, 4.444kW


Solution :
@58Hz

@57.5Hz

14. A. 2
Solution ;
REGISTERED ELECTRICAL ENGINEERS PRE-BOARD EXAMINATION
AUGUST 2015
PROFESSIONAL ELECTRICAL ENGINEERING SUBJECTS

15. D. 72
Solution:

( )( )( )
𝐶
16. C 18Ω
Solution :

17.
18. C. RA 9136
19. C. Collecting Efficiency
20. A 0 3174Ω
21. C. 178µC
Solution:
𝐶
𝐶 (( ) ( ) ( ) )
𝐶
( )( )
𝐶
22. D. 17.89
Solution:

23. D. 28.76
Solution:

( )( )

( )( )
REGISTERED ELECTRICAL ENGINEERS PRE-BOARD EXAMINATION
AUGUST 2015
PROFESSIONAL ELECTRICAL ENGINEERING SUBJECTS

24. C. 2.5 mH
Solution ;

( *

( *
25. B. 0.12 v
Solution :

( )( )( )( )
26.
27. D. 500 w
Solution ;

28. D. 3.48
Solution :
220/15+j20= 5.28+j 7.04

( )( )

29. B. Inductance
30. D. 360 watts

( )

31. B. 1 182.5
Solution :
( )

( )( )

32.
REGISTERED ELECTRICAL ENGINEERS PRE-BOARD EXAMINATION
AUGUST 2015
PROFESSIONAL ELECTRICAL ENGINEERING SUBJECTS

33. B. Copper
34. A. 0.0043⁰C
𝐶

35. C. They Producing less arching


36.
37. A. 0.0124
Solution:

( )( *

38. A. 5.98 A
( )( ) ( )

( )( )( )

39. D. half
40.
41. C. 31mH
Solution:
√ √( )( )

42. C. 3600 watts


Solution:

( )
43. A. 5.5 kg
Solution :
( )( )
( )
REGISTERED ELECTRICAL ENGINEERS PRE-BOARD EXAMINATION
AUGUST 2015
PROFESSIONAL ELECTRICAL ENGINEERING SUBJECTS

44. A. 400:4000
Solution:
√ ( )
45. B. Stator
46. D.35.71
Solution :

47. D. DC Source
48. A. 80 %
Solution:

49.
50. B. Conductor
51. B 2 74Ω
Solution :
( )( )
( )

52. A. 1.25 mJ
Solution :

𝐶 ( )( )

W= 1.25 mJ
53. B. 1.56 -38.7 A
Solution :

54. C. It decreases by 50 %
55. D. 0.001 F
Solution :
REGISTERED ELECTRICAL ENGINEERS PRE-BOARD EXAMINATION
AUGUST 2015
PROFESSIONAL ELECTRICAL ENGINEERING SUBJECTS

𝐶 ( )( *

𝐶 ( )( ) 001µF
56. D. 1387.5 rpm
Solution :

( )( )

( ) ( ) )

( )( )

57. B. Statically induced emf


58.
59. A. 1890 rad/sec
Solution :

( )( )

60. C. 727.5
Solution :
( )( )

@motor
( )( )

( ) ( )( )

61. D. is an extremely large number of charge carriers


62. B. 7200
REGISTERED ELECTRICAL ENGINEERS PRE-BOARD EXAMINATION
AUGUST 2015
PROFESSIONAL ELECTRICAL ENGINEERING SUBJECTS

( √ , √

( √ )( *

63. C. 2.25 -14.3
Solution :
( )

(( ) ( )( ) ( )( )

64. B. 1.08 x10^6 V


Solution :

( )

65. C. 3000
( )( )
= 3,000
66.
67. B. 94.92
Solution :
( )( )

( )
( ) ( )

68. D. Fault MVA


69. D. Base load Plant
70. B. 1000
Solution :
( )( )
71.
72. D. 215
Solution :
REGISTERED ELECTRICAL ENGINEERS PRE-BOARD EXAMINATION
AUGUST 2015
PROFESSIONAL ELECTRICAL ENGINEERING SUBJECTS

( )( )( ) ( )( )( )

73. B. 100 A
Solution :

74. B. 220
75.
76.
77.
78. B. 5
Solution :

√( * ( )

79.
Solution :

( )( )( )
( )( ) ( )

80. C 3 46 Ω
Solution :
√( )( )( )
)
(
( )
( )( )( )( )

81. C. 75 %
Solution :
( )( )
( )( )( )
REGISTERED ELECTRICAL ENGINEERS PRE-BOARD EXAMINATION
AUGUST 2015
PROFESSIONAL ELECTRICAL ENGINEERING SUBJECTS

82. A. Bandwith is 200 Hz


Solution :

83. A. Moving electric charge


84.
85. D. 0337
Solution :

√ ( )
( *
70.28

( ) )
86.
87. B. 1200 A
Solution :
( ) ( )( )( )

88. D. 3300A
Soltuion :
( )( )
( )

(√ )( )( )

89. B. 36
Solution :

( * ( )( *

90. D. can have either a positive or negative charge


91.
Solution :
( )( )

( )( )( )
REGISTERED ELECTRICAL ENGINEERS PRE-BOARD EXAMINATION
AUGUST 2015
PROFESSIONAL ELECTRICAL ENGINEERING SUBJECTS

5
92. D. 0.25 H
Solution :

93. B. two winding transformer with the short circuit secondary


94. A. 25, 653
Solution :
( )( )
( )
( )( )( )

𝐶 ( ) ( )( 𝐶 )
𝐶
( )

95. D. 127 cis (-120)


Explanation : since it positive sequence the rotation of Vbc is -120
96.
97.
98. B. 1500 ohms
Solution:

99. A. 37.18 km
Solution :

( )(√ )( )

( )( )

100. A. 836.7 A
REGISTERED ELECTRICAL ENGINEERS PRE-BOARD EXAMINATION
AUGUST 2015
PROFESSIONAL ELECTRICAL ENGINEERING SUBJECTS

√ √
MARCH 2016
REGISTERED ELECTRICAL ENGINEERS PRE-BOARD EXAMINATION
MARCH 2016
PROFESSIONAL ELECTRICAL ENGINEERING SUBJECTS

1. The color yellow reserved for se in equipment grounding conductor is


A. Yellow B. Green C. White D. Gray

2. What is the emf induced across an inductor with 150mH inductance and
current of 4 amperes?
A. 0.6v B. 0v C. 1.2v D. 12v

3. Which of the following is not a full-duplex?


A. Radar B. Telephone
C. Tolemetry D. Local Area Network

4. In a circuit breaker, the time duration from the instant of the fault extinction of
arc is known as
A. Operating time B. total clearing time C. Lag time D. lead time

5. A resistance of 6 ohms is connected in parallel with another resistor R across


a 120v supply. The total current drawn from a supply was found to be 40A. What
is the value of resistance R.
A. 1 ohms B. 2 ohms C. 4 ohms D. 6 ohms

SOLUTION:

Z= V/ I

Z= 120V/ 40A = 30 ohms

Current in R1= V/ R

= 120/ 6= 20A

R2= V/ I
REGISTERED ELECTRICAL ENGINEERS PRE-BOARD EXAMINATION
MARCH 2016
PROFESSIONAL ELECTRICAL ENGINEERING SUBJECTS

= 120V/ 20 = 6 ohms

Answer: D. 6 ohms

6. A 50 cm long is moved in a field of density 1Wb/m squared at a velocity of


30m/s. what is the emf induced if the motion is perpendicular to the field?
A. 150v B. 1.5v C.15v D. 0.15v

SOLUTION:

e= BLvsin Ф

= (1)(50x 10^-2)(30)(sin 90)

= 15v

Answer: C. 15v

7. Two capacitors C1= 50uF and C2= 30uF are connected in series. Find the
equivalent capacitance in uF.
A .1,875 B. 187.5 C. 18.75 D. 1.875

SOLUTION:

C total = ((C1-1) + (C2-1))-1)

= ((50uF-1) + (30uF-1))-1)

= 18.75 uF

Answer: C. 18.75 uF
REGISTERED ELECTRICAL ENGINEERS PRE-BOARD EXAMINATION
MARCH 2016
PROFESSIONAL ELECTRICAL ENGINEERING SUBJECTS

8. 34.5 KV to 69 KV is classified as
A .Low voltage B. Medium voltage C. High voltage D. Extra high vol.

9. Wavelength is the distance travelled by an electronic wave during the time of


one cycle. Given a wavelength of 12 meters, what is the frequency?
A. 250 KHZ B. 25KHz C. 250MHZ D. 25MHZ

SOLUTION:

F = VC/ wavelength

= (3x108) / 12

= 25MHZ

Answer: D. 25MHZ

10. A 4 pole dc generator has total of 600 conductors in its lap wound armature.
The flux per pole is 108 lines. Determine the generated emf if it is driven at 2400
rpm.
A. 230v B. 254v C. 234v D. 240v

SOLUTION:
Eg= PZON/ 60a
= ((4)(600)(105)(2400)/ 60(4))* 10-8

= 240V

Answer: D. 240V
REGISTERED ELECTRICAL ENGINEERS PRE-BOARD EXAMINATION
MARCH 2016
PROFESSIONAL ELECTRICAL ENGINEERING SUBJECTS

11. For an RL circuit, the power factor cannot be less than ______ or greater
than _____.
A .0,1 B. 1,0 C. 0, -1 D. -1, 0

12. In a certain country, the peak power demand for year 2005 is 9027 MW and
by year 2014 it is estimated to be 10064 MW. Calculate the growth rate.
A. 0.9 % B. 0.8% C. 0.76% D. 0.85%

SOLUTION:
F= Pern
10,064= 9287er(9)
r= 0.89 %
Answer: A. 0.9%

13. __________ is the overall energy program formulated and updated yearly by
the DOE, submitted to the congress pursuant to RA 7638.
A. Philippine Energization Program C. Philippine Electrification Program
B. Philippine Energy Plant D. Philippine Electrification Plan

14. A certain amount of fuel can be converted into 3x10 quads of energy.
Assume 30% overall efficiency for the power plant. If the average load on the
power plant over a 24 hr period is 60MW, how long in days the fuel will last?
A. 180 B. 175 C. 183 D. 185

15. The lighting load for a dwelling expressed in terms of a unit load in volt
ampere per square meter must be at least _______.
A. 24 B. 12 C. 16 D. 2

16. The capacitors of power factor correction are rated in terms of


REGISTERED ELECTRICAL ENGINEERS PRE-BOARD EXAMINATION
MARCH 2016
PROFESSIONAL ELECTRICAL ENGINEERING SUBJECTS

A. KV B. KW C. KVA D. KVAR

17. Find the maximum power drawn from a 12v battery its internal resistance is
0.25.
A. 72W B. 144W C. 576W D. 36W

SOLUTION:

P= V2/ 4R

= (12v)2/ 4(0.25)

= 144W

Answer: B. 144W

18. Two identical coupled coils have an equivalent inductance of 80mH when
connected in series aiding and 35mH in series opposing. What is the value of the
coefficient of coupling K.
A. 0.391 B. 0.319 C. 0.193 D. 0.139

SOLUTION:

M = (Lta – Lto)/ 4

= (80 – 35)/ 4

= 11.25 mH

Lta= 2L+ 2M

80= 2L+ 2(11.25)


REGISTERED ELECTRICAL ENGINEERS PRE-BOARD EXAMINATION
MARCH 2016
PROFESSIONAL ELECTRICAL ENGINEERING SUBJECTS

L= 28.75mH

K= M/ sq. rt of (L1L2)

= 11.25/ sq. rt of (28.752)

= 0.391

Answer: A. 0.391

19. What is the emf induced in a coil of 500 turns and with a constant flux of 10 8
maxwells?
A. 0v B. 5v C. 50v D. 500v

20. In a balanced three phase wye connected circuit, the ine voltage are equal
A. To the line current C. and so are the line currents
B. To the phase voltage D. but the line currents are unequal

21. If a given circuit when the power factor is unity the reactive power is
A. Maximum B. equal to i2r C. zero D. minimum

22. What is the branch ampacity of a hermectic motor with a full load current of
16A?
A. 15A B. 20A C. 25A D. 30A

SOLUTION:

Ampacity = Current full load x 80%

= 16x 80%

= 20A
REGISTERED ELECTRICAL ENGINEERS PRE-BOARD EXAMINATION
MARCH 2016
PROFESSIONAL ELECTRICAL ENGINEERING SUBJECTS

Answer: B. 20A

23. A three phase wye delta connected, 50MVA, 345/ 34.5 Kv transformer is
protected by differential protection. The current transformer on the high side for
differential protection is 150:5. What is the current on the secondary side of
CT’s?
A. 3.83A B. 2.53A C. 4.50A D. 4.83A

SOLUTION:

a= Ip= = 144.93 A

a=

=
( )( )
Is=

Is=4.831 A

Answer: D. 4.831 A

24. Joule found out that the heat produced in a current carrying conductor is
proportional to
A. The square of the current C. the square of resistance
B. The current D. inversely proportional to time
REGISTERED ELECTRICAL ENGINEERS PRE-BOARD EXAMINATION
MARCH 2016
PROFESSIONAL ELECTRICAL ENGINEERING SUBJECTS

25. A conductor has four identical strands arranged in diamond configuration.


Find the GMR of the conductor in terms of the radius r of each strand.
A. 1.6921r B. 1.7037r C. 1.3008r D. 1.3052r

26. The resistance of the same length f aluminum and copper wires are found to
be equal. Find the ratio of radii of aluminum to copper.
A. 0.78 B. 1.29 C. 0.88 D. 1.14

27. In a wye connected system, line current is


A. 0.707 times the phase current C. equal to the phase current
B. 1.735 times the phase current D. 1.414 the phase current

28. What is the device function number for overcurrent relay?


A. 27 B. 67 C. 50 D. 51

29. In a series RL circuit, the inductor current ________ the resistor current.
A. Tags B. leads C. is equal to D. is greater than

30. A three phase, 60Hz, transmission line has its conductors arranged in a
triangular configuration so that the two distances between conductors are 5m
and the third is 8m. the conductors have an outside diameter of 250 mils. Find
the inductive reactance in ohm per km per phase of the transmission line.
A. 0.567 B. 0.283 C. 0.586 D. 0.293
SOLUTION:
GMD = √( )( )( )
GMD = 5.84m
R= x

R=0.7788
REGISTERED ELECTRICAL ENGINEERS PRE-BOARD EXAMINATION
MARCH 2016
PROFESSIONAL ELECTRICAL ENGINEERING SUBJECTS

GMR=0.7788x0.3175
GMR=0.247cm
L=(2x10-7) ln

L=1.55
XL=2 fL
=2 (60)(1.55x10-6)(1000)

XL= 0.586 ϕ

Answer: C. 0.586
31. Natural gas has an energy content of 0.036 watt- year/ ft3, and coal has an
average energy content of 640 watt-years/ton. If 2o percent of the net energy
requirement of 2.82x 108 GHWR were to be met with gas and 80 percent with
coal, what amount of gas would be required?

A. 1.79x106ft3 B. 1.79x108ft3 C1.79x1010ft3 D. 1.79x1012ft3

32. When one coil of a magnetically coupled pair has a current of 5A, the
resulting fluxes Ф11 Ф12 are 0 4mWb and 0 8mWb, respectively If turns are N1=
500 and N2= 1500, find L1.
A. 20mH B. 40mH C. 60mH D. 120Mh

SOLUTION:

L=
( )
=

L= 120 Mh

Answer: D. 120mH
REGISTERED ELECTRICAL ENGINEERS PRE-BOARD EXAMINATION
MARCH 2016
PROFESSIONAL ELECTRICAL ENGINEERING SUBJECTS

33. Three capacitances C1= 6uF, C2= 7uF and C3= 18uF are connected in
series across a 120v supply. Find the voltages across C3.
A. 1.826v B. 18.26v C. 54.78v D. 5.487v

SOLUTION:

C total = ((C1-1)+ (C2-1)+ (C3-1))-1

= ((6-1)+ (7-1)+ (18-1))-1

= 2.73uF

Q = CV

= (2.73uF)(120v)

= 327.6 uC

V = Q/ C

= 327.6 uC/ 18uF

= 18.26v

Answer: B. 18.26V

34. The specific resistance p is defined as


A. Resistance of a conductor which has a length of 1m and cross section of
1 m squared at 2o Celsius.
B. Resistance of any conductor at 25 celsius
C. Resistance of any conductor at 20 celsius
D. Resistance of a conductor which has a length of 1m and cros section of 1 cm
squared at 20 celsius.
REGISTERED ELECTRICAL ENGINEERS PRE-BOARD EXAMINATION
MARCH 2016
PROFESSIONAL ELECTRICAL ENGINEERING SUBJECTS

35. A 15 ohm resistor connected in series with an inductor has an equivalent


impedance of 30 ohms with an unknown angle. Find the power factor angle in
degrees.
A. 30 B. 60 C. 45 D. 75

SOLUTION:
Cos θ =

Θ = Cos-1 ( )

Θ= 60o

Answer: B. 60o

36. The master control center for a cellular telephone system in the
A. Cell site C. central office
B. Mobile telephone switching office D. branch office

37. The permeability of a material means


A. Strength of permanent magnet
B. Strength of an electromagnet
C. The magnetization left in the iron after exciting field has been removed.
D. The conductivity of a material for magnetic lines of force

38 A 10KVA, 200/400v, 1Ф transformer is operated with pen circuited


secondary. Find the primary current.
A. 2A B. 4A C.6A D.8A

39. An office has a total of 500 square meters. Find the total lighting load for a
unit of 28 volt-amperes per square meter.
REGISTERED ELECTRICAL ENGINEERS PRE-BOARD EXAMINATION
MARCH 2016
PROFESSIONAL ELECTRICAL ENGINEERING SUBJECTS

A. 8KVA B. 10KVA C. 12KVA D. 14KVA

SOLUTION:

Load= (28volt- ampere per square meter)(500 square meter)

= 14KVA

Answer: D. 14KVA

40. Is an instrument connected across the load?


A. Voltmeter B. ammeter C. ohmmeter D. wattmeter

41. A balanced delta load of 3+j4 ohms per phase is connected to a balanced
110v source. Find the line current.
A. 22A B. 38.1A C. 11A D. 19.05A

SOLUTION:

I = sq. rt of (V)/ Z

= sq.rt. of (110v)/ 3+j4 ohms)

= 38.1A

Answer: B. 38.1A

42. Transform the current given by 6+j8 A to its time domain.


A. 100 cos (wt + 53.13) C. 100 cos (wt + 36.78)
B. 10cos (wt + 53.13) D. 10 cos (wt + 36.78)
REGISTERED ELECTRICAL ENGINEERS PRE-BOARD EXAMINATION
MARCH 2016
PROFESSIONAL ELECTRICAL ENGINEERING SUBJECTS

SOLUTION:

I = 6 +j8 A

= 10 < 53.13

Answer: B. 10cos (wt + 53.13)

43. What is the equivalent phasor of i(t) = 20 cos(wt + 40) A?


A. 15.3- j2.9 A B. -15.3+ j12.9 A
C. 15.3 + j12.9 A D. -15.3 – j12.9 A

SOLUTION:

i(t) = 20 cos(wt + 40) A

i= 20< 40 A

= 15.3 + j12.9 A

Answer: C. 15.3 + j12.9 A

44. A parallel RL circuit draws a total current of 17A and 1.8KW from a 120V
supply. Find R.
A. 7 ohms B. 8 ohms C. 9 ohms D. 10 ohms

SOLUTION:

P = V2/ R
REGISTERED ELECTRICAL ENGINEERS PRE-BOARD EXAMINATION
MARCH 2016
PROFESSIONAL ELECTRICAL ENGINEERING SUBJECTS

1800 w = 1202/ R

R= 8 ohms

Answer: B. 8 ohms

45. What will happen to the current of a transformer when the temperature of the
transformers winding increases?
A. remains unchanged C. decreases
B. increases D. dramatically unchanged

46. A 6 ohms resistor is connected in series with an inductor L across a 110v,


60Hz source and draws of a current of 10A. Find L.
A. 25.5mH B. 26.5mH C. 23.5mH D. 24.5Mh

SOLUTION:

Z= V/ I

= 110v/ 10A

= 11 ohms

Z2= R2+ Xc2

112= 62 + Xc2

Xc= 9.21 ohms

L= 9.21/ 2pi(60)

= 24.5 mH

Answer: D. 24.5mH
REGISTERED ELECTRICAL ENGINEERS PRE-BOARD EXAMINATION
MARCH 2016
PROFESSIONAL ELECTRICAL ENGINEERING SUBJECTS

46. The type of emf induced by neighboring coils.


A. Self- induced C. dynamically induced
B. statically induced D. mutually induced

47. Transformer are use to change the value of the following EXCEPT.
A. Alternating current C. power
B. Alternating voltage D. resistance
48 The power factor of a 20 MW , 13 2 KV, 3Ф motor is corrected to 0 9 a 230uF
capacitor. What is its initial power factor
A. 0.628 B. 0.965 C. 0.765 D. 0.828

SOLUTION:

Ф1= tan-1 ((2pi x 60 x 230uF x (13.2KV/ sq. rt. Of 3))/ (20MW/ 3)) + tan 25.84

= 51. 10

pf= cos 51. 10

= 0.628

Answer: A. 0.628

49. Two 6- ohm resistors are connected in series. When a resistor R is parallel
with one of them, the total resistance is ohms. What is the value of R?
A. 5 ohms B. 6 ohms C. 7 ohms D. 8 ohms

SOLUTION:
REGISTERED ELECTRICAL ENGINEERS PRE-BOARD EXAMINATION
MARCH 2016
PROFESSIONAL ELECTRICAL ENGINEERING SUBJECTS

6+ (6R/ 6+R)= 9

R= 6 ohms

Answer: B. 6 ohms

50. A coil of wire wound, with or without a magnetic core designed to have a
higher self-inductance than a straight wire.
A. Toroid B. Solenoid C. Inductor D. Inductive Relay

51. What capacitance must be placed in series with an inductance of 0.05 henry,
so that when the frequency is 100 Hz, the impedance becomes equal to the
ohmic resistance?
A. 50.7 µF B. 31.8 µF C. 67.5 µF D. 42 µF

SOLUTION:

Fr=100 HZ

L=0.5H

Fr= 1/2pi√ 𝐶

100=1/2pi√( )(𝐶)

C=50.7 µF

Answer: A. 50.7 µF

52. When a dc motor has no load, what will happen to the back emf?
REGISTERED ELECTRICAL ENGINEERS PRE-BOARD EXAMINATION
MARCH 2016
PROFESSIONAL ELECTRICAL ENGINEERING SUBJECTS

A. Reduces C. Becomes Maximum


B. Increases D. Becomes Zero

53. A three-phase induction motor has a nameplate speed of 1,720 rpm. What is
the rotor frequency?
A. 1.8 Hz B. 2.7 Hz C. 5Hz D. 4.4 Hz

SOLUTION:

F= 60 Hz, Nr Fr=%S x f

%S= Fr=4.4% x60Hz

%S= FR = 2.7 Hz

%S=4.4%

Answer: B. 2.7 Hz

54. A 50 MW coal-fired power plant has an average heat rate of 11,000


BTU/kWHR. Coal has a heating value of 13,000 BTU per pound. The plant has a
load factor of 80%. How much coal is burned per day?
A. 8.1 x 105 lb B. 8.1 x 105 lb C. 1.2 x 105 lb D. 7.2 x 105 lb

SOLUTION:

( )( ) ( ) ( )( )

Answer: B. 8.1 x 10^5 lb/day


REGISTERED ELECTRICAL ENGINEERS PRE-BOARD EXAMINATION
MARCH 2016
PROFESSIONAL ELECTRICAL ENGINEERING SUBJECTS

55. Three identical resistors are connected in wye across 220 v, 3-phase lines.
The line current is12.7 A. To what value in volts should the line voltage be
changed the same line current with the resistors connected in delta?
A. 98.6 B. 132.8 C. 120 D. 73.3

SOLUTION:

VL Delta=12.7(10) = 127v

IL= 12.7A

Vphase = 73.32 v

Answer: D. 73.32 v

56. A sinusoidal current having peak value 7.07 A is superimposed on dc of 5 A.


If a hot-wire ammeter is used to measure the combined current in the circuit, it
will indicate ___ amperes.
A. 7.07 B. 10 C. 12.07 D. 2.07

57. A 4-way switch controls a lamp from _______ different locations.


A. 1 B. 2 C. 3 D. 4

58. When a V-V system is converted into Δ-Δ system, increase in the capacity of
the system is _____ percent.
A. 86.8 B. 73.2 C. 66.6 D. 50
REGISTERED ELECTRICAL ENGINEERS PRE-BOARD EXAMINATION
MARCH 2016
PROFESSIONAL ELECTRICAL ENGINEERING SUBJECTS

59. A small wind generator is designed to generate 50 kW of power at a wind


velocity of 25 miles per hour. What is the approximate blade diameter?
A. 9.98 m B. 12.98 m C. 11.98 m D. 10.98 m

60. A synchronous generator having a subtransient reactance of 0.15 pu and


operating at 5 percent above its rated voltage supplies a synchronous motor
having a 0.20 pu subtransient reactance. The motor is connected to the
generator by a transmission line and a transformer of total reactance 0.305 pu. A
sudden three-phase fault occurs at the generator terminals. Determine the per
unit subtransient fault current.
A. –j7.079 pu B. –j8.0789 pu C. –j9.079 pu D. –j10.079 pu

61. The capacitance between any two conductors of a three-phase, three-


conductor cable is 2 µF. The cable operates at 11 kV line voltage and 50 Hz.
What is the charging current through the cable capacitance?
A. 7.98 A B. 4 A C. 6.93 A D. 12 A

62. A meter has a full-scale current of 50 µA, what is its sensitivity?


2 Ω/ B 20 v/kΩ C 50 kΩ/v D. 50 µA/v

SOLUTION:
Sensitivity = 1/ IFL
= 1/ 50x10-6
= 20 kΩ/V

nsw : 2 Ω/
REGISTERED ELECTRICAL ENGINEERS PRE-BOARD EXAMINATION
MARCH 2016
PROFESSIONAL ELECTRICAL ENGINEERING SUBJECTS

63. The current-carrying capacity of a copper wire having twice the diameter of
another copper wire is ______________ as great.
A. Twice B. Half C. 4 Times D. 3 Times

SOLUTION:

D2=2d1

( )

Answer: C. 4 times

64. In a 10-pole synchronous machine, 20 electrical degrees are equivalent to


how many mechanical degrees?
A. 2 B. 8 C. 4 D. 10

SOLUTION:
Electrical angle = (P/2) x Mechanical angle
20= (10/2)x mechanical degree
Mechanical degree= 4
Answer: C. 4

65. You have 120 volts at the panel and 115 volts at the volts at the load. What is
the percentage voltage drop?
A. 5% B. 4.35% C. 4.17% D. 3%

SOLUTION:
REGISTERED ELECTRICAL ENGINEERS PRE-BOARD EXAMINATION
MARCH 2016
PROFESSIONAL ELECTRICAL ENGINEERING SUBJECTS

VD=

VD= 4.17 %

Answer: C. 4.17%

66. A delta 3ᶲ, 4-wire secondary 230/115 v would have a high-leg to neutral
voltage of ________.

A. 208 v B. 230 v C. 277 v D. 199 v

SOLUTION:

3 230/115

High-leg neutral = √ 115

= 119.19V

Answer: D. 199.19 v

67. The conductors of a three-phase transmission line are arranged in the form of
an equilateral triangle with sides of 6 m each. If the conductors are 500 mils in
diameter and the line is 25 km long, what is its inductance per phrase?
A. 35.5 mH B. 3.55 mH C. 71 mH D. 7.1 Mh
SOLUTION:
Dm= √( )( )( )=6m
Ds=r=500/2=250mils
REGISTERED ELECTRICAL ENGINEERS PRE-BOARD EXAMINATION
MARCH 2016
PROFESSIONAL ELECTRICAL ENGINEERING SUBJECTS

250 mils x 1 inch/1000mils x 1ft/12 inch x 1m/ 3.28ft= 6.3516x10 -3m


L=2x10-7 ln Dm/Ds= (2x10-7 ln (6/6.3516x10-3)(25000)=34.25Mh OR 34.Mh
Answer: A. 35.5mH

69. A 10-km long, single-phase short transmission line has0 5 < 60 Ω/km
impedance. The line supplies a 316.8 kW load at 0.8 power factor lagging. What
is the voltage regulation if the receiving and voltage is 3.3 kV?
A. 12.94 % B. 16.94% C. 14.94% D. 10.94%

SOLUTION:

Z =0.5 cis 60 = 0.25+j0.43


Z= R+jxL

= 0 25 j 43 Ω/km x 10k

Z= 2 5 j 4 3Ω

R= 2 5Ω XL=4 3Ω

Vr= 3.3 kv

Vs= √( ) ( )

I=
√ ( )( )

I=120A

Vs= √( ( )( )) ( ( )( ))

Vs=3856.63v

%VR=
REGISTERED ELECTRICAL ENGINEERS PRE-BOARD EXAMINATION
MARCH 2016
PROFESSIONAL ELECTRICAL ENGINEERING SUBJECTS

%VR=16.94%

Answer: B. 16.94%

70. The positive, negative, and zero sequence reactances of a 20-MVA, 13.2 KV
synchronous generator are 0.3 pu, 0.2 pu and 0.1 pu, respectively. The
generator is solidly grounded and is not loaded. A line to ground fault occurs on
phase a. neglecting all the resistances, determine the positive sequence of the
fault current.
A. 4,374A B.1,458A C.2,525 A D.7,576A

SOLUTION:
L-G fault

√ √ ( )
( ) ( )( )

= 4373.89
= 1/3 If
= 1/3(4373.89)
=1458A

Answer: 1458 A

71. Converts energy of water to mechanical energy.


A. pump B. turbine C. generator D. draft tube
REGISTERED ELECTRICAL ENGINEERS PRE-BOARD EXAMINATION
MARCH 2016
PROFESSIONAL ELECTRICAL ENGINEERING SUBJECTS

72. Calculate the capacitance of a parallel plate capacitor having 20 cm x 20


cm square plates separated by a distance of 1 mm. Assume the dielectric
medium to be air with permittivity of 8.85 x 10-12 F/m.
A. 354 uF B. 354 pF C. 3.54 uF D. 3.54 Pf

SOLUTION:

A= (.2x.2m) =0.4m

D= Imm =0.001m

∑ =8.85x10 F/m

∑ [( )( ) )]
𝐶

Answer: 354 pF

73. A current of 10 A is flowing in a flexible conductor of length 1.5 m. A force of


15 N acts on it when it is placed in uniform field of 2 T. Calculate the angle
between the magnetic field and the direction of the current.
A. 90° B. 60° C. 45° D. 30°

SOLUTION:
F=15N, I=10

B=2T L= 1.5m

F=BIL sin

15 = (2) n (10) (1.5) sin


REGISTERED ELECTRICAL ENGINEERS PRE-BOARD EXAMINATION
MARCH 2016
PROFESSIONAL ELECTRICAL ENGINEERING SUBJECTS

=30

Answer: 30°

74. If the waveform of a current has form factor 1.2 and peak factor 1.7, find the
average value of the current if the maximum value of the current is 100 A.
A. 45 A B. 47 A C. 49 A D. 51 A
SOLUTION:

Form factor =1.2


Peak factor = 1.7

Answer: C. 49.02

75. The applied voltage in a transformer is increased by 50 % and frequency is


reduced by 50 %. The maximum core flux density will become ______________.
A. same B. 1.5 times C. 3 times D. 5 times
REGISTERED ELECTRICAL ENGINEERS PRE-BOARD EXAMINATION
MARCH 2016
PROFESSIONAL ELECTRICAL ENGINEERING SUBJECTS

76. A device that establishes an electrical connection to the earth.


A. Grounding conductor C. Grounding electrode
B. Grounded conductor D. Grounding electrode conductor

77. A transformer has 4 % resistance and 6 % reactance drop. Find the voltage
regulation at full-load 0.8 p.f. lagging.
A. 6.8 % B. 5.8 % C. 4.8 % D. 160%

SOLUTION:

%VR= √( ) ( )

%VR= 6.8%
Answer: A. 6.8%

78. A 6-pole, 50 Hz, three-phase induction motor has a maximum torque of 200
N-m when it is running at a speed of 900 rpm. The resistance of the rotor is 0.25
Ω Neglecting stator impedance, determine the torque at 5 % slip
A. 180 N-m B. 140 N-m C. 150 N-m D. 160 N-m

79. In a certain country, the equivalent fuel reserve for power generation is 3 x
106 MW-yoars. The present peak power demand is 200 GW, and the expected
power consumption growth rate is 2.1 percent. How long will the fuel reserve
last?
A. 8 years B. 11 years C. 13 years D. 14 years

SOLUTION:
F= Pern
3*106 = (200*103)0.021n
REGISTERED ELECTRICAL ENGINEERS PRE-BOARD EXAMINATION
MARCH 2016
PROFESSIONAL ELECTRICAL ENGINEERING SUBJECTS

N= 13 years
Answer: C. 13 years

80. Which of the following is not a standard branch circuit among rating?
A. 15 A B. 25 A C. 30 A D. 40 A

81. Once residual magnetism of a shunt generator is lost, it may be restored by


connecting its shunt field.
A. to earth C. to an external battery
B. in reverse D. to an alternator

82. Power in balance 3-phase system is measured by the two-wattmeter method


and it is found that the ratio of the two-wattmeter readings is 2 is to 1. What is the
power factor of the system?
A. 0.9 B. 0.82 C. 0.866 D. 0.707

SOLUTION:
P1= 1 P2 =2

p.f = cos

[(√ )( )]
( )

= 30

p.f. = cos30° = 0.866

Answer: C. 0.866
REGISTERED ELECTRICAL ENGINEERS PRE-BOARD EXAMINATION
MARCH 2016
PROFESSIONAL ELECTRICAL ENGINEERING SUBJECTS

83. A coil with 40 ohm resistance and 0.02 H inductance and a pure capacitor
are connected in parallel across a 60 Hz supply. Find the value of the capacitor
in uF if the circuit is resonant.
A. 14.35 B. 12.07 C. 11.67 D. 15.82

SOLUTION:

F= 60 ; R=40 ; L=0.02H

Fr= √

( )
60= √ ( )

C = 12.07 µF

Answer: B. 12.07 µF

84. A balance delta connected load draws 100 A line current form a balanced 3-
phase supply. Determine the zero sequence components of the currents.
A. -50 – j86.6 B. 50 – j86.6 C. 86.6 – j50 D. 0

SOLUTION:
When delta connected (zero sequence)
Io= 0

Answer: D. 0
REGISTERED ELECTRICAL ENGINEERS PRE-BOARD EXAMINATION
MARCH 2016
PROFESSIONAL ELECTRICAL ENGINEERING SUBJECTS

85. Surge impedance of tansmission line is given by:

A √C/L B √LC C 1/√LC D √L/

86. In full-wave rectification, if the in-out frequency is 60 Hz, then output has a
frequency of ______
A. 120 Hz B. 60 Hz C. 30 Hz D. 15 Hz

SOLUTION:

Fout = 2fin

Fout = 2(60Hz) = 120Hz

Answer: A. 120 Hz

87. Determine the total inductance of a single-phase overhead transimission line


of 10km long. The diameter of each conductor os 0.8 cm and the distance
between them is 1.2 meters.
A. 11.9 mH B. 23.8 mH C. 35.7 mH D. 47.6 mH

SOLUTION:
L=4x1017 ln

=4x1017 ln ( )
=
2.3815x10-6(10,000m)
L=23.81 Mh
REGISTERED ELECTRICAL ENGINEERS PRE-BOARD EXAMINATION
MARCH 2016
PROFESSIONAL ELECTRICAL ENGINEERING SUBJECTS

Answer: B. 23.8 mH

88. A diversity factor of 2 gives a saving of ___ % in generating equipment.


A. 33 B. 50 C. 67 D. 75

SOLUTION:

dF = 2

dF (saving) = 100%

2 (saving) = 1

Saving = 50%

Answer: B. 50%

89. A room 8 m x 12 m i slighted by 15 lamps wo fairl uniform illumination of 100


lumens per square meter. What is the coefficient of utilization if the output of
each lamp is 1,600 lumens.
A. 40 % B. 60 % C. 30 % D. 50 %

SOLUTION:
100= (x*1600*15)/ (8*12)
X= 40%
Answer: A. 40%
REGISTERED ELECTRICAL ENGINEERS PRE-BOARD EXAMINATION
MARCH 2016
PROFESSIONAL ELECTRICAL ENGINEERING SUBJECTS

90. A 440 -v, 70- HP shunt motor has an ammature of0 185 Ω and field
resistance of 350 Ω The current drawn byy this machine is 135 A at full-load. If
this machine is to deliver torque equal to 175 percent of that at full-load, what
would be the best resistance of its starter?
A 34 Ω B 56 Ω C 22 Ω D 7 Ω

SOLUTION:
Ist= Vs/ (Ra+ Rst)
1.75(135) = 440/ (0.185+ Rst)
Rst= 1.67 ohms= 1.7ohms
Answer: D. 1.7 ohms

91.In a parallel Rc circuit, as the capcitance decreases, the power factor


___________.
A. Increases B. decreases
C. Remains the same D. Becomes zero

92. Find the amount of electrical energy expanded in raising the temperature of
45 liters of water by 75 C. Assume a heater efficiency of 90%.
A. 2.64 KWHR B. 3.46 KWHR C. 4.36 KWHR D.6.34 KWHR

SOLUTION:

M= 45L x

m=45kg
REGISTERED ELECTRICAL ENGINEERS PRE-BOARD EXAMINATION
MARCH 2016
PROFESSIONAL ELECTRICAL ENGINEERING SUBJECTS

Q=45kg x x 75degree C x ( )( )

Q = 4.36KWhr

Answer: C. 4.36 kWhr

93. A capacitor of 40 μF capacitance in series with 2,000 ohms is suddenly


connected across a 200-volt source. What is the energy stored in the capacitor
after 0.04 sec?
A. 0.124 J B. 0.241 J C. 0.412 J D. 2.41 J

SOLUTION:
W = ½ (Q2/C)
= ½ ((It)2/ C)
= ½ ((0.1*0.04)2/ 40uF)
= 0.2
Answer: B.0.241J

94. Calculate the average voltage generated in a six-turn full-pitch coil of a 25-
cycle alternator if the flux pole is 7.2 x 10^5 maxwells.
A. 7.4 V B. 8.64 V C. 1.08 V D. 4.32 V

SOLUTION:

F= 25Hz

= 7.2 x 10^5 maxwells


REGISTERED ELECTRICAL ENGINEERS PRE-BOARD EXAMINATION
MARCH 2016
PROFESSIONAL ELECTRICAL ENGINEERING SUBJECTS

N= 6

Eave = 4 f N x 10^-8

= (25) (7.2x 10^5) (6) x 10^-8

Eave= 4.32 V

Answer: D. 4.32 V

95. A 13.8 kV/440 v, 50 kVA single-phase transformer has a leakage reactance


of 300 ohms referred to the 13.8 kV side. Determine the per unit value of the
leakage reactance of the low base.
A. 0.305 B. 0.40 C. 0.097 D. 0.079

SOLUTION:
S= V2/R
= (13800)2/300
= 634800
Per unit= 50000/ 634800
= 0.079
Answer: D. 0.079

96. A 100 km transmission line has a 1,000 ohms shunt reactance. What is the
per km shunt reactance?
A 1,000 Ω B , Ω C 10 Ω D 100 Ω

SOLUTION:
REGISTERED ELECTRICAL ENGINEERS PRE-BOARD EXAMINATION
MARCH 2016
PROFESSIONAL ELECTRICAL ENGINEERING SUBJECTS

100 km,

R = 1,000Ω

Rsh = 1000Ω/km 100km)

Rsh = 100,000 Ω

nsw : B , Ω

97. A 5,000 kVA synchronous condenser operates with a leading p.f. of 0.032.
The losses are 180 kW. What is the power input to the motor?
A. 5,160 kW B. 5,320 kW C. 160 kW D. 320 kW

SOLUTION:
Cos ϴ= P/ S
0.032= P/ 5000000
P= 160 KW
Answer: C. 160KW

98. The current flowing in the armature conductors of a dc motor is


A. ac B. dc C. ac as well as dc D. Transients

99. The sequence components of phase c current of a 3-phase system are as


follows: Ic0=0, Ic1=18.4 cis88.4, Ic2=3.23 cis48.2. Find the phase b current in
amperes.
19.24 cis -218 B. 18 cis 42 C. 18 cid -42 D. 19.24 cis 218
REGISTERED ELECTRICAL ENGINEERS PRE-BOARD EXAMINATION
MARCH 2016
PROFESSIONAL ELECTRICAL ENGINEERING SUBJECTS

SOLUTION:
Ia1= (18.4<88.40) (1<120) = 18.4<- 151.6
Ia2= (3.23<48.2) (1<120) = 3.23<168.2
Ib= (18.4<- 151.6)(1<120)+ (3.23<168.2)(1<240)
= 19.23<-22 (1<240)
= 19.23<218
Answer: D. 19.24 cis 218

100. A current of 20 A is flowing i =n an RL series circuit the instant before the


switch is opened. If R = 10 Ω and L= 10 H, find the current after 3 second has
elapsed.
A. 1 A B. 2 A C. 3 A D. 4 A

SOLUTION:

I= (E/R)(1- e –R/L(t))
= (200/10)(1-e-10/10(3))
= 19A
I= 20-19
= 1A
Answer:A.1A
AUGUST 2016
REGISTERED ELECTRICAL ENGINEERS PRE-BOARD EXAMINATION
AUGUST 2016
PROFESSIONAL ELECTRICAL ENGINEERING SUBJECTS

1. The color yellow reserved for equipment grounding conductor is


A. Yellow B. Green C. White D. Gray

2. What is the emf induced across an inductor with 150mH inductance and
current of 4 amperes?

GIVEN: L=150mH, I=4A

SOLUTION:
Emf=LI
Emf=(150mH)(4A)
=0.6V
A. 0.6v B. 0v C. 1.2v D. 12v

3. Which of the following is not a full-duplex?


A. Radar B. Telephone C. Tolemetry D. Local Area Network

4. In a circuit breaker, the time duration from the instant of the fault extinction of
arc is known as
A. Operating time B. total clearing time C. Lag time D. lead time

5. A resistance of 6 ohms is connected in parallel with another resistor R across


a 120v supply. The total current drawn from a supply was found to be 40A.
What is the value of resistance R?
A. 1 ohm B. 2 ohms C. 4 ohms D. 6 ohms

GIVEN: R=6ohms, V=120V, I=40A

SOLUTION:

Z= V/ I

Z= 120V/ 40A = 30 ohms

Current in R1= V/ R

= 120/ 6= 20A

R2= V/ I

= 120V/ 20
= 6 ohms
REGISTERED ELECTRICAL ENGINEERS PRE-BOARD EXAMINATION
AUGUST 2016
PROFESSIONAL ELECTRICAL ENGINEERING SUBJECTS

Answer: D. 6 ohms

6. A 50 cm long is moved in a field of density 1Wb/m squared at a velocity of


30m/s. what is the emf induced if the motion is perpendicular to the field?
A. 150v B. 1.5v C.15v D. 0.15v

GIVEN:

B=1Wb/m v=30m/s
L=50cm

SOLUTION:

e= BLvsin Ф

= (1)(50x 10^-2)(30)(sin 90)

= 15v

Answer: C. 15v

7. Two capacitors C1= 50uF and C2= 30uF are connected in series. Find the
equivalent capacitance in uF.
A. 1,875 B. 187.5 C. 18.75 D. 1.875

GIVEN: C1= 50uF and C2= 30uF

SOLUTION:

C total = ((C1-1) + (C2-1))-1)

= ((50uF-1) + (30uF-1))-1)

= 18.75 uF

8. 34.5 KV to 69 KV is classified as
A. Low voltage B. Medium voltage C. High voltage D. Extra high vol.

Answer: According to Phillipine Distribution Code, A Voltage exceeding 34.5


Kv upto 230kV is considered as High Voltage

C. High Voltage

9. Wavelength is the distance travelled by an electronic wave during the time of


one cycle. Given a wavelength of 12 meters, what is the frequency?
REGISTERED ELECTRICAL ENGINEERS PRE-BOARD EXAMINATION
AUGUST 2016
PROFESSIONAL ELECTRICAL ENGINEERING SUBJECTS

A. 250 KHZ B. 25KHz C. 250MHZ D. 25MHZ

GIVEN:
VC=3x108 m/s (speed of light)
Wavelength=12m

SOLUTION:
F = VC/ wavelength
= (3x108) / 12
= 25MHZ

10. A 4 pole dc generator has total of 600 conductors in its lap wound armature.
The flux per pole is 108 lines. Determine the generated emf if it is driven at
2400 rpm.
A. 230v B. 254v C. 234v D. 240v

GIVEN: P=4, Z=600, O=105 , N=2400

SOLUTION:
Eg= PZON/ 60a
= ((4)(600)(105)(2400)/ 60(4))* 10-8
= 240V
Answer: D. 240V

11. For an RL circuit, the power factor cannot be less than ______ or greater than
_____.
A. 0,1 B. 1,0 C. 0, -1 D. -1, 0

Explanation : Power factor is the ration of the actual electrical power


dissipated by an AC circuit to the product of RMS values of its current and
voltage. Since you can never use more power than what is flowing through
your circuit, this ratio or power factor can never be more than 1 and less than
0.

12. In a certain country, the peak power demand for year 2005 is 9027 MW and
by year 2014 it is estimated to be 10064 MW. Calculate the growth rate.
A. 0.9 % B. 0.8% C. 0.76% D. 0.85%

GIVEN : (From Continuously Compounding Periods topic)


REGISTERED ELECTRICAL ENGINEERS PRE-BOARD EXAMINATION
AUGUST 2016
PROFESSIONAL ELECTRICAL ENGINEERING SUBJECTS

P = 9207 (present value)


f = 10,064 (future value)
r = ? (periodic interest rate)
n = 2014-2005 = 9 (number periods)
Find r.

SOLUTION :

F= Pern
10,064= 9287er(9)
r= 0.89 %
Answer: A. 0.9%

13. __________ is the overall energy program formulated and updated yearly by
the DOE, submitted to the congress pursuant to RA 7638.
A. Philippine Energization Program C. Philippine Electrification Program
B. Philippine Energy Plan D. Philippine Electrification Plan

14. A certain amount of fuel can be converted into 3x10 quads of energy. Assume
30% overall efficiency for the power plant. If the average load on the power
plant over a 24 hr period is 60MW, how long in days the fuel will last?
A. 180 B. 175 C. 183 D. 185

SOLUTION :
1 Quad = 2.93x10^9 Mwh
0.003 Quad = 8.79x10^5

In 24 hrs: 60 x 24 = 1440 Mwh


At 30% efficiency : 1440/0.3 = 4800 Mwh
Hence, fuel will be consumed in : 8.79x10^5/4800 = 183.125 days

15. The lighting load for a dwelling expressed in terms of a unit load in volt
ampere per square meter must be at least _______.
A. 24 B. 12 C. 16 D. 2

16. The capacitors of power factor correction are rated in terms of


A. KV B. KW C. KVA D. KVAR\
REGISTERED ELECTRICAL ENGINEERS PRE-BOARD EXAMINATION
AUGUST 2016
PROFESSIONAL ELECTRICAL ENGINEERING SUBJECTS

Explanation : KVAR means Kilo Volt Ampere Reactive. Leading Reactive


Power is produced by synchronous Capacitors and Capacitor banks.
Consumed by Capacitive nature of Loads.

17. Find the maximum power drawn from a 12v battery its internal resistance is
0.25.
72W B. 144W C. 576W D. 36W

SOLUTION:

P= V2/ 4R

= (12v)2/ 4(0.25)

= 144W

Answer: B. 144W

18. Two identical coupled coils have an equivalent inductance of 80mH when
connected in series aiding and 35mH in series opposing. What is the value of
the coefficient of coupling K.
A. 0.391 B. 0.319 C. 0.193 D. 0.139

SOLUTION:

M = (Lta – Lto)/ 4

= (80 – 35)/ 4

= 11.25 mH

Lta= 2L+ 2M

80= 2L+ 2(11.25)

L= 28.75mH

K= M/ sq. rt of (L1L2)

= 11.25/ sq. rt of (28.752)

= 0.391
REGISTERED ELECTRICAL ENGINEERS PRE-BOARD EXAMINATION
AUGUST 2016
PROFESSIONAL ELECTRICAL ENGINEERING SUBJECTS

Answer: A. 0.391

19. What is the emf induced in a coil of 500 turns and with a constant flux of 10 8
maxwells?
A. 0v B. 5v C. 50v D. 500v

Explanation : According to Faraday’s Law of induction, an electromotive


force needs a rate of change of flux (dB/dt) to be induced. Since flux is
constant, the induced emf is zero.

20. In a balanced three phase wye connected circuit, the line voltage are equal
A. To the line current C. and so are the line currents
B. To the phase voltage D. but the line currents are unequal

21. In a given circuit when the power factor is unity the reactive power is
A. Maximum B. equal to i2r C. zero D. minimum

22. What is the branch ampacity of a hermectic motor with a full load current of
16A?
A. 15A B. 20A C. 25A D. 30A

Solution:

Ampacity = Current full load x 125%

= 16 x 1.25

= 20A

Answer: B. 20A

23. A three phase wye delta connected, 50MVA, 345/ 34.5 Kv transformer is
protected by differential protection. The current transformer on the high side
for differential protection is 150:5. What is the current on the secondary side
of CT’s?
A. 3.83A B. 2.53A C. 4.50A D. 4.83A

Solution:
REGISTERED ELECTRICAL ENGINEERS PRE-BOARD EXAMINATION
AUGUST 2016
PROFESSIONAL ELECTRICAL ENGINEERING SUBJECTS

IL-Primary = = 83.67A

Since wye connected , IL-Primary = IØ-Primary = 83.67A

a=

IØ-Secondary = 2.789A

IL-secondary = √ x 2.789A

IL-secondary = 4.83A

Answer: D. 4.83A

24. Joule found out that the heat produced in a current carrying conductor is
proportional to
A. The square of the current C. the square of resistance
B. The current D. inversely proportional to time

25. A conductor has four identical strands arranged in diamond configuration.


Find the GMR of the conductor in terms of the radius r of each strand.
A. 1.6921r B. 1.7037r C. 1.3008r D. 1.3052r

Solution:

d1 = 2r = d2 = d3, d4 = 2√ r

GMR = √

GMR = √( ) ( √ )

GMR = 1.6921r

Answer: A. 1.6921r
REGISTERED ELECTRICAL ENGINEERS PRE-BOARD EXAMINATION
AUGUST 2016
PROFESSIONAL ELECTRICAL ENGINEERING SUBJECTS

26. The resistances of the same length of aluminum and copper wires are found
to be equal. Find the ratio of radii of aluminum to copper.
A. 0.78 B. 1.29 C. 0.88 D. 1.14

Solution:

ρaluminum = 17 , ρcopper = 10.37

R= therefore r = √


Answer: B. 1.29

27. In a wye connected system, line current is


A. 0.707 times the phase current C. equal to the phase current
B. 1.735 times the phase current D. 1.414 the phase current

28. What is the device function number for overcurrent relay?


A. 27 B. 67 C. 50 D. 51

29. In a series RL circuit, the inductor current ________ the resistor current.
A. Tags B. leads C. is equal to D. is greater than

30. A three phase, 60Hz, transmission line has its conductors arranged in a
triangular configuration so that the two distances between conductors are 5m
and the third is 8m. the conductors have an outside diameter of 250 mils. Find
the inductive reactance in ohm per km per phase of the transmission line.
A. 0.567 B. 0.283 C. 0.586 D. 0.293

Solution:

d = 250mils x = 6.35x

r= = 3.175
REGISTERED ELECTRICAL ENGINEERS PRE-BOARD EXAMINATION
AUGUST 2016
PROFESSIONAL ELECTRICAL ENGINEERING SUBJECTS

GMR = 0.7788r = 0.7788(3.175 ) = 2.47

GMD = √( )( )( ) = 5.848m

L = (2 x ) ln( )

= (2 x ) ln ( )

= 1.55x

XL = 2πfL

XL = 2π(60)( 1.55x )(1000)

XL = 0.586 /Ø

Answer: C. 0.586

31. Natural gas has an energy content of 0.036 watt- year/ ft3, and coal has an
average energy content of 640 watt-years/ton. If 2o percent of the net energy
requirement of 2.82x 108 GHWR were to be met with gas and 80 percent with
coal, what amount of gas would be required?

A. 1.79x106ft3 B. 1.79x108ft3 C1.79x1010ft3 D. 1.79x1012ft3

32. When one coil of a magnetically coupled pair has a current of 5A, the
resulting fluxes Ф11 Ф12 are 0 4mWb and 0 8mWb, respectively If turns are
N1= 500 and N2= 1500, find L1.
A. 20mH B. 40mH C. 60mH D. 120mH\

Solution :

Ф11= 4mWB
Ф12= 8mWB
Ф1= Ф11 Ф12
= .4 + .8
Ф1= 1 2mWB`
Answer: D. 120mH
REGISTERED ELECTRICAL ENGINEERS PRE-BOARD EXAMINATION
AUGUST 2016
PROFESSIONAL ELECTRICAL ENGINEERING SUBJECTS

33. Three capacitances C1= 6uF, C2= 7uF and C3= 18uF are connected in
series across a 120v supply. Find the voltages across C3.
A. 1.826v B. 18.26v C. 54.78v D. 5.487v

SOLUTION:

C total = ((C1-1)+ (C2-1)+ (C3-1))-1

= ((6-1)+ (7-1)+ (18-1))-1

= 2.73uF

Q = CV

= (2.73uF)(120v)

= 327.6 uC

V = Q/ C

= 327.6 uC/ 18uF

= 18.26v

Answer: B. 18.26V

34. The specific resistance p is defined as


A. Resistance of a conductor which has a length of 1m and cross section
of 1 m squared at 2o Celsius.
B. Resistance of any conductor at 25 celsius
C. Resistance of any conductor at 20 celsius
D. Resistance of a conductor which has a length of 1m and cros section of 1 cm
squared at 20 celsius.

35. A 15 ohm resistor connected in series with an inductor has an equivalent


impedance of 30 ohms with an unknown angle. Find the power factor angle in
degrees.
A. 30 B. 60 C. 45 D. 75

36. The master control center for a cellular telephone system in the
A. Cell site C. central office
B. Mobile telephone switching office D. branch office
REGISTERED ELECTRICAL ENGINEERS PRE-BOARD EXAMINATION
AUGUST 2016
PROFESSIONAL ELECTRICAL ENGINEERING SUBJECTS

37. The permeability of a material means


A. Strength of permanent magnet
B. Strength of an electromagnet
C. The magnetization left in the iron after exciting field has been removed.
D. The conductivity of a material for magnetic lines of force

38. A 10KVA, 200/400v, 1Ф transformer is operated with pen circuited secondary


Find the primary current.
A. 2A B. 4A C.6A D.8A

Solution :

S=VI
I=10KVA/4KV
I= 2.5A

Answer: A. 2A

39. An office has a total of 500 square meters. Find the total lighting load for a
unit of 28 volt-amperes per square meter.
A. 8KVA B. 10KVA C. 12KVA D. 14KVA

SOLUTION:

Load= (28volt- ampere per square meter)(500 square meter)

= 14KVA

Answer: D. 14KVA

40. Is an instrument connected across the load?


A. Voltmeter B. ammeter C. ohmmeter D. wattmeter

41. A balanced delta load of 3+j4 ohms per phase is connected to a balanced
110v source. Find the line current.

A. 22A B. 38.1A C. 11A D. 19.05A

SOLUTION:
REGISTERED ELECTRICAL ENGINEERS PRE-BOARD EXAMINATION
AUGUST 2016
PROFESSIONAL ELECTRICAL ENGINEERING SUBJECTS

I = sq. rt of (V)/ Z

= sq.rt. of (110v)/ 3+j4 ohms)

= 38.1A

Answer: B. 38.1A

42. Transform the current given by 6+j8 A to its time domain.

A. 100 cos (wt + 53.13) B. 10cos (wt + 53.13) C. 100 cos (wt + 36.78) D. 10
cos (wt + 36.78)

What is the equivalent phasor of i(t) = 20 cos(wt + 40) A?

A. 15.3- j2.9 A B. -15.3+ j12.9 A C. 15.3 + j12.9 A D. -15.3 – j12.9 A

SOLUTION: SOLUTION:

i(t) = 20 cos(wt + 40) A I = 6 +j8 A

= 10 < 53.13 i= 20< 40 A = 15.3 + j12.9 A

Answer: B. 10cos (wt + 53.13) Answer: C. 15.3 + j12.9 A

43. A parallel RL circuit draws a total current of 17A and 1.8KW from a 120V
supply. Find R.

A. 7 ohms B. 8 ohms C. 9 ohms D. 10 ohms

SOLUTION:

P = V2/ R

1800 w = 1202/ R

R= 8 ohms

Answer: B. 8 ohms
REGISTERED ELECTRICAL ENGINEERS PRE-BOARD EXAMINATION
AUGUST 2016
PROFESSIONAL ELECTRICAL ENGINEERING SUBJECTS

44. What will happen to the current of a transformer when the temperature of the
transformers winding increases?

A. remains unchanged B. increases C. decreases D.


dramatically unchanged

46. A 6 ohms resistor is connected in series with an inductor L across a 110v,


60Hz source and draws of a current of 10A. Find L.

A. 25.5mH B. 26.5mH C. 23.5mH D. 24.5Mh

SOLUTION:

Z= V/ I

= 110v/ 10A

= 11 ohms

Z2= R2+ Xc2

112= 62 + Xc2

Xc= 9.21 ohms

L= 9.21/ 2pi(60)

L= 24.5 Mh

Answer: D. 24.5 Mh

47. The type of emf induced by neighboring coils.

A. Self- induced C. dynamically induced B.statically induced


D. mutually induced

48. Transformer are use to change the value of the following EXCEPT.

A. Alternating current B. Alternating voltage C. power D. resistance


REGISTERED ELECTRICAL ENGINEERS PRE-BOARD EXAMINATION
AUGUST 2016
PROFESSIONAL ELECTRICAL ENGINEERING SUBJECTS

49 The power factor of a 20 MW , 13 2 KV, 3Ф motor is corrected to 0 9 a 230uF


capacitor. What is its initial power factor

A. 0.628 B. 0.965 C. 0.765 D. 0.828

SOLUTION:

Ф1= tan-1 ((2pi x 60 x 230uF x (13.2KV/ sq. rt. Of 3))/ (20MW/ 3)) + tan 25.84

= 51. 10

pf= cos 51. 10

= 0.628

Answer: A. 0.628

50. Two 6- ohm resistors are connected in series. When a resistor R is parallel
with one of them, the total resistance is ohms. What is the value of R?

A. 5 ohms B. 6 ohms C. 7 ohms D. 8 ohms

SOLUTION:

6+ (6R/ 6+R)= 9

R= 6 ohms

Answer: B. 6 ohms

46. A coil of wire wound, with or without a magnetic core designed to have a
higher self-inductance than a straight wire.
A. Toroid B. Solenoid C. Inductor D. Inductive Relay

47. What capacitance must be placed in series with an inductance of 0.05 henry,
so that when the frequency is 100 Hz, the impedance becomes equal to the
ohmic resistance?

A. 50.7 µF B. 31.8 µF C. 67.5 µF D. 42 µF


REGISTERED ELECTRICAL ENGINEERS PRE-BOARD EXAMINATION
AUGUST 2016
PROFESSIONAL ELECTRICAL ENGINEERING SUBJECTS

SOLUTION:

Fr=100 HZ

L=0.5H

Fr= 1/2pi√ 𝐶

100=1/2pi√( )(𝐶)

C=50.7 µF

Answer: A. 50.7 µF

48. When a dc motor has no load, what will happen to the back emf?
A. Reduces C. Becomes Maximum
B. Increases D. Becomes Zero

49. A three-phase induction motor has a nameplate speed of 1,720 rpm. What is
the rotor frequency?

A. 1.8 Hz B. 2.7 Hz C. 5Hz D. 4.4 Hz

SOLUTION:

F= 60 Hz, Nr Fr=%S x f

%S= Fr=4.4% x60Hz

%S= FR = 2.7 Hz

%S=4.4%

Answer: B. 2.7 Hz

50. A 50 MW coal-fired power plant has an average heat rate of 11,000


BTU/kWHR. Coal has a heating value of 13,000 BTU per pound. The plant
has a load factor of 80%. How much coal is burned per day?
A. 8.1 x 105 lb B. 8.1 x 105 lb C. 1.2 x 105 lb D. 7.2 x 105 lb
REGISTERED ELECTRICAL ENGINEERS PRE-BOARD EXAMINATION
AUGUST 2016
PROFESSIONAL ELECTRICAL ENGINEERING SUBJECTS

SOLUTION:

( )( ) ( ) ( )( )

Answer: B. 8.1 x 10^5 lb/day

51. Three identical resistors are connected in wye across 220 v, 3-phase lines.
The line current is12.7 A. To what value in volts should the line voltage be
changed the same line current with the resistors connected in delta?
A. 98.6 B. 132.8 C. 120 D. 73.3

SOLUTION:

VL Delta=12.7(10) = 127v

IL= 12.7A

Vphase = 73.32 v

Answer: D. 73.32 v

52. A sinusoidal current having peak value 7.07 A is superimposed on dc of 5 A.


If a hot-wire ammeter is used to measure the combined current in the circuit,
it will indicate ___ amperes.
A. 7.07 B. 10 C. 12.07 D. 2.07

53. A 4-way switch controls a lamp from _______ different locations.


A. 1 B. 2 C. 3 D. 4

54. When a V-V system is converted into Δ-Δ system, increase in the capacity of
the system is _____ percent.
A. 86.8 B. 73.2 C. 66.6 D. 50

55. A small wind generator is designed to generate 50 kW of power at a wind


velocity of 25 miles per hour. What is the approximate blade diameter?
A. 9.98 m B. 12.98 m C. 11.98 m D. 10.98 m
REGISTERED ELECTRICAL ENGINEERS PRE-BOARD EXAMINATION
AUGUST 2016
PROFESSIONAL ELECTRICAL ENGINEERING SUBJECTS

SOLUTIONS

P=1/2pAv^3CP convert miles per hour into meter per


second

P=Power 25 miles per hour=11.176 meter per second

p=Air Density solving for diameter

A=Area(pi(r)^2) or (piD^2/4) P=1/2pAv^3CP

v^3= Velocity =½ (1.226) (11.176)^3(piD^2/4)(.59)

CP=Power Coefficient =11=10.98

51. A synchronous generator having a subtransient reactance of 0.15 pu and


operating at 5 percent above its rated voltage supplies a synchronous
motor having a 0.20 pu subtransient reactance. The motor is connected
to the generator by a transmission line and a transformer of total
reactance 0.305 pu. A sudden three-phase fault occurs at the generator
terminals. Determine the per unit subtransient fault current.

A. –j7.079 pu B. –j8.0789 pu C. –j9.079 pu D. –j10.079 pu

SOLUTION:

Using Thevenin’s; @ gen


)( )
Zth =

If = Vf/Zth = 1.05/j0.1156 = -j9.079 pu

Answer: C. –j9.079

52. The capacitance between any two conductors of a three-phase, three-


conductor cable is 2 µF. The cable operates at 11 kV line voltage and 50
Hz. What is the charging current through the cable capacitance?

A. 7.98 A B. 4 A C. 6.93 A D. 12 A
REGISTERED ELECTRICAL ENGINEERS PRE-BOARD EXAMINATION
AUGUST 2016
PROFESSIONAL ELECTRICAL ENGINEERING SUBJECTS

SOLUTION:

CLN = 2CLL

CLN = 2 (2x10^-6) = 4x10^-6

XC = 1/2πfC

Xc = 1/2π 50 4x10^-6 = 795 775 Ω

I = VLN/XC


I = 7.98 A

Answer: A. 7.98 A

53. A meter has a full-scale current of 50 µA, what is its sensitivity?


A. 2 Ω/ B 20 v/kΩ C 50 kΩ/v D. 50 µA/v

SOLUTION:

S = 1/I

S = 1/50x10^-6 = 20kΩ/V

Answer: A. 2 Ω/

54. The current-carrying capacity of a copper wire having twice the diameter
of another copper wire is ______________ as great.
A. Twice B. Half C. 4 Times D. 3 Times

SOLUTION:

D2=2d1

( )

Answer: C. 4 times

55. In a 10-pole synchronous machine, 20 electrical degrees are equivalent


to how many mechanical degrees?
REGISTERED ELECTRICAL ENGINEERS PRE-BOARD EXAMINATION
AUGUST 2016
PROFESSIONAL ELECTRICAL ENGINEERING SUBJECTS

A. 2 B. 8 C. 4 D. 10

SOLUTION:

E degree = P/2 x M degree

( )=4

Answer: C. 4

56. You have 120 volts at the panel and 115 volts at the volts at the load.
What is the percentage voltage drop?
A. 5% B. 4.35% C. 4.17% D. 3%

SOLUTION:

VD=

VD= 4.17 %

Answer: C. 4.17%

57. A delta 3ᶲ, 4-wire secondary 230/115 v would have a high-leg to neutral
voltage of ________.

A. 208 v B. 230 v C. 277 v D. 199 v

SOLUTION:

3 230/115

High-leg neutral = √ 115

= 119.19V

Answer: D. 199.19 v

58. The conductors of a three-phase transmission line are arranged in the


form of an equilateral triangle with sides of 6 m each. If the conductors
are 500 mils in diameter and the line is 25 km long, what is its inductance
per phrase?
A. 35.5 mH B. 3.55 mH C. 71 mH D. 7.1 Mh

SOLUTION:
REGISTERED ELECTRICAL ENGINEERS PRE-BOARD EXAMINATION
AUGUST 2016
PROFESSIONAL ELECTRICAL ENGINEERING SUBJECTS

= 0.0127 m
r = 0.0127/2 = 6.35x10^-3

r’ = 0 7788r = 4 9454x10^-3

Length = 25km = 25,000 m

L per-phase = 2x10^-7 ( )( )

L per-phase = 2x10^-7 (( ))( ) = 35.5 mH

Answer: A. 35.5 mH

59. A 10-km long, single-phase short transmission line has0 5 < 60 Ω/km
impedance. The line supplies a 316.8 kW load at 0.8 power factor
lagging. What is the voltage regulation if the receiving and voltage is 3.3
kV?
A. 12.94 % B. 16.94% C. 14.94% D. 10.94%

SOLUTION:

Z =0.5 cis 60 = 0.25+j0.43


Z= R+jxL

= 0 25 j 43 Ω/km x 10k

Z= 2 5 j 4 3Ω

R= 2 5Ω XL=4 3Ω

Vr= 3.3 kv

Vs= √( ) ( )

I=
√ ( )( )

I=120A

Vs= √( ( )( )) ( ( )( ))
REGISTERED ELECTRICAL ENGINEERS PRE-BOARD EXAMINATION
AUGUST 2016
PROFESSIONAL ELECTRICAL ENGINEERING SUBJECTS

Vs=3856.63v

%VR=

%VR=16.94%

Answer: B. 16.94%

60. The positive, negative, and zero sequence reactances of a 20-MVA, 13.2
KV synchronous generator are 0.3 pu, 0.2 pu and 0.1 pu, respectively.
The generator is solidly grounded and is not loaded. A line to ground fault
occurs on phase a. neglecting all the resistances, determine the positive
sequence of the fault current.

SOLUTION:
L-G fault

√ √ ( )
( ) ( )( )

= 4373.89
= 1/3 If
= 1/3(4373.89)
=1458A

Answer: 1458 A

71. Converts energy of water to mechanical energy.


A. pump B. turbine C. generator D. draft tube

72. Calculate the capacitance of a parallel plate capacitor having 20 cm x 20


cm square plates separated by a distance of 1 mm. Assume the dielectric
medium to be air with permittivity of 8.85 x 10-12 F/m.
A. 354 uF B. 354 pF C. 3.54 uF D. 3.54 Pf

Solution:
A = (20x10-2 m) (20x10-2 m) d = 1x10-3m
= 0.04m2 Σo = 8.85x10-12 F/m
REGISTERED ELECTRICAL ENGINEERS PRE-BOARD EXAMINATION
AUGUST 2016
PROFESSIONAL ELECTRICAL ENGINEERING SUBJECTS

1x10-3
C= = 354pF
-12
= (8.85x10 )(1)(0.04)

73. A current of 10 A is flowing in a flexible conductor of length 1.5 m. A force of


15 N acts on it when it is placed in uniform field of 2 T. Calculate the angle
between the magnetic field and the direction of the current.
A. 90° B. 60° C. 45° D. 30°

Solution:
I = 10A
L = 1.5m F = βILsinØ
F= 15N 15 = (2)(10)(1.5)sinØ
β = 2T Ø ˚

74. If the waveform of a current has form factor 1.2 and peak factor 1.7, find the
average value of the current if the maximum value of the current is 100 A.
A. 45 A B. 47 A C. 49 A D. 51 A

Solution:
F = 1.2
P = 1.7
Imax = 100A
P= ;
RMS = 58.82A
F= ;
AVE = 49A

75. The applied voltage in a transformer is increased by 50 % and frequency is


reduced by 50 %. The maximum core flux density will become
______________.
A. same B. 1.5 times C. 3 times D. 5 times

76. A device that establishes an electrical connection to the earth.


REGISTERED ELECTRICAL ENGINEERS PRE-BOARD EXAMINATION
AUGUST 2016
PROFESSIONAL ELECTRICAL ENGINEERING SUBJECTS

A. Grounding conductor C. Grounding electrode


B. Grounded conductor D. Grounding electrode conductor

77. A transformer has 4 % resistance and 6 % reactance drop. Find the voltage
regulation at full-load 0.8 p.f. lagging.
A. 6.8 % B. 5.8 % C. 4.8 % D. 160%
Solution:
%R = 4% = 0.04
%X = 6% = 0.06
pf = 0.8, lagging
θ = 36 87˚
%VR = √( ) ( )
%VR = √( ) [ ( ) ]
%VR = 6.8%

78. A 6-pole, 50 Hz, three-phase induction motor has a maximum torque of 200
N-m when it is running at a speed of 900 rpm. The resistance of the rotor is
0 25 Ω Neglecting stator impedance, determine the torque at 5 % slip.
A. 180 N-m B. 140 N-m C. 150 N-m D. 160 N-m

79. In a certain country, the equivalent fuel reserve for power generation is 3 x
106 MW-yoars. The present peak power demand is 200 GW, and the
expected power consumption growth rate is 2.1 percent. How long will the
fuel reserve last?
A. 8 years B. 11 years C. 13 years D. 14 years
Solution:
F = Pern
3x106 = (200x109)0.021n
n = 13 years
80. Which of the following is not a standard branch circuit among rating?
A. 15 A B. 25 A C. 30 A D. 40 A

81.Once residual magnetism of a shunt generator is lost, it may be restored by


connecting its shunt field.
A. to earth C. to an external battery
B. in reverse D. to an alternator

EXPLANATION:
REGISTERED ELECTRICAL ENGINEERS PRE-BOARD EXAMINATION
AUGUST 2016
PROFESSIONAL ELECTRICAL ENGINEERING SUBJECTS

Absolutely it can be restored provided the core material has not suffered
degradation. Excessive heating can do much damage. Re-exciting the
magnetic poles suing a dc exciter (DC power source) can restore the
residual magnetism required for starting. External Battery is a DC power
source.

82. Power in balance 3-phase system is measured by the two-wattmeter


method and it is found that the ratio of the two-wattmeter readings is 2 is
to 1. What is the power factor of the system?
A. 0.9 B. 0.82 C. 0.866 D. 0.707
SOLUTION:
P1= 1 ; P2= 2 ; pf= cos θ

θ= 30°
p.f. = cos30°
p.f. = 0.866

83. A coil with 40 ohm resistance and 0.02 H inductance and a pure capacitor
are connected in parallel across a 60 Hz supply. Find the value of the
capacitor in uF if the circuit is resonant.
A. 14.35 B. 12.07 C. 11.67 D. 15.82
SOLUTION:
F= 60 ; R= 40 ; L= 0.02H

Formula of Resonant Frequency:

C = 12.07 uF

84. A balance delta connected load draws 100 A


line current form a balanced 3-phase supply. Determine the zero
sequence components of the currents.
A. -50 – j86.6 B. 50 – j86.6 C. 86.6 – j50 D. 0
SOLUTION:
(100 )
Iao= 0 = Ibo = Ico
85. Surge impedance of transmission line is given by:
A √C/L B √LC C 1/√LC D √L/
SOLUTION:
REGISTERED ELECTRICAL ENGINEERS PRE-BOARD EXAMINATION
AUGUST 2016
PROFESSIONAL ELECTRICAL ENGINEERING SUBJECTS

Upon Simplifying:

Where,
f = Frequency of the system
L = Inductance per unit length of the line
l = Length of the line
Hence we get,

86. In full-wave rectification, if the in-out frequency is 60 Hz, then output has a
frequency of ______
A. 120 Hz B. 60 Hz C. 30 Hz D. 15 Hz
SOLUTION:
t= t/2
So the frequency,
f= 2f
f= 2(60)
f= 120 Hz

87. Determine the total inductance of a single-phase overhead transmission


line of 10km long. The diameter of each conductor of 0.8 cm and the
distance between them is 1.2 meters.
A. 11.9 mH B. 23.8 mH C. 35.7 mH D. 47.6 mH
SOLUTION:
( )
( )( )( )

LT = 23.8 mH

88. A diversity factor of 2 gives a saving of ___ % in generating equipment.


A. 33 B. 50 C. 67 D. 75
SOLUTION:
df = 2

df (saving) = 100%

2 (saving) = 1

saving= 1/2 x 100

saving = 50 %

89. A room 8 m x 12 m i slighted by 15 lamps wo fairl uniform illumination of


100 lumens per square meter. What is the coefficient of utilization if the
output of each lamp is 1,600 lumens.
A. 40 % B. 60 % C. 30 % D. 50 %
SOLUTION:
100= (x*1600*15) / (8*12)
X= 40%

90. A 440 -v, 70- HP shunt motor has an ammature of0 185 Ω and field
resistance of 350 Ω The current drawn by this machine is 135 A at full-
load. If this machine is to deliver torque equal to 175 percent of that at full-
load, what would be the best resistance of its starter?
REGISTERED ELECTRICAL ENGINEERS PRE-BOARD EXAMINATION
AUGUST 2016
PROFESSIONAL ELECTRICAL ENGINEERING SUBJECTS

A 34Ω B 56Ω C 22Ω D 7Ω

SOLUTION:

IST= Vs / (Ra+ RST)


1.75(135) = 440 / (0.185+ RST)
RST = 1.67 ohms
RST = 1.7 ohms

91. In a parallel Rc circuit, as the capcitance decreases, the power factor


___________.

A. Increases B. decreases C. Remains the same D. Becomes zero

92. Find the amount of electrical energy expanded in raising the temperature
of 45 liters of water by 75 C. Assume a heater efficiency of 90%.
A. 2.64 KWHR B. 3.46 KWHR C. 4.36 KWHR D.6.34 KWHR

SOLUTION:

M= 45L x

m=45kg

Q=45kg x x 75degree C x ( )( )

Q = 4.36KWhr

Answer: C. 4.36 kWhr

93. A capacitor of 40 μF capacitance in series with 2,000 ohms is suddenly


connected across a 200-volt source. What is the energy stored in the
capacitor after 0.04 sec?
A. 0.124 J B. 0.241 J C. 0.412 J D. 2.41 J

SOLUTION:
Ec=200 (1- e^-t/RC)
Ec=200 (1- e^-.04/(2000)(40x10^-6)
Ec=78.69 V

W = ½ (Ec2/C)
= ½ (Ec2/ C)
REGISTERED ELECTRICAL ENGINEERS PRE-BOARD EXAMINATION
AUGUST 2016
PROFESSIONAL ELECTRICAL ENGINEERING SUBJECTS

= ½ (78.692/ 40uF)

Answer: B.0.124J

94. Calculate the average voltage generated in a six-turn full-pitch coil of a 25-
cycle alternator if the flux pole is 7.2 x 10^5 maxwells.
A. 7.4 V B. 8.64 V C. 1.08 V D. 4.32 V

SOLUTION:

F= 25Hz

= 7.2 x 10^5 maxwells

N= 6

Eave = 4 f N x 10^-8

= (25) (7.2x 10^5) (6) x 10^-8

Eave= 4.32 V

Answer: D. 4.32 V

95. A 13.8 kV/440 v, 50 kVA single-phase transformer has a leakage


reactance of 300 ohms referred to the 13.8 kV side. Determine the per unit
value of the leakage reactance of the low base.
A. 0.305 B. 0.40 C. 0.097 D. 0.079

SOLUTION:
S= V2/R
= (13800)2/300
= 634800
Per unit= 50000/ 634800
= 0.079
Answer: D. 0.079

96. A 100 km transmission line has a 1,000 ohms shunt reactance. What is
the per km shunt reactance?
A 1,000 Ω B , Ω C 10 Ω D 100 Ω
REGISTERED ELECTRICAL ENGINEERS PRE-BOARD EXAMINATION
AUGUST 2016
PROFESSIONAL ELECTRICAL ENGINEERING SUBJECTS

SOLUTION:

100 km,

R = 1,000Ω

Rsh = 1000Ω/km 100km

Rsh = 100,000 Ω

Answer: B. , Ω

97. A 5,000 kVA synchronous condenser operates with a leading p.f. of


0.032. The losses are 180 kW. What is the power input to the motor?
A. 5,160 kW B. 5,320 kW C. 160 kW D. 320 kW

SOLUTION:

Cos ϴ= P/ S
0.032= P/ 5000000
P= 160 KW
Answer: C. 160KW

98. The current flowing in the armature conductors of a dc motor is


A. ac B. dc C. ac as well as dc D. Transients

99. The sequence components of phase c current of a 3-phase system are as


follows: Ic0=0, Ic1=18.4 cis88.4, Ic2=3.23 cis48.2. Find the phase b
current in amperes.
A. 19.24 cis -218 B. 18 cis 42 C. 18 cid -42 D. 19.24 cis 218

SOLUTION:

Ia1= (18.4<88.40) (1<120) = 18.4<- 151.6


Ia2= (3.23<48.2) (1<120) = 3.23<168.2
Ib= (18.4<- 151.6)(1<120)+ (3.23<168.2)(1<240)
= 19.23<-22 (1<240)
= 19.23<218
Answer: D. 19.24 cis 218

100. A current of 20 A is flowing i =n an RL series circuit the instant before the


switch is opened. If R = 10 Ω and L= 10 H, find the current after 3 second
has elapsed.
REGISTERED ELECTRICAL ENGINEERS PRE-BOARD EXAMINATION
AUGUST 2016
PROFESSIONAL ELECTRICAL ENGINEERING SUBJECTS

A. 1 A B. 2 A C. 3 A D. 4 A

SOLUTION

I= (E/R)(1- e –R/L(t))
= (200/10)(1-e-10/10(3))
= 19A
I= 20-19
= 1A
Answer: A. 1A
MARCH 2017
REGISTERED ELECTRICAL ENGINEERS PRE-BOARD EXAMINATION
MARCH 2017
PROFESSIONAL ELECTRICAL ENGINEERING SUBJECTS

1. A 4 pole lap winding motor with 16 slots, 42 conductors per slot, 480 v runs at 1800
rpm. Find the flux produced per pole.

A. 11.9 mWb B. 47.6 mWb C. 35.7 mWb D. 23.8 mWb

SOLUTION:
=
60
= . 𝒃

2. Compute for the core loss if the transformer consumes 75 watts when it is connected
across a 240 v source and taking 1.4 A. The primary resistance is 0.25 Ω

A. 73.6 W B. 74.65 W C. 74.75 W D. 74.51 W

SOLUTION:

2
= = 1.42(0.25) = 0.49 = 75 − 0 49 = .

3. What is the total power consumed by the load when the readings of two
wattmeters are 5 kW and -0.5kW?

A. 5.5 kW B. 4.5 kW C. 5 kW D. 4.05 kW

SOLUTION:

W1 = 5kW , W 2 = -0.5kW

PT = W 1 + W 2
=5 −05 = .

4. Find the reactive factor of the load having 30 kW and 50 kVA.

A. 0.6 B. 0.5 C. 0.8 D. 0.7


REGISTERED ELECTRICAL ENGINEERS PRE-BOARD EXAMINATION
MARCH 2017
PROFESSIONAL ELECTRICAL ENGINEERING SUBJECTS

SOLUTION:

Q2 = S 2 – P 2 rf = Q = 40 = 0.8

Q2 = 502 - 302 S 50

Q = 40kVAR

5 Find the total resistance if 3Ω is connected in series with the parallel combination of
4Ω, 5Ω and 20Ω
A 4Ω B. Ω C 6Ω D 7Ω

SOLUTION:

RT = (R1-1 + R2-1 + R3-1)-1 + R4


= (4-1 + 5-1 + 20-1)-1 + 3 =

6. Given a 100 kVA 2,400/240 v single phase transformer. Find the current
transformer (CT) ratio at the low voltage side.

A. 700:5 B. 600:5 C. 500:5 D. 400:5

SOLUTION:

CTR = Ip Is = S / V Ans. 500:5

Is = 100x103 / 240 = 416.67 A

7. One KWHR is equal to __________ BTU.

A. 360 B. 3,413 C. 3,600 D. 1,000


SOLUTION:
1kWHR = 3413 BTU = 3600 kJ

8. Convert the delta connected impedance of 12 36 Ω to balanced wye connected


impedance.
REGISTERED ELECTRICAL ENGINEERS PRE-BOARD EXAMINATION
MARCH 2017
PROFESSIONAL ELECTRICAL ENGINEERING SUBJECTS

A. 3 36° B. 6 36° C. 4 36° D. 12 36°

SOLUTION:

Zwye = Zdelta / 3 = 12cis36 / 3 = 4cis36

9. Heating effect of current has an undesirable side effect in


A. Electric oven B. immersion heater C. electric iron D. vacuum cleaner

10. The primary of transformer has 200 turns and is excited by a 240 v, 60 Hz source.
What is the maximum value of the core flux?

A. 4.04 mWb B. 4.4 mWb C. 4.13 mWb D. 4.32 mWb

SOLUTION:

= 4.44 240 = 4.44(60)(200) = . 𝒃

11. When the sole purpose of an alternating current is to produce heat, the selection of
conductor is based on _______ value of the current.

A. average B. instantaneous

C. root mean square D.peak

12. What is the primary advantage of FM over AM?

A. FM has a greater bandwidth than AM B. noise immunity

C. more complex D. greater capture effect

13. A Walkman has 4 – AA batteries. The combination has a capacity of 200 watt-sec.
How long will it take to discharge if Walkman consumes 10 mA at 6 v?

A. 55.6 min B.65.6 min C. 45.6 min D. 75.6 min


REGISTERED ELECTRICAL ENGINEERS PRE-BOARD EXAMINATION
MARCH 2017
PROFESSIONAL ELECTRICAL ENGINEERING SUBJECTS

SOLUTION:

= = (10 × 10−3)(6) = 0.06 =

= = 200 = 3333.33 × 1
= .
0.06 60

14 What is the total resistance of eleven 33 kΩ resistors connected in parallel?

A 33 kΩ B. Ω C 363 kΩ D 11 kΩ

SOLUTION:
33
= = =
11

15. Which of the following is not included in the evaluation of the Technical Performance
of a Distribution System according to Philippine Distribution Code (PDC)?

A. Power Quality B. Reliability

C. Collecting Efficiency D. System Efficiency

16. The phase angle for a series RC circuit may be computed as the angle between the
__________ and __________ phasors.

A. resistance, impedance C. resistance, capacitance

B. resistance, reactance D. impedance, reactance


17. What is the formula for the synchronous speed of a motor?

A. NN = 120P B. Ns = 120fP
f
C. Ns = 120f D. Ns = 60f
P P
18. Which is not a characteristic of an ideal transformer?
REGISTERED ELECTRICAL ENGINEERS PRE-BOARD EXAMINATION
MARCH 2017
PROFESSIONAL ELECTRICAL ENGINEERING SUBJECTS

A. finite permeability B. infinite permeability


C. no losses D. no leakage flux

19.How many 240 v, 20 A branch circuits are required to supply 60,000 va load?

A. 13 B. 14 C. 15 D. 12

SOLUTION:

240 × 20 = 4800

20. Two capacitors in series are changed across 24 v source. If C 1 is 12 picofarads, and
the voltage across it is 6 v, find C2.

A. 6 pF B. 8 pF C. 2 pF D. 4 pF

Solution:

1 = 𝐶 1 +𝐶𝐶22 𝐶2 = 1−𝐶 11 = 6 1224×10−6−12 =

21. It is defined as the total number of sustained Customer power interruption within a
given period divided by the total number of Customers served within the same period.
A. System Average Interruption Frequency Index
B. System Average Interruption Duration Index
C. Customer Average Interruption Frequency Index
D. Customer Average Interruption Duration Index

22. Calculate for the dc voltage when the converter consumes 1.5 kW of power at a
source voltage of 230 v ac.
A. 253 v B. 352 v C. 235 v D. 325 v
SOLUTION:
Vdc = Vac(sqrt. of 2)
= 230(sqrt. of 2)
= 325 v

23. A 115 kV three-phase transposed transmission line is composed of 2-ACSR 336,


400 cmil, 30/7 Oriole conductor per phase with horizontal configuration: D 12 = 7 ft, D23 =
REGISTERED ELECTRICAL ENGINEERS PRE-BOARD EXAMINATION
MARCH 2017
PROFESSIONAL ELECTRICAL ENGINEERING SUBJECTS

7 ft, D13 =14 ft. The line spacing is measured from the center of the bundled conductors.
The distance between the conductors of the bundle is 6 inches. The conductors have a
diameter of 0.741 inch and a GMR of 0.02255 ft. Find the capacitance per phase for 20
km of the line in microfarads.
A. 0.1300 B. 0.2719 C. 0.2889 D. 0.2609

24. The specific resistance ρ is defined as


A. resistance of a conductor which has a length of 1 m and cross-section of
1 m2 at 20°C
B. resistance of any conductor at 25°C
C .resistance of any conductor at 20°C
D. resistance of a conductor which has a length of 1 m and cross-section of 1
cm2 at 20°C

25. The impedance triangle is similar to the __________ triangle with the resistance
phasor in place of the __________.
A. Current, resistor current C. voltage, impedance
B. Current, resistor voltage D. voltage, resistor voltage

26. What is the IEEE device function number 50?


A. AC time overcurrent relay C. AC circuit breaker
B. instantaneous overcurrent relay D. Differential protective relay

27. In long transmission line, at no-load the receiving end voltage as ccompared to the
sending end voltage is
I. higher II. lower III. Remains the same
A. I only B. II only
C. Either I or II D. III only

28 How many 100 kΩ will be connected in parallel to obtain 20 kΩ


A. 8 B. 6 C. 5 D. 4
SOLUTION:
Rt = R/n ,
n = R/Rt , n = (100k)/(20k)
n=5

29. What determines the direction of force in a conductor?


A. Lenz law B Faraday’s law
C. Left hand rule D. Right hand rule
REGISTERED ELECTRICAL ENGINEERS PRE-BOARD EXAMINATION
MARCH 2017
PROFESSIONAL ELECTRICAL ENGINEERING SUBJECTS

30. Which of the following plants will take least time in starting from cold condition to full
load operation?
A. gas turbine C. nuclear power plant
B. steam power plant D. hydroelectric power plant

31. The study of illumination involves

A. design of lighting system


B. study of lighting and wiring system
C. calculation of wiring sizes for various lighting fixtures
D. study of photometry and its application

32. When a power plant is not able to meet the demand of consumers it will resort to

A. load shedding
B. efficiency plant operation
C. power factor improvement at the generators
D. penalizing high load consumers by increasing the charges of electricity

33. A distributor is designed from the point of view of

A. voltage drop C. voltage drop and current


carrying capacity
B. current carrying capacity D. load capacity

34. The annual load curve is a straight line from 40 MW to 8 MW. What is the annual
load factor?

A. 0.8 B. 0.7 C. 0.6 D. 0.9

35. Conservator of a transformer consists of

A. an air-tight metal drum fixed at the top of the tank


B. drum placed at the bottom of the tank
C. overload protection
D. none of these

36. Cellular phones operate in the frequency range of


REGISTERED ELECTRICAL ENGINEERS PRE-BOARD EXAMINATION
MARCH 2017
PROFESSIONAL ELECTRICAL ENGINEERING SUBJECTS

A. EHF B. VHF C. UHF D. SHF

37.Varmeter when paired with other devices measures __________ factor?

A. diversity B. power C. load D. demand

38. The type of ac distribution system commonly used to supply both light and power is
the
A. open delta system C. three phase wye system with neutral wire
B. three phase delta system D. three phase wye system without neutral wire

39.A 40μF capacitor is supplied with a voltage of v t = 100 sin 377t What is the current
at t=0?
A. 0 B. 1.51 A C. 3.02 A D. 1.07 A

SOLUTION:
𝑿 = /𝒘 = /( )( µ )= . Ω
= /𝑿 = / . = .

40.If an electrical network having one or more than one voltage source is transformed
into equivalent electrical network with a single current source (which is the circuit
current of previous circuit) with parallel internal resistance of the network with all current
source replaced by their internal resistance; the above illustration is called
A. Thevenin’s theorem C. Reciprocity theorem
B. o ton’s th o m D. Superposition theorem

41.Which is the most reliable power plant?


A. Steam B. Hydro C. Wind D. Tidal

42.What governs the minimum safety requirement in the Philippines for distribution and
grid?
A. Philippine Electrical Safety Code C. Philippine Grid Code
B. Philippine Electrical Code D. Philippine Distribution Code
REGISTERED ELECTRICAL ENGINEERS PRE-BOARD EXAMINATION
MARCH 2017
PROFESSIONAL ELECTRICAL ENGINEERING SUBJECTS

43.In a diode modulator, what supplies the negative halfwave?


A. Inductor B. Capacitor C. Resistor D. Tuned

Circuit

44.Which is not a source of power?


A. Thermocouple B. Fuel coil C. Photovoltaic cell D. Solar cell

45. When the frequency of a synchronous motor increases, what happens to its speed?
A. decrease B. increase C. remains the same D. none of these

46. A copper wire is drawn down such that its length is doubled but the volume remains the same,
what is the new resistance of the wire?
A. R1 B. R1/2 C. 2R1 D. 4R1

SOLUTION:

=ƿ / = , =
= ƿ ) )/ ƿ ( )/
= ƿ )( )/
= ƿ )( )( /ƿ ^
=

47.What instrument measures the insulation of a powerline?


A. growler B. ohmmeter C. galvanometer D. megger

48.What is the frequency where the reactances of 57 μH inductor and a 57 μF capacitor


are equal?
A. 7.92 kHz B. 9.72 kHz C. 2.79 kHz D. 2.97 kHz

SOLUTION:
𝑿 𝑿
𝝅 / 𝝅
^ / 𝝅^

√ (𝝅 ) ( ) )
REGISTERED ELECTRICAL ENGINEERS PRE-BOARD EXAMINATION
MARCH 2017
PROFESSIONAL ELECTRICAL ENGINEERING SUBJECTS

(
.

49 A resistance of 30 Ω and a capacitance of 132 6 μF are connected in series across


a 220 v, variable frequency ac supply. At what frequency in Hz will the current be 4.4 A?

A. 30 Hz B. 40 Hz C. 50 Hz D. 60

SOLUTION:

𝒁
𝑿

𝑿 / 𝝅

= 𝝅( )( . − )
=

50. At steady state in an RC circuit, the capacitance will act as __________.

A. open circuit B. short circuit C. transient circuit D.


coupled circuit

51. A certain amount of fuel contains 15 x 10 10 BTU of energy. What is the

corresponding energy in kilocalories?

A. 3.78 x 10 13 kcal B. 3.78 x 10 12 kcal

C. 3.78 x 10 11 kcal D. 3.78 x 10 10 kcal

SOLUTION:

15×10^10 btu × .252kcal = 3.78×10^10kcal

1btu
REGISTERED ELECTRICAL ENGINEERS PRE-BOARD EXAMINATION
MARCH 2017
PROFESSIONAL ELECTRICAL ENGINEERING SUBJECTS

52. In a 3-phase, 4-wire system the current in the a, b, and c lines under

abnormal condition of loading are, I a = 100 30° A, I b = 50 300° A and I c = 30

180° A. Find the zero sequence currents.

A. 18.3 6.81° A B. 17.3 3.15° A

C. 6.71 5.4° A D. 27.29 4.69° A

SOLUTION:

Ia=100 &lt;30° Iao=1/3 [ (100 &lt;30°)+(50 &lt;300°)+(30 &lt;180°)]

Ib=50 &lt; 300° Iao= 27.29 4.69° A

Ic= 30 &lt;180°

53. A transmitter supplies 10 kW of carrier power to the antenna. The total

radiated power with 40% modulation is __________ kW.

A. 14 B. 10.8 C. 1.6 D. 25

SOLUTION:

Pt= Pc(1+m^2/2)

Pt= 10 (1+4^2/2) =10.8

54. In a balanced 3-phase voltage source with phase sequence abc, phase A

voltage is 127 cis 0° volts. Find the line C voltage.

A. 127 -240° v B. 220 -150° v

C. 220 -120° v D. 127 -60° v

SOLUTION:

Va=127 angle 0° =Va angle;0°


REGISTERED ELECTRICAL ENGINEERS PRE-BOARD EXAMINATION
MARCH 2017
PROFESSIONAL ELECTRICAL ENGINEERING SUBJECTS

Va=127 angle -120° =Vb angle -120°

Vc=127 angle 240° =Va angle -240

55. A 6-pole lap wound dc generator has 240 armature conductors, a flux of 0.04

Wb per pole and runs at 1,500 rpm. If the armature resistance is 0 8 Ω, determine the
current through each conductor.

A. 300 A B. 100 A C. 50 A D. 25 A

SOLUTION:

N=6 Pole Z=240

N=1500 Ra=.8

𝒁ф
𝑬=

( )(. )( )
𝑬=
( )( )
𝑬=

56. An air cored toroidal coil has 450 turns and a mean diameter of 300 mm and

a cross-sectional area of 300 mm 2 . Determine the self-inductance of the coil.

A. 81 x 10 -4 H B. 8.1 x 10 -4 H

C. 0.81 x 10 -4 H D. 0.081 x 10 -4 H

SOLUTION:

N=450 turns A=300×(.001m/1mm)^2= 3×10^-4


REGISTERED ELECTRICAL ENGINEERS PRE-BOARD EXAMINATION
MARCH 2017
PROFESSIONAL ELECTRICAL ENGINEERING SUBJECTS

d=300 mm L= (3×10^-4)(450)^2(4×3.14×10^-7)(1)

A=300mm^2 300×10^-3 (3.14)

L= 0.81 x 10 -4 H

57. A 4-pole, three-phase 50 Hz induction motor develops a maximum torque of

20 N-m at 1,440 rpm. Obtain the torque exerted by the motor at 5% slip.

A. 16.51 N-m B. 17.51 N-m C. 18.51 N-m D. 19.51 N-m

58. A transformer has negligible resistance and a p.u. reactance of 0.1. What is

its voltage regulation on full load with a leading p.f. angle of 30°?

A.-10% B. 10% C. 5% D. -5%

59. In an ac circuit, a low value of KVAR compared with KW indicates

A. maximum load current B. low efficiency

C. high power factor D. unity power factor

60. The frequency of the emf in the stator of a 4-poleinduction motor is 60 Hz,

and that in the rotor is 2 Hz. At what speed is the motor running?

A. 1,720 RPM B. 1,740 RPM C. 1,760 RPM D. 1,780 RPM

SOLUTION:

= / = ( )/ =
REGISTERED ELECTRICAL ENGINEERS PRE-BOARD EXAMINATION
MARCH 2017
PROFESSIONAL ELECTRICAL ENGINEERING SUBJECTS

= ( )/ = =
= =

61 The voltage across a 50 μF capacitor rises at a constant rate of 10 v/ms Calculate

the current through the capacitor.

A. 50 mA B. 500 mA C. 5 mA D. 5 A

SOLUTION:

I= CE

= (50x10-6)(10)(1/1x10-3)

I= 0.5A or 500mA

62. Branch circuit larger than __________ amperes shall only supply non-

lighting outlet load.

A. 15 B. 20 C. 30 D. 50

63.In a 4-pole dynamo, the flux per pole is 15 mWb. Calculate the maximum emf

induced in one of the armature conductors if armature is driven at 600 rpm.

A. 0.6 v B. 0.3 v C. 0.94 v D. 0.47 v

SOLUTION:

E= 𝒁ф

= ( ) (
REGISTERED ELECTRICAL ENGINEERS PRE-BOARD EXAMINATION
MARCH 2017
PROFESSIONAL ELECTRICAL ENGINEERING SUBJECTS

. .

64. How much larger is the rotor reactance of a SCIM at start-up than it is when the

motor operates at 5 percent slip?

A. 20 times B. 10 times C. 5 times D. 25 times

65. The velocity of electromagnetic waves in free space is given by

A. B. C. 1/ 𝜺 D. √

66. A 220 v, 3-phase, 4-wire abc system supplies unbalanced wye-connected

resistance of 10 Ω, 15 Ω, and 20 Ω Determine the total power of the system in

watts.

A. 3,895 B. 3,560 C. 3,385 D. 3,495

Solution:

=

)
. 𝒘
REGISTERED ELECTRICAL ENGINEERS PRE-BOARD EXAMINATION
MARCH 2017
PROFESSIONAL ELECTRICAL ENGINEERING SUBJECTS

67. Assuming a certain driving voltage causes the current to an initially discharged

1,000 μF capacitor to increase at a constant rate of 0 06 A/s , determine the voltage

across the capacity at t = 10 seconds.

A. 600 v B. 1,500 v C. 2,000 v D. 3,000 v

SOLUTION:
E = Q/C
Q = It
E= 0.06C
=(0.06A/s)(10s) 1000x10-6F
= .06 C E=600
68.The illumination on a surface is 100 lm/m 2. The reflection coefficient of the surface is

80%. What is the maximum brightness of the surface in Cd/m2?

A. 80 B. 125 C. 100 D. 64

SOLUTION:

1lm/m2 = 1cd

0.80x100lm/m2=80lm/m2=80cd/m2

69. Wye-delta starting is equivalent to autotransformer starting with __________

tapping.

A. 33% B. 58% C. 85% D. 52%

70. With reference to armature windings, wave windings are often called __________

windings.

A. cascade B. ring C. multiple or parallel D. series

70. With reference to armature windings, wave windings are often called

__________ windings.

A. cascade B. ring C. multiple or parallel D. series

71. Induced charge on insulators

A. resides on surface C. changes into electric current

B. moves inside D. moves outward


REGISTERED ELECTRICAL ENGINEERS PRE-BOARD EXAMINATION
MARCH 2017
PROFESSIONAL ELECTRICAL ENGINEERING SUBJECTS

72. Human body is

A. Insulator B. low resistive C. conductor D. electrolyte

73. The presence of an earth in case of overhead lines

A. increases the capacitance C. increases the inductance

B. decreases the capacitance D. decreases the inductance

74. A string insulator has 6 units. The voltage across the bottom most unit is 20%

of the total voltage. Its string efficiency is

A. 20% B. 60% C. 75% D. 83.3%

SOLUTION:

String Efficiency =Operating V(phase)/(nXVoltage nearest to the conductor) x100

String Efficiency = Vph / (6 x .20Vph) x 100 = 83.3%

75. The per unit value of 2 ohm resistor at 100 MVA base and 10 KV base

voltage is

A. 4 pu B. 2 pu C. 0.5 pu D. 0.2 pu

SOLUTION:

Zpu = Zold (Sbase) / (Vbase)^2

Zpu = 2 (100000000) / (10000)^2

Zpu = 2 pu

76. A 75 MW power plant has an average load of 35,000 KW and a load factor of

65%. Find the reserve over peak.

A. 23.41 MW B. 21.15 MW C. 25.38 MW D. 18.75 MW


REGISTERED ELECTRICAL ENGINEERS PRE-BOARD EXAMINATION
MARCH 2017
PROFESSIONAL ELECTRICAL ENGINEERING SUBJECTS

SOLUTION:

LF= Average/Peak

Peak= Average/Load Factor

Peak= 35x10^6/0.65 = 53.846x10^6

Reserve OverPeak= Orig.-Peak = (75x10^)(53.846x10^6) = 21.153x10^6

77. Transformer breaths in when

A. load on it increases C. load remains constant

B. load on it decreases D. none of these

78. A hydroelectric generating station is supplied from a reservoir of capacity

6,000,000 m 3 at a head of 170 m. Assume hydraulic efficiency of 80% and electrical


efficiency of 90%. The fall in the reservoir level after a load of 15 MW

has been supplied for 3 hours, if the area of the reservoir is 2.5 km 2 is closest to

A. 5.39 cm B. 4.32 cm C. 5.98 cm D. 4.83 cm

79. The active length of each conductor placed in a DC generator armature is 0.4

m. The armature is related at a speed of 1,800 rpm in a magnetic field of average

flux density 1.2 T. The diameter of the armature is 0.5 m. Find the emf induced in

the conductor when it cuts the magnetic field at right angles.

A. 5.65 v B. 11.31 v C. 45.24 v D. 22.62 v

SOLUTION :

e = βlv sin theta = βlv


REGISTERED ELECTRICAL ENGINEERS PRE-BOARD EXAMINATION
MARCH 2017
PROFESSIONAL ELECTRICAL ENGINEERING SUBJECTS

v = πDN/60 = π 0 5 1800 /60 = 47 12 m/s

e = βlv = 1 2 0 4 47 12 = 22 62 V

80. The flux produced in the air gap between two electromagnetic pole faces 6 x

10 -2 Wb. Length of air gap is 1.4 cm and cross-sectional area of the gap is 0.3 m 2 .

Find the magnetic field intensity.

A. 153.2 x 10 3 AT/m B. 155. 2 x 10 3 AT/m

C. 157.2 x 10 3 AT/m D. 159.2 x 10 3 AT/m

81. What is the empirical formula to calculate the number of strands?

A. 3n(n + 2n) + 2 B. 3n(n + 1) + 2 C. 3n(n + 1) + 1 D. 3n(3 + 3n) + 1

82. The shaft of synchronous motor is made of

A. mild steel B. chrome steel C. alnico D. stainless steel

83. For a single line to ground fault, the zero sequence current is given by j3 pu. The
current carried by the neutral during the fault is

A. j1 pu B. j3 pu C. j9 pu D. j6 pu

SOLUTION:

𝒁 𝒁

84. A moving coil instrument requires 50 mA and 100 mV for full-scale deflection.
What is the value of the shunt required to convert it to read up to 5 A?
A 0 0101 Ω B. 2 2Ω C 0 0303 Ω D 0 0404 Ω
REGISTERED ELECTRICAL ENGINEERS PRE-BOARD EXAMINATION
MARCH 2017
PROFESSIONAL ELECTRICAL ENGINEERING SUBJECTS

SOLUTION:
( )( )

85.The resistance of a winding wire made up to copper is 25 ohms at 30°C. What is its
resistance when it operates at a temperature of 75°C? Assume temperature coefficient
at 20°C equals 0.004/°C.
A 31 36 Ω B 24 04 Ω C 27 45 Ω D. 2 Ω

SOLUTION:

( *
86. Bundled conductors are used in transmission lines, the effective capacitance and
inductance will respectively.

A. decrease and increase C. remain same and increase


B. increase and decrease D. decrease and remain same

87. A 40 m metallic conductor of cross-sectional area 1 mm2 has a resistance of 12 Ω


Calculate the conductivity of the metal.

A. 3.33 KS/m B. 33.3 MS/m C. 33.3 KS/m D. 3.33 MS/m

SOLUTION:

( )( )( )

88. A car battery supplies 48 J of energy at 12 v over a certain period of time.


Determine the charge moved during this period.

A. 0.33 C B. 0.25 C C. 4 C D. 2 C

SOLUTION:
𝑬
𝑬
REGISTERED ELECTRICAL ENGINEERS PRE-BOARD EXAMINATION
MARCH 2017
PROFESSIONAL ELECTRICAL ENGINEERING SUBJECTS

89. An inductive coil consumes 500 W of power at 10 A and 110 v and 60 Hz.
Determine the inductance of the coil.

A. 24 mH B. 25 mH C. 26 mH D. 27 mH

SOLUTION:


𝑿 √𝒁 √ ( )( )( )

90. Two coupled coil have self-inductances L1 = 50 mH and L2 = 200 mH, and a
coefficient of coupling K = 0.50. If I1 = 5 sin 400t A, find the voltage at coil 2.

A. 100 cos 400t B. 100 sin 400t C. 400 cos 400t D. 400 sin 400t

SOLUTION:

√ √

( )

91. In a three-phase system, when the loads are perfectly balanced, the neutral

current is

A. zero C. two-thirds of maximum

B. one-third of maximum D. at maximum

92 A parallel RLC circuit has R = 0 5 Ω, L = 10 mH and C = 2.5 mF. Determine

the resonance frequency

A. 63.66 Hz B. 79.79 Hz C. 112.82 Hz D. 31.83 Hz

SOLUTION:

Fr= (1/2pi)(sqrt. (1/LC)-(R^2/L^2))


REGISTERED ELECTRICAL ENGINEERS PRE-BOARD EXAMINATION
MARCH 2017
PROFESSIONAL ELECTRICAL ENGINEERING SUBJECTS

Fr= (1/2pi)(sqrt. (1/(10x10^3)(2.5x10^3))-(0.5^2/10x10^2)) = 31.83HZ

93. A double squirrel-cage induction motor has

A. two rotors moving in opposite direction

B. two parallel windings in stator

C. two parallel windings in rotor

D. two series windings in stator

94. A balanced delta connected load draws 10 A of line current and 3 KW at 220

v. Determine the value of resistance of each phase of the load.

A 38 1 Ω B Ω C 23 5 Ω D 33 3 Ω

SOLUTION:

R= P/ I^2= 3000W/ 10^2= 30 OHM

95. Three sinusoidal generators and a battery are connected in series with a coil

whose resistance and inductance are 8 Ω and 26 53 mH, respectively The

frequency and rms voltage of the respective generators are 20 Hz, 15 v, 60 Hz,

30 v, 80 Hz, 50 v. The battery voltage is 6 v. Determine the circuit rms voltage.

A. 10 v B. 60.51 v C. 101 v D. 69.5 v

96. A 12 KVA load A operating at 0.70 p.f. lagging and a 10 KVA load B

operating at 0.80 p.f. lagging are connected in parallel and supplied by a 440 v,

60 Hz generator. Determine the system power factor.

A. 0.767 B. 0.727 C. 0.747 D. 0.787


REGISTERED ELECTRICAL ENGINEERS PRE-BOARD EXAMINATION
MARCH 2017
PROFESSIONAL ELECTRICAL ENGINEERING SUBJECTS

SOLUTION:

I= S/ V= 12000KVA /440V= 27.27angle 45.57

I= S/V = 10000KVA/ 440V= 22.73angle 36.87

It=I+ I= (27.27angle 45.57)+(22.73angle 36.87)= 49.86angle 41.618

arcos(41.617)= 0.747

97. Suppose that the consumption of energy in a certain country has a growth

rate of percent per year. In how many years will the energy consumption be

tripled?

A. 20.60 B. 13.73 C. 9.16 D. 27.47

98 A 345 KV transmission line has a series impedance 4 j60 Ω and a shunt

admittance of j2 x 10 -3 S. Using 100 MVA and the line voltage as base values,

calculate the per unit shunt admittance of the line.

A. j2.38 pu B. j3.57 pu C. j4.76 pu D. j5.95 pu

SOLUTION:

Z= 1/Y= 1/j2x10-3= -j500

Zpu= (j500x100x10^6)/(345x10^3)= j0.42

Ypu= 1/ Zpu= 1/j0.42= j2.38 pu

99. The purpose of providing dummy coils in a generator is

A. to reduce eddy current losses C. to amplify voltage

B. to enhance flux density

C. to amplify voltage
REGISTERED ELECTRICAL ENGINEERS PRE-BOARD EXAMINATION
MARCH 2017
PROFESSIONAL ELECTRICAL ENGINEERING SUBJECTS

D. to provide mechanical balance for the rotor

100. The conductors of a three-phase transmission line are arranged in the form

of an equilateral triangle with sides of 6 m each. If the conductors are 500 mils in

diameter and the line is 25 km long, what is its inductance per phase?

A. 40 mH B. 17.75 mH C. 26.62 mH D. 35.5 mH


AUGUST 2017
REGISTERED ELECTRICAL ENGINEERS PRE-BOARD EXAMINATION
AUGUST 2017
PROFESSIONAL ELECTRICAL ENGINEERING SUBJECTS

1. A conductor of length 1 meter moves at right angle to a magnetic field of flux density
1 Wb/m^2 with a velocity of 25m/s. What is the induced emf in the conductor?
Solution:
e= wsinᴓ

=25(1)(1)

=25V

Answer: A. 25v

2. It is difficult to magnetize steel because of its

Answer: C. Low Permiability

3. Present day capacitors which have high capacitance in small size use a dielectric of

Answer: C. Ceramic

4. The current-carrying capacity of a copper wire having twice the diameter of another

copper wire is ______________ as great.

Solution:

Therefore,

= =4

Answer: B. Four times

5.The output current corresponding to maximum efficiency for a transformer having

core loss of 100 watts and equivalent resistance referred to secondary of 0 25Ω is

________ amperes
REGISTERED ELECTRICAL ENGINEERS PRE-BOARD EXAMINATION
AUGUST 2017
PROFESSIONAL ELECTRICAL ENGINEERING SUBJECTS

Solution:

Pco = Pcu


100= I= = 20A

Answer: B.20

6. What kind of device is a diode?

Answer: D. Nonlinear

7. What is the value of inductance of a coil that induces 100v when the current changes

at the rate of 2mA in 2 microseconds?

Solution:

e= L

=L

L = 0.1H

Answer: D. 0.1H

8. In a generator, if the voltage regulation is 5.5% and the full-load voltage is 220 volts,

what is the no-load voltage equal to?

Solution:

= ( )

= ( )

= 232.1

Answer: A. 232.8v

9. If the line frequency is 60Hz, the output frequency of a bridge rectifier is


REGISTERED ELECTRICAL ENGINEERS PRE-BOARD EXAMINATION
AUGUST 2017
PROFESSIONAL ELECTRICAL ENGINEERING SUBJECTS

Answer: C. 120Hz

10. At what frequency will a 100µF capacitor offer a reactance of 100Ω

Solution:

C=

100 = ( )

= 15.9 Hz

Answer: B. 15.9 Hz

11. A 4-pole wave wound generator has 220 coils of 10 turns each. The resistance of

each turn is 0 02Ω find the resistance of armature winding

Solution:

= Ω
( )

Answer: A. Ω

12. A three-phase 4-pole 60Hz induction motor has a rotor resistance of 0 02Ω/phase

and stands still reactance 0f 0 5Ω/phase Calculate the speed at which the

maximum torque is developed.

Solution:

%slip= = = 0.004

Nr= Ns(1-%slip)

= (1-%slip)

( )
= (1-0.004)

Nr = 1728 RPM
REGISTERED ELECTRICAL ENGINEERS PRE-BOARD EXAMINATION
AUGUST 2017
PROFESSIONAL ELECTRICAL ENGINEERING SUBJECTS

Answer: D. 1728 rpm

13. In a shunt generator the voltage builds up till constrained by

Answer: C. Saturation of iron

14. Slip rings are made of

Answer: B. copper

15. How much energy is stored by a 100mH inductance with a current of 1A?.

Solution:

W= ½ (100 x )(1)

W = 0.05J

Answer: B. 0.05J

16. For proper parallel operation, ac polyphaser alternators must have the same______

Answer: C. Voltage Rating

17.Suppose that the consumption of energy in a certain country has a growth rate of

4% per year. In how many years will the energy consumption be tripled?

Solution:

Q = Qo

3Q = Qo

ln 3 = ln

t= 29.47

Answer: B. 29.47

18. The capacitance between any two conductors of a three-conductor cable, with the

third conductor grounded is 0.6µF/km. Calculate the line to ground capacitance of a


REGISTERED ELECTRICAL ENGINEERS PRE-BOARD EXAMINATION
AUGUST 2017
PROFESSIONAL ELECTRICAL ENGINEERING SUBJECTS

25-km length of the cable.

Solution:

C= (0.6 ) (25km) = 15

CT = 2 (15 )

CT = 30

Answer: A. 30µF

19. A transmitter supplies 10kW of carrier power to the antenna. The total radiated

power with 40% modulation is ________ kW

Solution:

Pr= Pc (1 + )

= 10 (1+ )

Pr = 10.8 kW

Answer: D. 10.8

20. A 3-phase, 4-pole induction motor with the open circuited rotor is connected to a

400-v, 50Hz, 3-phase supply. The motor will run at _________rpm.

Answer: D. 0

21. A lamp has a luminous efficiency of 11 lm/watt and gives out a luminous flux of 275

lm. What is the wattage of the lamp?

Solution:

P=

P = 25 W

Answer: B. 25

22. The series field of a short-shunt compound dc generator is excited by ______


REGISTERED ELECTRICAL ENGINEERS PRE-BOARD EXAMINATION
AUGUST 2017
PROFESSIONAL ELECTRICAL ENGINEERING SUBJECTS

current.

Answer: D. External

23.If copper loss of a transformer at 7/8th of full load is 4900W, then what is its full-load

copper loss?

Solution:

= size x

=( )

P =6400W

Answer: B. 6400 W

24.The Reciprocal of permeability is

Answer: C. reluctivity

25. For which of the following sinusoids, the rms value and mean value is the same?

Answer: C. Square Wave

26.A 440-v; three-phase source supplies a wye connected 10KVA load at 0.8 lagging

power factor and a delta-connected 10KVA unity power factor load. Calculate the

total apparent power input to the two loads.

Solution:

= 10 cis – (0.8) +10

= 18.97KVA

Answer: D. 18.97KVA

27. The maximum efficiency of a half –wave rectifier is


REGISTERED ELECTRICAL ENGINEERS PRE-BOARD EXAMINATION
AUGUST 2017
PROFESSIONAL ELECTRICAL ENGINEERING SUBJECTS

Solution:

=( )x 100

 = 40.5%

Answer: A. 40.6%

28. A resistance of 30 ohms and a capacitance of 132.6µF are connected in series

across a 220v variable frequency ac supply. At what frequency in HZ will the current

be 4.4A?

Solution:

Z=

= = 50

= +

= (√ )

= 40 =

= ( )(

f = 30 Hz

Answer: D. 30 Hz

29.A balanced wye-connected load with resistance of 8ohms/phase and no reactance

is supplied by a 240v, 3-phase source. Find the real power of the load in watts.

Solution:

V= IR

I=30

P=
REGISTERED ELECTRICAL ENGINEERS PRE-BOARD EXAMINATION
AUGUST 2017
PROFESSIONAL ELECTRICAL ENGINEERING SUBJECTS

= ( )

P= 7200 W

Answer: B. 7200

30. Given three unbalanced line currents Ia=0+j12A, Ib=5-j10A and Ic=-10-j5A. Find the

negative sequence component of Ic in amperes.

Solution:

= [ + +a ]

= 2.25 cis 105.70

= 2.25cis 105.70 (1cis240)

= 2.25 cis -14.3

Answer: C. 2.25cis-14.3deg

31.A 40ohm resistor and a pure capacitor are connected in parallel across a 230v,

60Hz ac supply. The total current taken from the supply is 8A. Find the capacitance

of the capacitor in µF.

Solution:

Z = V/I = 230/8 = 28.75


( )
Z= 28.75 =

Trial and error substitute the given to find Z = 28.75

Therefore, 64.15µF

Answer: A. 64.15µF

32. Two electrons in a vacuum experience a force of 2 x 10^-15 N. How far apart are

the electrons?

Solution:
REGISTERED ELECTRICAL ENGINEERS PRE-BOARD EXAMINATION
AUGUST 2017
PROFESSIONAL ELECTRICAL ENGINEERING SUBJECTS

P= kQ1Q2 / d^2

= =

K=

√( )( )
d=

d= 3.4 x 10^-7 m

Answer: A. 3.4 x 10^-7m

33.A 120 volts battery having an internal resistance of 0 5Ω is connected through line

resistance of 9 5Ω to a variable load resistor What maximum power will the battery

deliver to the load resistor?

Solution:

P=

= ( )

P = 360W

Answer: D. 360 W

34. Three resistors of 10Ω, 15Ω and 20Ω are connected in parallel What is the total

resistance?

Solution:

= [ + + ] ^ -1

4 2Ω

Answer: B. 4 2Ω

35. 10^13 ergs/sec is equal to how many KW?


REGISTERED ELECTRICAL ENGINEERS PRE-BOARD EXAMINATION
AUGUST 2017
PROFESSIONAL ELECTRICAL ENGINEERING SUBJECTS

Solution:

=1000KW

Answer: A. 1000 KW

36. Ideal transformer assumptions do not include

Answer: D. zero reactance of the winding

37.A dc ammeter has an internal resistance of 0 1Ω A shunt 1 01 x 10^-3Ω is

connected to the ammeter. What is the multiplier of the set up?

Solution:

R = 0 1Ω

R= = 100 Ω

Answer: C. 100

38. Two conductors carrying 50 amperes and 75 amperes respectively are placed 10cm

apart. Calculate the force between them per meter

Solution:

F= ( )( )

F=( )( )( )

F=

Answer: C. 0.0075N/m

39. A three-phase motor is rated 50HP, 440 volts and 85% power factor. What is its

rated current?

Solution:
REGISTERED ELECTRICAL ENGINEERS PRE-BOARD EXAMINATION
AUGUST 2017
PROFESSIONAL ELECTRICAL ENGINEERING SUBJECTS

P= √3 VL IL pf

I= ; P= 50HP x

=
√ ( )( )

I = 57.5A

Answer: B. 57.5A

40.A 100-km transmission line has 1200Ω shunt reactance What is the per km shunt

reactance?

Solution:

Unit Analysis

= (1,200)(100)

= 120,000

Answer: B. 120, Ω

41. Two 500-watt lamps connected in series across a 220-volt line draw 1A. What is the

total power consumed?

Solution:

P= IV

= 1 (220)

P = 220w

Answer: B. 220w

42. A switch is connected across a 220v supply. What is the voltage across the switch

if it is opened?

Answer: C. 220v
REGISTERED ELECTRICAL ENGINEERS PRE-BOARD EXAMINATION
AUGUST 2017
PROFESSIONAL ELECTRICAL ENGINEERING SUBJECTS

43. Calculate the AT/m to produce a flux density of 0.001 Wb/m^2 at the center of a

long straight solenoid.

Solution:

µ = 4pi x

β = µH

H= = 795.77 796

Answer: D. 796
44. A series RLC circuit consists of 20 ohms resistance ,0.2H inductance and an

unknown capacitance if the circuit has a leading angle of 45% at 60Hz/

Solution:

tanᴓ =

20=

= 2 fL – 20 =

C= == = 47.9 µF
( ) ( )( )( )( )

Answer: B. 47.9µF

45.A circuit which has wo= 10^6 rad/sec, C= 10pf and Q=100, must have a resistance

of ____kΩ

Solution:

f= ; Q=
√ √

√LC =

√ 𝐶 =

L= =( = 0.1
)( )
REGISTERED ELECTRICAL ENGINEERS PRE-BOARD EXAMINATION
AUGUST 2017
PROFESSIONAL ELECTRICAL ENGINEERING SUBJECTS

R=

=

Answer: C. 1

46.A coil of copper wire has a resistance a 5 46Ω at 75deg C What will be its

resistance at 25deg C.

Solution:

Copper = 234.5 [T(C)]


( )
= ( )

=4 Ω

Answer: D. 4.58 Ω
47.A coil of 50Ω resistance and 150m H inductance is connected in parallel with a 50µF

capacitor. The source voltage is 100 sin (wt + 30deg) v. What is the equation of the

line current?

Solution:

Hz

( )( )

( )( )

( )( )
REGISTERED ELECTRICAL ENGINEERS PRE-BOARD EXAMINATION
AUGUST 2017
PROFESSIONAL ELECTRICAL ENGINEERING SUBJECTS

Answer: C. 1.25 sin (wt + 75.5deg)


48. A lead-acid battery having a 100Ah capacity is 25% charged. After being charged at

10A for 5hours, It will be approximately _________ charged.

Solution:

( )

( )

Answer: B. 3/4
49.The current in a coil decays as follows; when t=0.002 sec, the current is 10 mA ;

when t=0.006 sec, the current drops to 4 mA. The time constant of the coil is

_______

Solution:

0.01= Io

Io = =

0.01 = 0.004
REGISTERED ELECTRICAL ENGINEERS PRE-BOARD EXAMINATION
AUGUST 2017
PROFESSIONAL ELECTRICAL ENGINEERING SUBJECTS

ln = ln

= ln (0.4) –

Tc = 4.36 ms

Answer: B. 4.36 ms

50. If the load current and flux of a dc motor are held constant voltage applied across its

armature is increased by 10%, its speed will ________________.

Answer: C. increase by about 10%

51.An abc phase sequence three phase voltage source connected in a balanced wye

has a line voltage of Vab= 208cis0° v rms. Determine the phase voltage Vbn.

Solution:

Vab = 208cis0

Vbc = 208cis-120

VL lags Vᴓ by 30°

Vbn = Vbc/√ cis30

= 208/√ cis30

= 120cis- 150

Answer: C. 120cis-150°v

52. A 15-hp motor operates at an efficiency of 87.5 percent at full-load. If the stray

power loss is approximately one-fourth of the total loss, calculate the copper loss.

Solution:

Pi = Pcu + SPL + Po

TL = Pcu + SPL

Pi = TL + Po
REGISTERED ELECTRICAL ENGINEERS PRE-BOARD EXAMINATION
AUGUST 2017
PROFESSIONAL ELECTRICAL ENGINEERING SUBJECTS

TL = Pi - Po = 1598.57

SPL = TL

therefore: Pcu = TL = ( 1598.57)

Pcu 1200 W

Answer: A. 1,200 W

53.A 4-pole shunt motor develops 20 lb-ftcf torque when the flux per pole is 700,000

maxwells. If the armature winding has 264 conductors and is wound wave, calculate

the total armature current.

Solution:

T=

= 9.55 N-m = 7.04 lb ft

1 N-m=

( )( )( )
Ia = ( )( )

Ia = 46.12A

Answer: C. 46 A

54. A dc shunt motor has a nameplate rating of 15HP, 230 v, 57.1 amperes, and 1,400

rpm The field circuit has a resistance of 115 and the armature circuit resistance is

0 13 Neglecting the effect if armature reaction, find the on-load line current.

Solution:

Shunt Motor:

Ic = Ia + If Eb = V2- IaRa

If = 230/115 = 2A = 230 – 55.1(0.13) = 222.837


REGISTERED ELECTRICAL ENGINEERS PRE-BOARD EXAMINATION
AUGUST 2017
PROFESSIONAL ELECTRICAL ENGINEERING SUBJECTS

Ia = 57.1 – 2 Pd = EaIa

= 55.1A = 12,278.32W

Po = 15 x 746 = 11,190W

Pin =Pd – Po = 1088.3187 W

Pm = EbIa

Pm = [ 230 – IaRa] [Ianl] ^2

1088.32 = 230Ianl -0.13Ianl^2

Ianl^2 -1769.23Ianl + 8371.64 = 0

Ianl1 = 1764. 48546 – Too High

Ianl2 = 4.74 – useful

Ilnl = Ianl + If

= 4.74 + 2 = 6.74A

Answer: C. 6.74 A

55. A capacitor of 0.1µF capacitance charged to a potential difference between plates of

100 v, Is discharged through a resistor of 1 megaohm. Calculate the initial value of

decay of capacitor voltage.

Solution:

Answer:

56.A 400-volt 50Hz star connected induction motor takes a line current of 50 amperes

at a p.f. of 0.8 lagging. Three mess connected capacitors are connected pf raise the

p.f. to 0.9. Find the value of each capacitor in microfarads.

Solution:

Q = P (Tan - Tan )

P = √ VL IL pf = √ (400)(50)(0.8)
REGISTERED ELECTRICAL ENGINEERS PRE-BOARD EXAMINATION
AUGUST 2017
PROFESSIONAL ELECTRICAL ENGINEERING SUBJECTS

= 27,712.8 W

=36.87

25.84

Q = 27,712.8 [ Tan36.87 – Tan 25.84]

= 7362.68VAR

Q/3= Xc = 2 fC

C = 7362.68 / 3(400)^2 (2pi)(50) = 48.9µF

Answer: B. 48.9

57. Two transformers are connected on open delta and deliver a balanced three-phase

load of 245 kW at 460 volts and power factor of 0.707. Calculate the power

delivered by the individual transformers

Solution:

= = 346534.65

= 200071.88 =

= cos-1 (.707) = 450°

cos-1 ( 45 + 30 )= 0.2588

= cos-1 ( 30 - 45 ) = 0.966

=S =S

= 193.2 KW = 51.8 KW

Answer: B. 193.2 kW, 51.8 kW

58.A 500 volts 1 Φ synchronous motor is taking 50 amperes at a p f of 0 8 leading The

output is 25 HP and the effective resistance of the armature is 0.25 ohm. Find the

iron and firction losses.


REGISTERED ELECTRICAL ENGINEERS PRE-BOARD EXAMINATION
AUGUST 2017
PROFESSIONAL ELECTRICAL ENGINEERING SUBJECTS

Solution:

Pin = PL + Po ; PL = Pin – Po

PL = 500 (50)(0.8) – 25 (746)

= 20,000 – 18,650

= 1350

PL = Pcu + Pco

Pcu =

= (0.25)

= 625

Pco = PL-Pcu

= 1350 – 625

= 725 W

Answer: D. 725 W

59. A two-pole three-phase 50 Hz star connected ac generator has 9 slots/pole and 6

conductors/slot. What is the flux per pole if the terminal voltage of open circuit is

1,100 volts. Take the coil span as unity.

Solution:

= = = 20 deg

 = slots per pole per phase

= =3

( )

d= = = 0.959

kp = sin (90 x pitch ) = sin (90 x1 ) = 1


REGISTERED ELECTRICAL ENGINEERS PRE-BOARD EXAMINATION
AUGUST 2017
PROFESSIONAL ELECTRICAL ENGINEERING SUBJECTS

Z= = 36

Ep = 2.22kpdf Z


( )( )( )( )

0.1756 Wb

Answer: A. 01756 Wb

60. A 500-KVA alternator operates at 80% of full-load with an efficiency of 92 percent. If

the load power factor is 0.8, calculate the total loss.

Solution:

 = Po / Po + PL
( )( )( )
PL =  - Po = – 0.8 (500x )(0.8)

PL = 27,826

PL = 27.83 KW

Answer: A. 27.83 kW

61. If there is an open circuit in the rotor of a JCIM,

Answer: C. motor will not start

62. In a synchronous machine when the rotor speed becomes more than the

synchronous speed during hunting, the damping bars develop______ torque.

Answer: C. induction generator


63.A 20 KV/7, 87KV auto transformer has a secondary line current of 100 A. What is

the current in the common winding?

Solution:
REGISTERED ELECTRICAL ENGINEERS PRE-BOARD EXAMINATION
AUGUST 2017
PROFESSIONAL ELECTRICAL ENGINEERING SUBJECTS

= =

x= 39.35

Icw = 100- 39.35

Icw = 60.65 A

Answer: B. 60.65 A

64. A 3Φ, 6-pole, 50 Hz induction motor develops 3.73 KW at 960 rpm. What will be the

stator input if the stator loss is 200 watts?

Solution:

Pd = 3.73 kw

Nr = 960
( )
Ns = = 1000

SPI = SCL + RPI

RPI = RCL + RPD =

RDD = ( ) = RPI (1-s)

RPI =

SPI =

=4165.4W

Answer: D. 4,165 W

65. A three –phase, wye-connected, 6.25 kVA, 220-volt synchronous generator has a

reactance of 8.4 ohms per phase. Using the rated kVA and voltage as base value,

determine the per unit reactance.


REGISTERED ELECTRICAL ENGINEERS PRE-BOARD EXAMINATION
AUGUST 2017
PROFESSIONAL ELECTRICAL ENGINEERING SUBJECTS

Solution:
(
= = = .626
√ ( )

Answer: D. 0.627

66.A two-pole alternator is running at 1,500 rpm, what is the angular velocity?

Solution:

1550 x x

= 157.1

Answer: C. 157.1 rad /sec

67. At leading p.f., the armature fkux in an alternator________ the rotor flux.

Answer: C. opposes

68. Synchronous motors are

Answer: B. not self-staring

69.A three-phase induction motor is taking 40kWm the slip being 4% and stator losses

amount to 800 watts, Calculate the total power developed.

Solution:

SPI = 40kW

S = 0.04

SCL = 800

RPI = SPI – SCL =39,201

RPD = RPI (1-s)

RPD = 37,632W

Answer: A. 37.63 kW
REGISTERED ELECTRICAL ENGINEERS PRE-BOARD EXAMINATION
AUGUST 2017
PROFESSIONAL ELECTRICAL ENGINEERING SUBJECTS

70.The per-unit reactance of a synchronous generator are <d = 1.1, Xd^i = 0.24, Xd^a

= 0.15. The Generator is operating without load at 5 percent above rated voltage

when a three-phase short circuit occurs at its terminals. What is the per unit

subtranient fault current?

Solution:

Answer:

71.Three 30.1 step-down transformers are connected Y - for stepping down the

132,000-volt three-phase transmission voltage. Calculate the secondary line

voltage.

Solution:


=a

√ ( )
Vs =

Vs = 7,621v

Answer: C. 7,621 v

72.A hydroelectric generating station is supplied form a reservoir of capacity 5 x 10^8

m^3 at a head of 200 meters. Find the total energy available in KWHR if the overall

efficiency is 75%

Solution:

 = Po / PN20 x 100

Po = PN20 = 0.75 (2.75x10^6 kWHr) = 2.044x10^6 kWHr

PN20 = QWH

=( )(9810 N/m^3) (200m)


REGISTERED ELECTRICAL ENGINEERS PRE-BOARD EXAMINATION
AUGUST 2017
PROFESSIONAL ELECTRICAL ENGINEERING SUBJECTS

=( ) N-m x

= kWHr

Answer: B. 2.044x10^6kWHr

73. This is the rating of power circuit breakers used to be the basis in replacing its

contacts.

Answer: C. Duty Rating

74.A uniform source of light gives 1,884 lumens and is suspended 4 m above the

center of floor 3 m square, Find the illumination in lm/m^2 at the floor level directly

below the lamp.

Solution:

E = I / s^2

I= = = 150

E= = 9.41lm/

Answer: A. 9.4

75.An amplifier has an output of 50 watts. What is its output in dBm?

Solution:

dBm=

= 47dBm

Answer: A. 47dBm

76. A consumer requires 2 x 10 ^6 KWHR per year and its yearly load factor is 30

percent. What is its maximum demand?


REGISTERED ELECTRICAL ENGINEERS PRE-BOARD EXAMINATION
AUGUST 2017
PROFESSIONAL ELECTRICAL ENGINEERING SUBJECTS

Solution:

MD = ( )

( )
= ( )

= 761 kW

Answer: D. 761 kW

77.If a half-wave rectifier is used 117-v rms ac (house mains), the average dc output

voltage is about ;

Solution:

√ ( ) = 165.46

= 0.318 (165.46)

=52.67V

Answer: A. 52.7 V

78. A short 3Ø transmission line with an impedance of 6 j8 per phase has

sending and receiving end voltages of 120 kv and 110 kv respectively for some

receiving end load of a p.f of 0.9 lagging. Determine the sending end power factor.

Solution:

R= 6 Ω

XL = 8 Ω

P.f = 0.9

ƟR = cos-1 0.9 = 0.451

VS = = 69.28 Kv

REGISTERED ELECTRICAL ENGINEERS PRE-BOARD EXAMINATION
AUGUST 2017
PROFESSIONAL ELECTRICAL ENGINEERING SUBJECTS

VR = = 63.51 Kv

VS = VR + I (R Cos ƟR XL Sin ƟR )

69.28 = 63.51 + I ( 6 X 0.9 + 8 X Sin 0.451)

I = 650. 3 A

Sending end P.f = (VR Cos ƟR + IR)/ VS


( )( ) ( )( )
=

= 0.88 Lag

Answer: A. 0.88 lag

79. A generating plant has a minimum and maximum outputs of 50 MW and 100 MW,

respectively. It generates an energy of 1,800,000 KWHR for 24 hours at average

power factor of 90%. What is the load factor of the plant?.

Solution:

Max Demand (MD) = 100 MW

Total Energy Produced (TEP) = 1,800,000 KWHr

Hours of Service (SH) = 24 Hours

LF = ( )( )

LF = 75 %

Answer: A. 75 %

80. A 75 MW steam power station uses coal of calorific value of 6,400 kcal/kg. Thermal

efficiency of the station is 30% while electrical efficiency is 80%. Calculate the coal

consumption per hour when the station is delivering its full output.

Solution:
REGISTERED ELECTRICAL ENGINEERS PRE-BOARD EXAMINATION
AUGUST 2017
PROFESSIONAL ELECTRICAL ENGINEERING SUBJECTS

noverall = nthermal x nelectrical

= 0.3 x 0.8

= 0.24

Units generated per hour : @ 1 KWHr = 860 Kcal

(75 x 10 3 ) x 1 = 75000 KWHr

Heat Produce / Hour = Electrical Output in Heat Units / noverall


( )

Coal Consumption / hour =

𝐶
𝐶

= 41,992 Kg x

= 42 tons

Answer: D. 42 Tons

81. An air core toroidal coil has a 120 turns and the length is 12 cm. What is the current

required to set up a flux of 1.2 x 10^5 weber? If the cross-sectional area is 12

square cm?

Solution:

Φ=

( )

Answer: B. 0.7960 A
REGISTERED ELECTRICAL ENGINEERS PRE-BOARD EXAMINATION
AUGUST 2017
PROFESSIONAL ELECTRICAL ENGINEERING SUBJECTS

82.A shunt motor with an armature resistance of 0.15 ohm is running 1,200 rpm for a

load which requires an armature current of 50 A form a 230 v source. At no load, the

armature current is 5 A. If the effect of armature reaction has reduced the air-gap

2% form no load to full load, what is the no load speed?

Solution:

Answer:

83. A 60 KV, three-phase short transmission line is 16 km long. The line has a per

phase series impedance of 0.125 + j0.4375 ohm per km. Determine the

transmission efficiency when the line delivers 70 MVA, 0.80 lagging power factor at

64 KV.

Solution:

Answer:

84. A copper conductor with a diameter of 0.10 inch has a resistance of 1 ohm per

1,000 ft. What is the resistance of 200 ft copper wire with a diameter of 40 mils?

Solution:

d1 = 0.10 in

d2 = 40 mils x = 0.04 in

( *

( )

2 2 Ω

nsw : 2 Ω

85. A three-phase 60 HZ transposed 230 KV transmission line has a horizontal

configuration of Dab = D and Dbc = D and Dac = 2D. The line reactance is 0.486
REGISTERED ELECTRICAL ENGINEERS PRE-BOARD EXAMINATION
AUGUST 2017
PROFESSIONAL ELECTRICAL ENGINEERING SUBJECTS

ohm per km. The conductor GMR is 2.0 cm. What is the phase spacing D?

Solution:

Answer:

86. A straight conductor 100 cm long and carrying 40 A lies perpendicular to a magnetic

field of 1.5 T (Wb/sq.m). What is the mechanical power in watts required to move

the conductor at a uniform speed of 5 meters per second?

Solution:

e=βlv

= 1.5 X 100 X5

= 7.5 V

P = e x I = 7.5 x 40

P = 300 W

Answer: D. 300 W

87. A 9.6 KW, 240 volts shunt generator having an armature resistance of 0.10 ohm and

a shunt field resistance of 240 ohms, delivers full load at rated voltage and 800 rpm.

The machine is now run as a motor while taking 250 v. Neglect the brush contact

drop, what is the speed of the motor?

Solution:

Answer:

88. A single-phase, 10-km transmission line has a 16.65 mH total inductance. If the

distance between the conductors is 1.0 m, what is the conductor radius?

Solution:

Answer:

89. A room measuring 16 m is to be illuminated by 8 lamps with an average illumination


REGISTERED ELECTRICAL ENGINEERS PRE-BOARD EXAMINATION
AUGUST 2017
PROFESSIONAL ELECTRICAL ENGINEERING SUBJECTS

of 72 lm/sq. m at floor level. If 40 percent of the total light emitted is absorbed by the

walls and ceiling, calculate the mean spherical candle power required for each lamp.

Solution:

Answer:

90. A 3-phase circuit 60 HZ, 230 kV transposed transmission line composed of two

1,272 MCM 54/19 conductors per phase with horizontal configuration. The bundle

conductors are 50 cm, In distance while phase spacing between the centers of the

bundle is 10 meters. If the GMR of the ACSR conductors is 0.0466 ft, find the

inductive reactance on ohm/km/phase of the transmission line?

Solution:

Answer:

91. The voltage applied across the resistor was reduced by 50%. The power consumed

by the resistor will be reduced by ________%.

Solution:

Therefore
( )

Answer: C. 25
REGISTERED ELECTRICAL ENGINEERS PRE-BOARD EXAMINATION
AUGUST 2017
PROFESSIONAL ELECTRICAL ENGINEERING SUBJECTS

92. A balanced delta connected load of 4 – j3 per phase is fed by 380 v, 3 – phase

system. Find the reactive power in VAR.

Solution:

( *( )

Answer: A. 52 KVAR

93. The effect of the dielectric is to

Answer: D. reduce the working voltage

94. A transmission line cable consists of 26 strands of identical copper conductor, each

1,255 mm in diameter. The length of the cable is 10km, but because of the twist of

the strands, the actual length of each conductor is increased by 5%. If the resistivity

of the copper is 1.72 x 10 raised to -8 ohm/meter, what is the resistance of the

cable in ohms?

Solution:

( )( )( )

( )
REGISTERED ELECTRICAL ENGINEERS PRE-BOARD EXAMINATION
AUGUST 2017
PROFESSIONAL ELECTRICAL ENGINEERING SUBJECTS

Answer: D. 5.62

95. Two lamps having luminous intensity of 250 and 300 candelas, respectively are

placed 100 m apart. Lamp A is placed 10 m above the ground while lamp B is

placed 20 m above the same level. What is the illumination at the center of the lamp

posts at ground level?

Solution:

Answer:

96. A short 230 kV transmission line has an impedance of 10 cis 80 The sending end

powers 150 MW at 230 kV and 82% power factor. What is the voltage at the other

end?

Solution:

Z = 5√ P = VL IL Cos Ɵ

P.f = 0.82 150 x IL (0.82)

Ɵ = 34.415 IL = 795 A

Sin Ɵ = 0 572

VS2 = (VR + IL Cos ƟR IX Sin ƟR)2 IX Cos ƟR – IL ƟR)2

VR = 227 KV

Answer: D. 225.45 KV

97. In a short circuit study, the positive, negative and zero sequence impedances are

0.25p.u., 0.25 p.u. and 0.3 p.u., respectively. The base MVA is 100. Determine the

fault current for a three phase fault at the 115 kV level.

Solution:
REGISTERED ELECTRICAL ENGINEERS PRE-BOARD EXAMINATION
AUGUST 2017
PROFESSIONAL ELECTRICAL ENGINEERING SUBJECTS

3 Phase Fault


( )

√ ( )
( )

Answer: A. 3000 A

98. Two single phase transformers each rated 75 kVA are connected in V or open-delta

to serve a three-phase load of 120 kW at 0.80 power factor lagging. To prevent the

overloading of transformers, determine the size of the capacitor in kVAR.

Solution:

Answer:

99. A 1 5 M resistor is connected in series with a 20 µF capacitor. The voltage source

is 120 volts dc. Determine the time at which the voltage across the capacitor is 75%

of the source voltage.

Solution:

Vc = E (1 - ) ; RC = 30 shift solve: t = 41.58 sec ≃ 41.60 sec

Answer:A. 41.6 sec

100.An induction motor with 8 poles, 60 Hz is operated with a slip of 3%. Calculate the

rotor speed.

Solution:
( )
P= = ;
REGISTERED ELECTRICAL ENGINEERS PRE-BOARD EXAMINATION
AUGUST 2017
PROFESSIONAL ELECTRICAL ENGINEERING SUBJECTS

%S =

0.03 =

P = 873

Answer: C. 873
MARCH 2018
REGISTERED ELECTRICAL ENGINEERS PRE-BOARD EXAMINATION
MARCH 2018
PROFESSIONAL ELECTRICAL ENGINEERING SUBJECTS

1. In Delta connected circuit, when one resistor is open, then power will be
A. 0 C. decreased by a factor of 3
B. increased by a factor of 3 D. remain unchanged
2. Which does not belong
A. lines per square centimeter C. flux
B. maxwells per square centimeter D. gauss

3. The diameter of a conductor is 0.064 inch. What is the area in circular mils?
A. 5,000 B. 10,000 C. 3,217 D. 4,096

Solution:

d=0.064inch

d= 0.064inch (1000mils/1inch) = 64mils

A = d² = 64² = 4096 CM

4. In a shunt motor, what is the relative connection of the armature winding with
respect to the field winding?
A series B. parallel-series C. series-parallel D. parallel
5. A series RLC circuit has R = 10 ohms , L = 40 x 10^-6 H and C = 60 x 10^-12
F. What is the resonant frequency of the circuit in MHZ?
A. 20.17 B. 3.24 C. 4.49 D 1.62
Solution:
R= 10 ohms

L= 40 x 10^-6 H

C= 60 x 10^-12 F

√ 𝐶 √( )( ) 𝒁

6. A capacitor charged with 10,000 uC. If the energy stored is 1 joule, find the
capacitance.
A 500 uF B. 5 uF C. 5,000 uF D. 50 Uf
REGISTERED ELECTRICAL ENGINEERS PRE-BOARD EXAMINATION
MARCH 2018
PROFESSIONAL ELECTRICAL ENGINEERING SUBJECTS

Solution:
Q= 10000 uC

W= 1 J

C=?

W = ½ Q^2 / C

C = ½ Q^2 / W = ½ (10000x10^-6)² / 1 = 50 x 10^-6 F or 50 uF

7. A 440-volt synchronous motor is taking 50 amperes at a pf of 0.9 lagging. The


effective resistance of armature is 0.8 ohm. If the iron and friction losses amount
to 600 watts, find the horsepower output.
A 25.7 В 37 1 C. 23.1 D. 40.5
Solution:
VL= 440v

IL= 50A

PF= 0.9

Ra= 0.8 ohms

Pfw= 600W

Pi = √3 VL IL PF = √3 440 50 0 9 =34 3 KW

Pcus= IL²Ra = 50²(0.8) =2000 W

Pout = 32.3 KW – 2000 W – 600 W = 31.7 KW ÷ 746 = 42.5 HP

8. What is considered as the most important value of a sine wave?


A. Effective value C. Average value
B. Peak value D. Instantaneous value
9. For a 3-phase load balanced condition, each phase has the same value of _
A. impedance B. resistance C. power factor D. all of these
10. Three 100 uF capacitors are connected in delta to a 500-volt, three-phase, 50
Hz supply. Find the line current.
REGISTERED ELECTRICAL ENGINEERS PRE-BOARD EXAMINATION
MARCH 2018
PROFESSIONAL ELECTRICAL ENGINEERING SUBJECTS

A 15.71 A B. 27.21 A C. 31.83 A D 8.66 A


Solution:
C= 100pF

VL= 500v, 50Hz


IL=?
Xc= 1/ 2πfC = 1/ 2π(50)(100uF)= 31.8
Iphase= 500/31.8 = 15.72A

IL= √3 Iphase = √3 15 72 = 27.23 A

11. Assuming the most efficient technologies for the extraction of energy, the fuel
with the largest known reserves is

A. Coal B. Oil C. Natural gas D. Nuclear

12. A three-phase 60 Hz line has flat horizontal spacing. The conductors have an
outside diameter of 3.28 cm with 12 m between conductors. Determine the
capacitive reactance of the line to neutral if its length is 125 miles.

^ Ω B. 202.4 x 10^3 Ω C. 25.3 x 10^3 Ω D. 16.2 x 10^3 Ω

Solution:
. Given : f = 60 Hz, D = 12m, d = 3.28 = .0328, r = .0164m

L = 125mi = 201.168 km

C = 8.138x10-12 F/m ( 1000m/ km )(201.168 km)

C = 1.637x10-6 F

Xc = 1/wC = 1/ 2πx60x1 637x10-6) = 1.6x103 Ω

13. A circuit has a resistance of 20 ohms and reactance of 30 ohms. What is the
p.f of the circuit ?

A. 0 B. 0.55 C. 0.832 D. 0.99

Solution:
Given: R = 20, X = 30

Z = R + jX
REGISTERED ELECTRICAL ENGINEERS PRE-BOARD EXAMINATION
MARCH 2018
PROFESSIONAL ELECTRICAL ENGINEERING SUBJECTS

Z = 20 j30 = 36 06∟56 31˚

Pf = cos(56.31) = 0.55

14. A capacitive load always has a __________ power factor

A. Leading B. Lagging C. Zero D. Unity

15. A road is illuminated by two 500 candela lamps place 6.1 m above the center
line of the road and 18.3 m apart. Find the illumination produced on the surface
of the road at a point on the center line midway between lamps

A. 2.29 lux B. 4.58 lux C. 9.16 lux D. 18.32 lux

Solution:
Given: I = 500

h1 = h2 = 6.1m, d = 18.3/2 m = 9.15 m

d2 = 6.12 + 9.152= 120.93

tanƟ = 9 15/ 6 1, Ɵ = 56 31

Ep 1= (I/d2 cosƟ
Ep1 =(500/120.93)cos(56.31) = 2.29 = Ep2
Eptotal = Ep1 + Ep2 = 2.29 + 2.29 = 4.58 lux

16. A magnetizing force of 1000 AT/m will produce a flux density of ___________
in air

A. 1.257 mWb/m^2 B. 0.63 mWb/m^2 C. 1.257 Wb/m^2 D. 0.63 Wb/m^2

Solution:
Given: H = 1000 AT/m

ᵦ = H x uo = 1000 4πx10-7) = 1.257x10-3 mWb/m^2

17. In hydro electric power plant, it is use to prevent the sudden rise and drop of
water pressure.

A. Penstock B. Intake tower C. Dam D. Surge tank

18. The basis where the flow of current is from positive to negative was derived
from __________
REGISTERED ELECTRICAL ENGINEERS PRE-BOARD EXAMINATION
MARCH 2018
PROFESSIONAL ELECTRICAL ENGINEERING SUBJECTS

A. Superstition B. Tradition C. Scientific D. Convention

19. The following are the given loads of a particular circuit


5 - 15 HP motors
3 - 5 HP motors
8 KW - lighting loads
3.5 KW - miscellaneous loads
Calculate for the maximum demand if the demand factor is 65%

A. 78.64 B. 45.81 C. 51.12 D. 120.28

Solution:
Given : 5 - 15 HP motors = 5( 15x746 ) = 55950
3 - 5 HP motors = 3( 5x746 ) = 11190
8 KW - lighting loads
3.5 KW - miscellaneous loads
DF = PFxTCL = (0.65)(55950 + 11190 + 8000 + 3500)/1000
DF = 51.12
20. By using two wattmeter method, power can be measured in

A. 3-phase, 2-wire B. 3-phase, 3-wire C. 3-phase, 4-wire D. All of these

21. The double energy transient occur in the


A. purely inductive circuit C. R-C Circuit
B. R-L Circuit D. R-L-C Circuit

22. Its primary source is to provide ground source


A. zig-zag transformer C. surge arrester
B. sectionalizer D. tertiary transformer

23. A 20 KV/7.87 KV autotransformer has a secondary line current of 100 A.


What is the current in the common winding?
A. 139.25 A B. 100 A C. 60.65 A D. 39.35 A
REGISTERED ELECTRICAL ENGINEERS PRE-BOARD EXAMINATION
MARCH 2018
PROFESSIONAL ELECTRICAL ENGINEERING SUBJECTS

Solution:

Given:

Find: Icb

24. In a star connected system, the current flowing through the line is
A. Greater than the phase current C. Lesser than the phase current
B. Equal to the phase current D. None of these

25. The capacitive reactance of a transmission line is 90,000 ohms per kilometer.
Find the total capacitive reactance if the transmission line is 50-km long.
A. 4.5 x 106 B. 1800 C. 90,000 D. 180,000

Solution:

26. Three phase induction motor is more suitable than single phase because
A It’s self-starting C. it has better power factor
B.it has better efficiency D. all of these

27. A 100 watts 230 volts gas fitted lamp has a mean spherical candlepower of
92 Find it’s efficiency in lumens per watt
A. 11.56 B. 1.156 C. 115.6 D. 1,156
REGISTERED ELECTRICAL ENGINEERS PRE-BOARD EXAMINATION
MARCH 2018
PROFESSIONAL ELECTRICAL ENGINEERING SUBJECTS

Solution:

( )

28. What is the speed of an induction motor of six poles if the percent slip is
2.5%?
A. 1462 rpm B.1170 rpm C.877 rpm D.1755 rpm

Solution:

Given:
Find: Nr

( )

( )

( )

29. Two (2), 1 ⌀, 25-KVA transformers are connected in open-delta bank.


Determine the maximum KW the bank can supply without overloading at 0.8 p.f
lagging.
A. 43.3 B. 34.64 C. 40 D. 50

Solution:

Given: ⌀

( )( ⌀)

( )( )
REGISTERED ELECTRICAL ENGINEERS PRE-BOARD EXAMINATION
MARCH 2018
PROFESSIONAL ELECTRICAL ENGINEERING SUBJECTS

( )

( )

30. The luminous intensity is defined as


A. lumen per square meter C.illumination per square meter
B.luminous flux per unit solid angle D. candela per unit solid angle

31. MHO relay is used for

A. rectifier B. circuit breaker C. transmission lines D. feeders

32. Three units of transformers /γ are connected to supply a 3-phase load from
a 400 volts, 3-phase source. What is the voltage on the load side?

A. 2,000v B. 3,464v C. 693v D. 1,200v

Solution:
VL= √3 (400)
=693V

33. In a serious motor, what is the relation of the connection of motor field
winding with the respect of the motor armature?

A. series B. parallel C. series-parallel D. delta

34. The field circuit of a 4-poledc generator has a resistance of 200Ω and an
inductance of 100H. If it is connected to a 400v supply, find the time taken for the
current to reach ljkjhggfdfd1.5A.

A. 0.125 sec B. 1.5 sec C. 0.369 sec D. 0.693 sec

Solution:
I= 1.5A, R= 200Ω, L= 100H, E= 400v, t=?

I= E/R(1-e^(R/L) t

1.5=400/200(1-e^-(200/100) t
REGISTERED ELECTRICAL ENGINEERS PRE-BOARD EXAMINATION
MARCH 2018
PROFESSIONAL ELECTRICAL ENGINEERING SUBJECTS

t= 0.693sec

35. Calculate the effective voltage in one phase of an alternator, given the
following particulars; f= 60 cps; turns per phase N = 240; flux per pole =
2.08x10^6.

A. 1,330v B. 2,304v C. 1,225v D. 2,122v

Solution:
f= 60 cps, N= 240rpm, = 2.08x10^6

E= (240) (2.08x10^6) =1324V ≠ 1330V

2 (60)

36. a 60 F capacitor is connected in series with 40,000 Ω resistor If the


capacitor is initially uncharged. Determine the resistor voltage when t= 1.5 times
the time constant for a suddenly applied source emf of 120volts.

A. 93.22v B. 120v C. 80v D. 26.78v

Solution:
I= E/R(1-e^(R/L) t

I= 120 = 3 x 10^-3

40,000

3x10^-3= E/40,000 (1-e^-(40,000/60) (1.5)

E= 120v

37. What is the force on a +5x10^-11 C charge that is 5 cm form each of two
+2x10^-9 C charge that are separated by 8cm?

A. 0.432x10^-6N B. zero C. 0.36x10^-6N D. 0.14x10^-6

Solution:
Q1= 5x10^-11, Q2= 2x10^-9, d= 0.05

5cm x 1m = 0.05m

100cm

F= KQ1Q2
REGISTERED ELECTRICAL ENGINEERS PRE-BOARD EXAMINATION
MARCH 2018
PROFESSIONAL ELECTRICAL ENGINEERING SUBJECTS

d^2

F= (9x10^9)(5x10^-11)(2x10^-9) = 0.36x10^-6

(0.05) ^2

38. A balanced delta connected load consist of 20 cis 25 Ω impedances The


60Hz line voltage 208v. What is the line current?

A. 6A B.10.4A C. 12A D. 18A

Solution:
R= 20∟25 Ω, V= 208V

IL= 208v = 10.4A

20Ω

39 The impedance of 69 kV transformer is 14 28Ω when referred to primary. The


transformer is a 10-MVA unit. What is the percent impedance of transformer?

A. 4 B. 6.2 C. 3 D. 4.5

Solution:
Zpu= (Zbase x Sbase)

Vbase^2

Zpu= (10 x 14.28) x 100 = 2.99≠3

69^2

40. What is the cost of operating a 70percent 20hp pump for 20 days, 8 hrs per
day? Cost of energy is P0.05/kW-hr.

A. P112 B.P170.45 C. P228.60 D. P83.55

Solution:
Pin= 20Hp, ɳ= .70, Cost of energy= P0.5kW/hr

Pin= Pout = 20x746 =21.31x10^3kW-hr

ɳ 0.70

=20days x 8hrs = 160hrs


REGISTERED ELECTRICAL ENGINEERS PRE-BOARD EXAMINATION
MARCH 2018
PROFESSIONAL ELECTRICAL ENGINEERING SUBJECTS

1day

= 160 x 0.5 = 80kW-hr

= 80 kW-hr (21.31 x 10^3)

= 170.45

41. The ratio of the wattmeter readings in three-phase inductive system is 2:1
when power is measured by the two-wattmeter method. What is the power factor
of the load?

A. 0.5 B. 0.866 C.0.75 D. 0.9

Solution:

Given: ratio: 2:1

Find: Pf=?

√ ( ) √ ( )

( )

42. A coil of 100 turns is cut by a magnetic field that increases at the rate of 40
Wb/min. What is the induced voltage?

A. 66.7 v B.16.7 v C. 4,000 v D. 6.67 v

Solution:

Given: N = 100 turns

Find: e =?

( )

43. Find the horsepower rating of a motor running at 1,500 rpm and developing a
torque of 105 lb-ft.
REGISTERED ELECTRICAL ENGINEERS PRE-BOARD EXAMINATION
MARCH 2018
PROFESSIONAL ELECTRICAL ENGINEERING SUBJECTS

A. 22 B. 28 C. 30 D. 36

Solution:

Given: N = 1,500 rpm

T = 105 lb-ft

Find: Hp=?
( )( )

44. The rms value of a sinusoidal wave is equal to _________ of maximum


value.

A. 0.637 B. 0.707 B. 0.506 D. 1.414

45. The voltage across a 50-µF capacitor rises at a constant rate of 10 V/ms.
Calculate the current through the capacitor.

A. 2 mA B. 5 mA C. 50 mA D. 500 mA

Solution:

Given: C =

Find: I=?

( ) (𝐶) ( )( )

46. One advantage of disturbing the winding in alternator is to

A. reduce harmonics C. reduce the amount of copper

B. improve voltage waveform D. decrease the value of the voltage

47. A series RLC circuit has R = 100 ohms, L = 100µH and C = 100 pF. What is
the frequency in MHZ at unity power factor?
REGISTERED ELECTRICAL ENGINEERS PRE-BOARD EXAMINATION
MARCH 2018
PROFESSIONAL ELECTRICAL ENGINEERING SUBJECTS

A. 1.59 B. 1.36 C. 15.9 D. 13.6

Solution:

Given:

Find: F=?

(√ ) (√( )( ))

48. In dc machines, lap winding is used for

A. high voltage low current C. high voltage high current

B. low voltage low current D. low voltage high current

49. The ratio of active power to apparent power is known as _______________


factor.

A. demand B. load C. power D. form

50. At ________________ frequencies, the parallel R-L circuit behaves as purely


resistive.

A. very low B. low C. high D. very high

51. One kilowatt equals to _ horsepower.


A. 1.26 B. 1.36 C. 1.46 D. 1.56

Solution:

52. The output of motor in watts when in takes a power of 3 kW and its efficiency
is 75%, is
A. 2000 B. 2250 C. 2500 D. 2750

Solution:
REGISTERED ELECTRICAL ENGINEERS PRE-BOARD EXAMINATION
MARCH 2018
PROFESSIONAL ELECTRICAL ENGINEERING SUBJECTS

Given:
Find:

53. In a certain Y-Y system, the source phase currents each have a magnitude of
9 A. The magnitude of each load current for a balanced load condition is
A. 12 A B.3 A C. 9 A D. 27 A

Solution:

Given:

54. In a three-phase system, the voltages are separated by


A. 90° B. 45° C. 120° D. 180°

Solution:

55. In a three-phase system, when the loads are perfectly balanced, the neutral
current is
A. one-third of maximum C. two-thirds of maximum
B. zero D. at maximum

56. When the motor runs on no load, then


A. Back emf is almost equal to applied voltage
B. Back emf will be greater than applied voltage
C. Back emf will be less than applied voltage
D. None of these

57. What is the source of heat generation in cables?


REGISTERED ELECTRICAL ENGINEERS PRE-BOARD EXAMINATION
MARCH 2018
PROFESSIONAL ELECTRICAL ENGINEERING SUBJECTS

A. Copper loss in conductor


B. Dielectric losses in cable insulation
C. Losses in metallic sheathings and armouring
D. All of these

58. In a 3 core cable, the capacitance between two conductors is 3 µF. What will
be the capacitance per phase?
A. 1.5 µF B. 3 µF C. 6 µF D. 12 µF

Solution:

Given: 𝐶
𝐶 𝐶

59. The process by which a heavy nucleus is splitted into two light nuclei is
known as
A. Splitting B. Fission C. Fusion D. Disintegration

60. Efficiency of a power plant is more in summers or winters?


A. summers B. winters C. same in both D. depends on variation

61. If there are eleven resistors of 33 kΩ each in parallel, what is the total
resistance?
A. 363 kΩ B. 3.3 kΩ C. Ω D. 16 5 kΩ

Solution:

Given:

Find:
REGISTERED ELECTRICAL ENGINEERS PRE-BOARD EXAMINATION
MARCH 2018
PROFESSIONAL ELECTRICAL ENGINEERING SUBJECTS

62. To reverse the direction of rotation of a three-phase induction motor, it is


necessary to
A. interchange all the three line connections
B. reverse the polarity of the rotor circuit
C. increase the resistance of the rotor circuit
D. interchange any two of the three line connections

63. The law that induced emf and current always opposes the cause producing
them was discovered by
A. Lenz B. Maxwell C. Faraday D. Ohm

64. The power factor of an alternator is determined by its


A. speed B. load C. excitation D. prime mover

65. A heater wire of length 50 cm and 1 mm 2 in cross-section carries a current of


2 A when connected across a 2 v battery. What is the resistivity of the wire?
A. 2 x 10-6 Ω-m B. 2 x 10-8 Ω-m C. 4 x 10-6 Ω-m D. 4 x 10-8 Ω-m

Solution:

Given:

( )

Find: ρ

( )( )

66. A six-pole three-phase 440-v, 60 Hz induction motor develops 10-hp at 1,150


rpm, the power factor being 80% lagging. Stator and core losses amount to
REGISTERED ELECTRICAL ENGINEERS PRE-BOARD EXAMINATION
MARCH 2018
PROFESSIONAL ELECTRICAL ENGINEERING SUBJECTS

400 w and 350 w, respectively. Frictional losses amount to 0.5-hp. Calculate the
motor line current.
A. 12.24 A B. 13.41 A C. 11.42 A D. 14.64 A

Solution:

Given:

Find:

( )
( )
( )( )

√ √ ( )( )

67. In a circuit containing R, L, and C, power loss can take place in


A. C only B. L only C. R only D. all of these

68. The efficiency of thermal power plant is approximately


A. 10% B. 30% C. 60% D. 80%

69. The time constant of a series RC circuit is given by


A. R/C B. RC2 C. RC D. R2C
REGISTERED ELECTRICAL ENGINEERS PRE-BOARD EXAMINATION
MARCH 2018
PROFESSIONAL ELECTRICAL ENGINEERING SUBJECTS

70. A power plant consumes 3,600 tons of coal per day. If the coal has an
average energy content of 10,000 BTU/lb, what is the plant’s power output?
Assume an overall efficiency of 15 percent.
A. 132 MW B. 130 MW C. 128 MW D. 126 MW

Solution:

Given:

Find:

( )

71. A resistance of 40 ohms and inductance of 79.6 mH are connected in parallel


across a 240 v, 60 Hz ac supply. Find the total current in amperes.
A. 11 B. 12 C. 10 D. 14

Solution:

Given:

Find:

( )( )
REGISTERED ELECTRICAL ENGINEERS PRE-BOARD EXAMINATION
MARCH 2018
PROFESSIONAL ELECTRICAL ENGINEERING SUBJECTS

√ √

72. A 230 v, 3-phase source supplies a balanced delta connected load with
impedance of 8-j6 ohms per phase. Calculate the line current in amperes.
A. 33.1 B. 39.8 C. 26.5 D. 29.3

Solution:

Given:

Find:

√ √ ( )

73. Three unbalanced 3-phase currents are given as follows:


Ia=10 -30° A, Ib=0, Ic=10 -150° A
A. 3.34 150° A B. 3.34 -150° A C. 3.34 30° A D. 3.34 -30° A

Solution:

Given:

| | | | | |

Find:
REGISTERED ELECTRICAL ENGINEERS PRE-BOARD EXAMINATION
MARCH 2018
PROFESSIONAL ELECTRICAL ENGINEERING SUBJECTS

( )

[( ) ( )( ) ( )( )]

74. A three-phase, 6 pole, 50 Hz, induction motor develops a maximum torque of


30 N-m at 960 rpm. Calculate the torque produced by the motor at 6% slip. The
rotor resistance per phase is 0 6 Ω
A. 24.7 N-m B. 25.7 N-m C. 26.7 N-m D. 27.7 N-m

Solution:

Given:

Find:

( )
( ) ( )

( )( )

( )
( ) ( )

( )
( ) ( )
REGISTERED ELECTRICAL ENGINEERS PRE-BOARD EXAMINATION
MARCH 2018
PROFESSIONAL ELECTRICAL ENGINEERING SUBJECTS

75. A device that repeatedly reverses the polarity of a magnetic field in order to
keep a dc motor rotating is known as
A. a solenoid B. an armature coil C. a commutator D. a field coil

76. The form factor is the ratio of


A. peak value to r.m.s value C. average value to r.m.s value
B. r.m.s value to average value D. none of these

77. The per unit impedance of a circuit element is 0.30. if the base KV and base
MVA are halved, then the new value of the per unit impedance of the circuit
element will be.
A. 0.30 B. 0.60 C. 0.0030 D. 0.0060

Solution:

Given:
⁄ ⁄

Find:

( )( )


( +( +

𝒁 𝒘

78. If two sinusoids of the same frequency but of different amplitudes and phase
angles are subtracted, the resultant is
A. a sinusoidal of the same frequency C. a sinusoidal of double the freq
B. a sinusoidal of half the original frequency D. not a sinusoid
REGISTERED ELECTRICAL ENGINEERS PRE-BOARD EXAMINATION
MARCH 2018
PROFESSIONAL ELECTRICAL ENGINEERING SUBJECTS

79. A 170-mile, 230 V, 60 Hz, three-phase single circuit transmission line uses a
triangular arrangement with 20 ft, 20 ft, and 36 ft spacing respectively. If the line
conductors have a radius of 0.0217 ft, determine the capacitive susceptance to
neutral per mile.
A. 4.81 x 10-8 mho per mile C. 7.92 x 10-6 mho per mile
B. 12.74 x 10-8 mho per mile D. 2.47 x 10-6 mho per mile

Solution:

Given:

√ √( )( )( )

( )
𝐶

𝐶 ( )

80. An 8-pole lap wound dc generator has 1,000 armature conductors, flux of 20
mWb per pole and emf generated is 400 v. What is the speed of the machine?
A. 1,000 rpm B. 1,800 rpm C. 1,200 rpm D. 1,500 rpm

Solution:

Given:

Find:

( )( )
REGISTERED ELECTRICAL ENGINEERS PRE-BOARD EXAMINATION
MARCH 2018
PROFESSIONAL ELECTRICAL ENGINEERING SUBJECTS

( )( )( )
( )( )( )

81. In a balanced 3-phase system, wye-connected voltage source with phase


sequence abc, Vab = 230 10o V. Find the phase voltage Vcn.
A. 230 -230oV B. 132.8 -80o V C.132.8 -140o V D. 132.8 -260o V

Solution:

Given:
Find:

82. A three-phase 60 Hz wye-connected wound rotor synchronous generator


rated at 10 kVA, 230V has a synchronous reactance of 1.2 ohm per phase and
an armature resistance of 0.5 ohm per phase. Calculate the percent regulation at
full-load with 0.8 lagging power factor.
A. 18.3% B. 20.8% C. 21.8% D. 22.5%

Solution:

Given:
Find:
REGISTERED ELECTRICAL ENGINEERS PRE-BOARD EXAMINATION
MARCH 2018
PROFESSIONAL ELECTRICAL ENGINEERING SUBJECTS

√( ) ( )

83. Two equally charged spheres repel each other with a force of 0.1kg. If their
centers are 20 cm apart, find the charge on each.
A. 2.09 x 10-6 coul B. 2.09 x 106 coul C. 2.00 x 10-6 coul D. 2.00 x 106 coul

Solution:

Given:
Find:

√ √

( )( )( )

84. The area of one plate of a two-plate mica capacitor is 0.0025 m2 and the
separation between plates is 0.02 m. If the dielectric constant of mica is 5, what
is the capacitance of the capacitor?
A. 5.53 µF B. 5.53 pF C. 7.74 µF D. 7.74 pF

Solution:

Given:
Find: 𝐶

𝐶
REGISTERED ELECTRICAL ENGINEERS PRE-BOARD EXAMINATION
MARCH 2018
PROFESSIONAL ELECTRICAL ENGINEERING SUBJECTS

( )( )( )
𝐶
𝐶

85. For coil having a span of 2/3 of pole pitch, the coil span factor is
A. 0.8 B. 0.866 C. 0.786 D. 0.966

Solution:

Given:
Find:

( )
( *

86. The number of parallel paths in a simplex lap winding is equal to


A. 2 B. number of poles C. number pair of poles D. 1

Solution:

Given:
Find:

( )

87. A 240 V DC series motor develops a shaft torque of 200 N-m at 92%
efficiency while running at 600rpm. Calculate the motor current.
A. 62.09 A B. 52.36 A C. 48.17 A D. 56.91 A

Solution:

Given:
Find:
REGISTERED ELECTRICAL ENGINEERS PRE-BOARD EXAMINATION
MARCH 2018
PROFESSIONAL ELECTRICAL ENGINEERING SUBJECTS

( )( )

88. A voltage of e = 70.7sin50t V is applied to a series circuit containing a


resistance of 4 ohms and an inductance of 60 mH. What is the impedance of the
circuit in ohms?
A. 5 30˚Ω B. 5 60˚Ω C. 5 53 13˚Ω D. 5 7˚Ω

Solution:

Given:
Find:

( )

89. An office, 17m by 8m, is lighted by 10 lamps and each lamp has a luminous
intensity of 200 cd. Allowing an absorption loss of 5,000 lm for the reflectors,
walls, and ceilings, calculate the average illumination on the working plane.
A. 177 lm/m2 B. 136 lm/m2 C. 118 lm/m2 D. 148 lm/m2

90. A 480-V rms line feeds two balanced three-phase loads.


If the two loads are rated as follows,
Load 1: 5 kVA at 0.8 p.f. lagging Load 2: 10 kVA at 0.9 p.f. lagging
Determine the magnitude of the line current from the 480-V rms source.
A. 17.97 A B. 14.94 A C. 13 A D. 7.36 A

Solution:
REGISTERED ELECTRICAL ENGINEERS PRE-BOARD EXAMINATION
MARCH 2018
PROFESSIONAL ELECTRICAL ENGINEERING SUBJECTS

Given:

Find:

√ ( )

91. A colliery workshop 20 m x 10 m requires an average illumination of 135


lm/m2 on the working plane. Calculate the electrical power and current required
for the purpose. Allow a coefficient of utilization of 0.45 and a depreciation factor
of 1.4. The supply is 230-V at 50 Hz and the efficiency of each of the lamps is 12
lm/W.
A. 8kW, 34.8 A B. 9kW, 39.1 A C. 6kW, 26.1 A D. 7kW, 30.4 A

Solution:

92 A series RL circuit has R = 6 Ω and XL = 8 Ω What is the circuit admittance?


A. 0.06 mho B. 0.1 mho C. 0.08 mho D. 0.125 mho

Solution:

Y = Z-1
Y = (6 + j8)-1
Y = 0.06 – j0.08 0.1 - 59.03
Y = 0.1 mho

93. A balanced 6,600-volt, 3-phase, 3-wire, 60 Hz transmission line is designed


to transmit power to a 450 kVA inductive load at 0.8 power factor. The line has a
resistance of 3.5 ohms and a reactance of 10.5 ohms per wire. Find the voltage
regulation of the line.
A. 7.6% B. 5.6% C. 9.6% D. 11.6%
REGISTERED ELECTRICAL ENGINEERS PRE-BOARD EXAMINATION
MARCH 2018
PROFESSIONAL ELECTRICAL ENGINEERING SUBJECTS

Solution:

94. A balanced Y-connected load of 300 j100 Ω s supplied by a 3-phase line


40 km long line with an impedance of 0 6 j0 7 Ω per km line to neutral Find
the voltage at the receiving end when the voltage at the sending end is 66 kV.
A. 60 kV B. 62 kV C. 58 kV D. 64 kV

Solution:

ZT = 300 j100 40 0 6 j0 7 = 324 j128 Ω

Is = = = 189.46 -21 56˚ = IR


( )

VR = IR ZT = (189.46 -21 56˚ (300 + j100)


VR = 59.9 kV -3 12˚
VR 60 kV

95. A plant has a total generating capacity of 800 kW. The coal consumption is
1,900 lbs. per hour. Heating value of the coal is 9,500 BTU per lb. What
approximate percent in heat in the coal is converted into useful energy?
A. 8.7 % B. 17.5 % C. 12.3 % D. 15%

Solution:

9,500 x 1900 = 18.05 x 106 x = 5288.6 kW (100%)

x = 15.13% 15%

96. A 100 km transmission line has a 1,200 ohms shunt reactance. What is the
shunt reactance per km?
A 1,200 Ω B 120,000 Ω 2Ω D 120 Ω

Solution:
REGISTERED ELECTRICAL ENGINEERS PRE-BOARD EXAMINATION
MARCH 2018
PROFESSIONAL ELECTRICAL ENGINEERING SUBJECTS

= 12

97. A relay is used to


A. receive a signal and record it C. connect a line to the source
B. protect a circuit D. relay a message to a remote place

98. A power plant consumes 100,000 lbs. of coal per hour. The heating value of
the coal is 12,000 BTU per lb. the overall plant efficiency is 30%. What is the kW
output of the plant?
A. 175,000 kW B. 142,500 kW C. 105,500 kW D. 205,000 Kw

Solution:

100 x 12,000 = 1.2 x = 351. 597 kW

Po = 105, 500 kW

99 The condition in Ohm’s law is that


A. ratio V/I should be constant
B. current should be proportional to voltage
C. the temperature should remain constant
D. the temperature should vary

100. How is the storage battery rated in capacity?


A. volts B. watts C. amperes D. ampere-hour
AUGUST 2018
REGISTERED ELECTRICAL ENGINEERS PRE-BOARD EXAMINATION
AUGUST 2018
PROFESSIONAL ELECTRICAL ENGINEERING SUBJECTS

1. In the transformers
A. high tension winding is used near the core
B. high tension winding is used away from the core
C. low tension winding is used near the core
D. low tension winding is used away from the core

2. Which of the following parameters can be neglected for a short line?


A. resistance B. inductance C. reactance D. capacitance

3. Find the frequency of a radio wave whose wavelength is 600m.


A. 500Hz B. 500kHz C. 150Hz D. 150kHz

Given: L= 600m Req’d: f

Solution:

L= v ; f = v = 3 x 105 m/s = 500kHz


f L 600m

4. The curve representing Ohm’s law is


A. a parabola B. linear C. a sine function D. a hyperbola

5. A series connected circuit has R= 4Ω and L=25mH Calculate the value of C that will
produce a quality factor of 50.
A. 62.5µF B. 6.25µF C. 0.625µF D. 25µF

Given: R= 4Ω Req’d: C

L=25mH

Qfactor = 50

Solution:

Q.F. = 1/ R (√L/C)
50 = ¼ (√25 x 10-3 / C)
C= 0.625µF

6. The overcurrent protective device is caused to operate by ___________________.


A. voltage surge B. current flow C. voltage drop D. impedance
REGISTERED ELECTRICAL ENGINEERS PRE-BOARD EXAMINATION
AUGUST 2018
PROFESSIONAL ELECTRICAL ENGINEERING SUBJECTS

7. What is the maximum number of bands permitted in rigid conduits between outlets?
A. 4 quarter bends C. 2 quarter bends
B. 6 quarter bends D. 3 quarter bends

8. Which of the following insulators will be selected for high voltage application?
A. strain type B. disc type C. suspension type D. pin type

9. At a 115kV substation, the PT ratio is 1000 and the CT ratio is 1200/5. The potential
going into the wattmeter is 115volts. What is the MW indicated when the wattmeter
reads 800 watts?
A. 150 MW B. 192 MW C. 15 MW D. 19.2 MW

Given: V=115kV Req’d: Pactual in MW


PTR=1,000
CTR=1200/5
VW= 115V
Prdg= 800W
Solution:

Pactual = Prdg (PTR x CTR)


= (800) (1000 x 1200/5)
= 192 x 106
= 192 MW

10. An inductive coil consumes active power of 500W and draws 10A from a 60Hz ac
supply of 110V. What is the coil inductance?
A. 26mH B. 28mH C. 30mH D. 32mH

Given: V= 110Vf = 60Hz Req’d: L


I = 10A
P= 500W
Solution:

R= P/ I2 = 500 / (10)2 = 5Ω
Z= V / I = 110 / 10 = 11Ω

Z= √R2 + XL2 L= XL / 2лf


XL= √Z2 – R2 = 9 78 / 2л 60
XL= √112 - 52 = 25.94 x 10-3
XL= 9 78Ω L=26mH
REGISTERED ELECTRICAL ENGINEERS PRE-BOARD EXAMINATION
AUGUST 2018
PROFESSIONAL ELECTRICAL ENGINEERING SUBJECTS

11. If the torque angle continuously increases, system will become


A. stable C. oscillatory stable
B. unstable D. asymptotic stable

12. The condition in Ohm’s Law is that


A. ratio V/I should be constant
B. current should be proportional to voltage
C. the temperature should remain constant
D. the temperature should vary

13. Which of the following shall be included in the calculation of the reliability standard
indices of the distributor asset by the Philippine Distributor Code?
A. Outages that occur on the secondary lines of the distribution system
B. Planned outages where the customers have been notified at least three(3)
days prior to the loss of power
C. Outages that occur on the primary lines of the distribution system
D. Outages that occur in the transmission line

14. If 25 ohms and 125 amperes are the base impedance and base current, respectively
for a given system, what is the base KVA?
A. 781.25 B. 39.1 C. 390.625 D. 76.12

Given: Zbase= 25Ω Req’d: kVAbase

Ibase = 125A
Solution:
kVAbase= Ibase2 x Zbase
= (125)2 (25)
kVAbase= 390.625

15. A type of electronics communication in which only one party transmits at a time.
B. Full duplex B. Half duplex C. Bicom D. Simplex

16. An equal number of electrons is placed on two metal spheres 3cm apart in air. How
many electrons are on each sphere if the resultant force is 4kN?
A. 1.47 x 1014 electrons C. 1.25 x 1014 electrons
B. 1.33 x 1014 electrons D. 1.40 x 1014 electrons
Given: d= 3cm Req’d: e
REGISTERED ELECTRICAL ENGINEERS PRE-BOARD EXAMINATION
AUGUST 2018
PROFESSIONAL ELECTRICAL ENGINEERING SUBJECTS

F= 4kN

Solution:

F= kQ1Q2 / d2 Qe = 1.6x10-19
4x103 = (9x109)(Q)2 / (3x10-2)2 e = 2x10-5 / 1.6x10-19
Q= 2x10-5 e = 1.25 x 1014 electrons

17. Base load plants are designed to supply power at


A. low capital cost and low operating cost
B. high capital cost and low operating cost
C. low capital cost and high operating cost
D. high capital cost and high operating cost

18. Peak load plants are designed to supply power at


A. low capital cost and low operating cost
B. high capital cost and low operating cost

C. low capital cost and high operating cost


D. high capital cost and high operating cost

19. A symmetrical three-phase 240V supplies a balanced delta connected load of 6-j8
ohms per phase. What is the line current?
A. -25 + j32A B. 25 – j5.78A C. -25 - j32A D. 25 + j32A

Given: V= 240V Req’d: IL


Z= (6-j8 Ω

Solution:

Iɸ = V / Z = 240 / 6-j8 = (14.4+j19.2) A


IL = Iɸ x √3 = (14.4+j19.2) (√3)
= 24.9+j33.3 A
IL = 25 + j32A

20. A 440V, 60Hz, 6 pole, 3 phase induction motor has a rotor input power of 33kW at
rated voltage and frequency. Under this condition, the rotor emf makes 126
complete cycles/minute. Find the rotor resistance in ohms/phase if the rotor current
is 60A/phase.
A. 0.0118 B.0.0107 C. 0.107 D. 0.118
REGISTERED ELECTRICAL ENGINEERS PRE-BOARD EXAMINATION
AUGUST 2018
PROFESSIONAL ELECTRICAL ENGINEERING SUBJECTS

21. If the effect of earth is taken into account, then the capacitance of line to ground
A. decreases C. remains unchanged
B. increases D. becomes infinite

22. The effective resistance of conductor is increased by


A. skin effect C. corona effect
B. proximity effect D. both (a) and (b)

23. Surge impedance of transmission line is given by


A. √C/L B √LC C 1/√LC D √L/

24. A 100 watts 230 volts gas fitted lamp has a mean spherical candle power of 92. Find
its efficiency in lumens per watt.
A. 9.56 B. 10.56 C. 11.56 D. 12.56

Given: P= 100W
I = 92mscp
V=230V
Solution:

n = Im / P = 4лI / P = 4л 92 / 100 = 11.56

25. The overload KVA of each transformer of an open-delta bank is 11.56. If each
transformer is rated 37.5 KVA, then what is the KVA of the balanced 3ɸ load?
A. 70 B. 80 C. 85 D. 90

Given: Req’d: Sload

S1ɸ = (37.5 + 11.55) KVA


Solution:

S1ɸ = Svv / √3 = Sload / √3


S1ɸ = Sload / √3
Sload = S1ɸ √3 = (37.5 + 11.55) √3 = 84 96 85

26. In a 6-pole alternator how many mechanical degrees is the equivalent of 180
electrical degrees?
A. 60 B. 30 C. 120 D. 90
REGISTERED ELECTRICAL ENGINEERS PRE-BOARD EXAMINATION
AUGUST 2018
PROFESSIONAL ELECTRICAL ENGINEERING SUBJECTS

Given: Req’d: Θm

P= 6
Θe= 180
Solution:

Θe= P / 2 Θm

Θm= Θe / (P / 2) = 180 / (6 / 2) = 60
27. A two terminal pair of network of a transmission line can be represented by a
A. Л- network B. T-network C. either (A) or (B) D. tree network

28. In television system, channels 2 and 13 are classified as


A. VHF B. UHF C. SHF D. EHF

29. An aluminum cable has 10 identical strands, a length of 1,500m and a diameter of
2.50 mm each. What is the resistance of the cable?
A. 86 3Ω B Ω C 56 3Ω D 0 563Ω

Given: Req’d: R

n = 10 identical strands
L = 1, 500m
d = 2.50mm = 2.5 x 10-3 m
ρ = 2 83 x 10-8 Ωm @ 20 0C

Solution:

R=ρ L/A

= ρ { L / л / 4 d2 n) }

= 2.83 x 10-8 { 1, 500 / л / 4 2 5 x 10-3 )2 x 10 ) }


R= Ω

30. An ACSR conductor has (n-1) layers around its single center strand, the total
number of strands will be
A. 3n2 + 3n +1 B. 3n2 - 3n -1 C. 3n2 + 3n -1 D. 3n2 - 3n +1
REGISTERED ELECTRICAL ENGINEERS PRE-BOARD EXAMINATION
AUGUST 2018
PROFESSIONAL ELECTRICAL ENGINEERING SUBJECTS

31. Three resistors A, B and C are connected in series to a 120V supply. If the the
resistor A=50 ohms and the voltage across resistor B=40V when the current is 1/2A,
what is the resistance of resistor C in ohms?
A. 105 B. 90 C.110 D.100

Given: Req’d: Rc

RA = 50Ω
VT = 120V
VB = 40V
I = 1/2A
Solution:

RB = VB / I = 40 / 1/2 = 80Ω

RT = VT / I = 120 / 1/2 = 240Ω

RT = RA + RB + Rc

240 = 50 + 80 + Rc

Rc = 240 - 50 – 80 = Ω

32. An inductor motor with 8 poles, 60Hz is operated with a slip of 3%. Calculate the
rotor speed.
A. 1,684 B.1,024 C. 873 D. 927

Given: Req’d: Nr

P=8 poles
f= 60Hz
%s= 3%
Solution:

Ns= 120f / p = 120(60) / 8 = 900rpm

%s = (Ns-Nr) / Ns
0.03 = (900- Nr) / 900
Nr = 900 – 0.03(900) = 873rpm

33. Which of the following methods of generating electric power from the sea water is
more advantageous?
REGISTERED ELECTRICAL ENGINEERS PRE-BOARD EXAMINATION
AUGUST 2018
PROFESSIONAL ELECTRICAL ENGINEERING SUBJECTS

A. water power B. ocean power C. tidal power D. none of these


34. When installing two grounding electrodes, the minimum distance between them is
A. 1,800 mm B. 2,000 mm C. 1,500 mm D. 2,400 mm

35 A 36 resistor is connected in parallel with an unknown resistor R Their


combination is them connected in series with a 12 resistor Find the value of R such
that the power drawn by the parallel combination of 36 and R is equal to the power in
the 12 resistor
A 6 B C 24 D 48

Solution:

P1 = Peq V1=V2

V1 = V1

R1 Req

R1 = Req

R1 = R (36)/R + 36

R = 18 Ω

36. The current flowing in L and C at parallel resonance are


A. zero B. equal C. infinite D. Difference

37. What does PEMC in EPIRA stands for


A. Philippine Energy Market Corporation
B. Private Energy Marketing Company
C. Philippine Energy Market Corporation
D. Philippine Electrical Marketing Corporation

38. For two alternators operating in parallel, if the load shared by one of the, to be
increased, as field excitation is
A. to be strengthened keeping input torque the same
B to be weakened keeping input torque the same
C. to be kept constant but input torque should be increased
D. to be kept constant but input torque should be decreased
REGISTERED ELECTRICAL ENGINEERS PRE-BOARD EXAMINATION
AUGUST 2018
PROFESSIONAL ELECTRICAL ENGINEERING SUBJECTS

39. A 7.5 MVA, 69/13.8 kV, 3-phase transformer has 8% impedance. What us the per
unit impedance at 10 MVA base?
A. 0.24 B. 0.106 C. 0.06 D. 0.096

Given: Zpuold = 0.08 pu

Sold = 7.5 MVA

Snew =10 MVA

Solution:

Zpunew = Zpuold (Zpuold / Sold)

= 0.08 ( 10 / 7.5 )

= 0.106 pu

40. Open circuit test on transformer is conducted to determine


A. hysteresis loss B. copper loss C. core loss D. eddy current loss

41. Which of the following power plant is a conventional source of energy?


A. Solar energy B. Geothermal energy C. Coal energy D. Wind energy

42. A transformer is rated 10kVA, 440/110V and 60 Hz when operated as conventional


2 winding transformer. This transformer is to be used as 550/440 V step-down
autotransformer in a distribution system. What is the new kVA rating of the transformer
in this manner?
A. 20 B. 10 C. 37.5 D. 50

43. In a synchronous motor, the magnitude of stator back emf depends on


A. load on the motor C. both the speed and rotor flux
B. dc excitation only D. none of theses

44 A dc ammeter has an internal resistance of 0 1 A shunt of 1 01 x 10 -3 is


connected to be ammeter. What is the multiplier of the set up?
A. 10 B. 50 C. 80 D. 100

Given: Rg = 0.1 Ω
Rsh = 1.01x10-3 Ω
REGISTERED ELECTRICAL ENGINEERS PRE-BOARD EXAMINATION
AUGUST 2018
PROFESSIONAL ELECTRICAL ENGINEERING SUBJECTS

Solution:
Multiplier = (Rg + Rsh) / Rsh
= (0.1 + 1.01x10-3) / 1.01x10-3
= 100
45. A single-phase, 10-km transmission line has 16.65 mH total inductance. If the
distance between the conductors is 1 m, what is the conductor radius?
A. 2 cm B. 1.5 cm C. 1 cm D. 0.75

Given : Lt = 16.65 mH

D=1

Solution:

Lt = 4x10-7 ln (d/ 0.7788r)

Lt = 16.65 mH x (1 / 10,000 m)

Lt = 1.665x10-6 H/m

1.665x10-6 = 4x10-7 ln (d/ 0.7788r)

r = 0.0199 m or 2 cm

46 4-pole wave wound generator has 220 coils of 10 turns each. The resistance of each
turn is 0 02 Find the resistance of armature winding.
B 22 C 02 D 0 44

Solution:

Ra = (220 coils)(10 turns x 0.02 Ω) / 4 Poles

Ra = 11 Ω

47. PDC in EPIRA means


A. Philippine Distribution Commission C. Philippine Distribution Corporation
B. Philippine Distribution Code D. Philippine Distribution Company

48. What is the maximum plug and cord rating of a 30 A branch circuit?
A. 24 A B. 30 A C. 26 A D. 28 A
REGISTERED ELECTRICAL ENGINEERS PRE-BOARD EXAMINATION
AUGUST 2018
PROFESSIONAL ELECTRICAL ENGINEERING SUBJECTS

Solution:

Rating = 80 % of OCPD

= 0.8 ( 30 A )

Rating = 24 A
49. Wavelength is the distance travelled by an electronic wave during the time of one
cycle. Given a wavelength of 12 meters, what is the frequency?
A. 250 kHz B. 25 kHz C. 250 MHz D. 25 MHz

Given: λ = 12 m

Electronic wave = 3x108 m/sec.

Solution:

λ = V/f

f = (3x108) / 12

f = 25 MHz

50. Alternators installed in a hydro power station, are of


A. low speed B. high speed C. medium speed D. very high speed

51. A sine wave has a frequency of 60 Hz. Its angular frequency is


A. 120 π B 60 π C 30 π D 6π

Solution:

f= 60 Hz

ω= 2π (60)

ω 2 π

52. In a loss-free RLC circuit, the transient current is


A. oscillating B. Square wave C. sinusoidal D. non-oscillating

53. What is the maximum flux in mWb of a transformer with 200 primary turns when
connected to a 60 Hz, 200 V supply?
REGISTERED ELECTRICAL ENGINEERS PRE-BOARD EXAMINATION
AUGUST 2018
PROFESSIONAL ELECTRICAL ENGINEERING SUBJECTS

A. 10.35 B. 6.25 C. 50.5 D. 3.75

Given: N= 200 turns

f= 60 Hz

E= 200V

⌀= ?
Solution:

E= 4.44 Nf⌀

⌀ = E / 4.44Nf
⌀= 200 / (4.44x200x60)
⌀= 3.75 mWb
54. A 60 Hz, 4 pole, 3 phase induction motor has a slip of 2.5 %. What is the slip speed
in rpm ?
A. 55 B. 40 C. 60 D. 45

Given: f = 60 Hz

P=4

S= 2.5%

NSLIP =?

Solution:

Ns = 120f/ P

Ns = 120(60) / 4

Ns = 1800 rpm

Nr = Ns (1-s)

= 1800 (1 – 0.025)
REGISTERED ELECTRICAL ENGINEERS PRE-BOARD EXAMINATION
AUGUST 2018
PROFESSIONAL ELECTRICAL ENGINEERING SUBJECTS

Nr = 1755 rpm

NSLIP = Ns-Nr

NSLIP= 1800-1755

NSLIP = 45 rpm

55. A 15 hp, 460 V, 60 Hz, 6-pole, 3 phase induction motor has full-load slip of 4%. Find
the full-load torque in N-m.
A. 85 B. 88 C. 93 D. 90
Given: POUT = 15 HP

f = 60 Hz

P = 6 Poles

S=4%

T=?

Solution:

HP = 2πNrT / 44,760

Nr = (120f/P) / (1-s)

= ((120x60)/6) / (1-0.04)

Nr = 1152 rpm

T = HP (44,760)/ 2πNr

= (15) ( 44,760)/ 2π(1152)

T= 92.76 N-m or 93 N-m

56. At very low frequencies a series RC circuit behaves as almost purely


A. resistive B. inductive C. capacitive D. none of these
57. A power plant gets water from a dam from a height of 122.45 m at the rate of 1,000
cubic meters per minutes. If the output of the plant is 15,000 kW, what is the plant
efficiency?
A. 80% B. 75% C. 70% D. 65%
REGISTERED ELECTRICAL ENGINEERS PRE-BOARD EXAMINATION
AUGUST 2018
PROFESSIONAL ELECTRICAL ENGINEERING SUBJECTS

Given: h = 122.45 m

Q = 1000m3 / min. x (1 min/ 60 sec.) = 16.67 m3 / sec.

Pout = 15,000 kW or 15 MW

Solution:

Pin = Qwh

Specific weight of water W = 9810 N/m3

Pin = (16.67)(9810)(122.45)

= 20 MW

n = (Pout / Pin) x 100

= (15 MW / 20 MW) x 100

n = 75 %

58. A 500 kVA, 34.5/13.8 kV, 3-phase transformer is delta-wye connected. The primary
and secondary winding resistances are 35 ohms and 0.876 ohm per phase,
respectively. Calculate the percent efficiency at full-load and unity power factor id core
loss is 3 kW.
A. 97.8 B. 98.7 C. 96.4 D. 97.2

Given: S = 500 kVA rp = 35 ohms

34.5/13.8 kVA rs = 0.876 ohms

Pcore = 3 kW

n=?

Solution:

Refer to secondary side

Rs = rs + rpa2

= 0.876 + 35 ((13.8/ √3)/34.5)2

= 2.743 ohms
REGISTERED ELECTRICAL ENGINEERS PRE-BOARD EXAMINATION
AUGUST 2018
PROFESSIONAL ELECTRICAL ENGINEERING SUBJECTS

Is = S / √3 Vs)

= 500x103 / √3x13.8103 )

= 20.92

@ Unity pf

Po = S = 500kW

Pcu = 3Is3Rs

= 3 (20.92)2 (2.743)

= 3601.39 W

n = (Po/Pi)x100 = (Po/Po+Pcu+Pcore)x100

= 500x103 / (500x103 + 3x103 + 3601.39)

n = 98.7 %

59. One of the advantages of distributing the winding in alternators is to


A. reduce harmonics C. improve voltage waveform
B. reduce the amount of copper D. decrease the value of the voltage

60. The effect of corona is


A. increased reactance C. Increased energy loss
B. increased inductance D. all of these

61. The rms value of pure cosine function is


A. 0.5 of peak value C. same as peak value
B. 0.707 of peak value D. zero

62. Calculate the use factor of a power plant if the capacity factor is 35% as it operates
8,000 hours during the year.
A. 38.32% B. 33.82% C. 36.82% D. 32.54%

63. How many commutator bars does a 6-pole dc generator with 4 coil elements per slot
and 20 slots per pole has?
A. 60 B. 120 C. 240 D. 480
REGISTERED ELECTRICAL ENGINEERS PRE-BOARD EXAMINATION
AUGUST 2018
PROFESSIONAL ELECTRICAL ENGINEERING SUBJECTS

Solution:

Z = (4 Coil/ Slot x 20 Slot/Pole) ( 6 Pole )

= 480 Coils

Nc = Z/2

= 480 / 2

Nc = 240 Coils

64. A coil having a resistance of 10 and an inductance of 4 H is switched across a 20


V dc source. Calculate the rate of current change when the current reaches 50% of its
final steady value.
A. 2.5 A/s B. 3.5 A/s C. 4.5 A/s D. 5.5 A/s

Solution:

XL = 2π(60)(4)

= 480π

ISV = V/Z

IFSV (50%) = 20 / (√102 + 480π2)

IFSV = 0.026 (100)

IFSV = 2.65 A or 2.5 A/s

65. Calculate the torque in N-m developed by a 440-V dc motor having an armature
resistance of 0 25 and running at 750 rpm when taking a current of 60 A
A. 240 B. 325 C. 285 D. 342

Given: V = 440 N = 750 rpm


Ra = 0.25 IL = 60 A
Solution:

Eg = VL – IaRa

= 440 – (60)(0.25)
REGISTERED ELECTRICAL ENGINEERS PRE-BOARD EXAMINATION
AUGUST 2018
PROFESSIONAL ELECTRICAL ENGINEERING SUBJECTS

= 425 V

Pd = (425)(60)

= 25,500 N

HP = 25,500/746 = 34.182 HP

HP = 2πNrT / 44,760

34.182 = (2πx750xT / 44760

T = 324.67 or 325 T-N

66. In a 13.8 kV, 10 MVA base three-phase system, a phase to phase fault occurs. If
the Thevenin’s equivalent impedance of the system is 2 63%, what is the per unit fault
current?
A. 19.01 pu B. 32.93 pu C. 57.03 pu D. 98.78 pu
Solution:

ILL = √3/2 10 MVA/ √3 13 8 kV 0 0263

ILL = 13776.38

Ipu = I FAULT/ I BASE

= 13776.38 / ( ( 10MVA/ √3 13 8kVA

= 32. 93 pu

67. A single-phase transmission line having a length of 15 miles is composed of two


copper conductors spaced 7ft apart having a diameter of 0.0575inch. Determine the line
to line capacitance in farad if the system frequency is 60Hz.
A. 4.9 x10-12 B. 9.8 x10-12 C. 11.8 x10-9 D. 118 x10-9

68. What is the equivalent IEEE device function number for directional power relay?
A. 21 B. 32 C. 37 D. 87

69. Energy radiated continuously in the form of lighting waves is called?


A. luminous intensity B. lumen C.luminousflux D. illumination
REGISTERED ELECTRICAL ENGINEERS PRE-BOARD EXAMINATION
AUGUST 2018
PROFESSIONAL ELECTRICAL ENGINEERING SUBJECTS

70. Efficiency is secondary consideration in case of


A. base load plants C. both base load and peak plants
B. peak load plants D. none of these

71. A capacitor is charged with 10,000 µC. If the energy stored is 1 joule. Find the
capacitance
A. 5µF B. 50µF C. 500µF D. 5,000µF
Solution :

W= 1/2 (Q2/C)

1j = 1/2 ( (10,000 x10-6)2/C)

C= 50µF
72. To a series RLC circuit, a voltage of 10 v is applied. if Q of the coil at resonant
frequency is 20., the voltage across the inductor at resonant frequency will be
A. 200 v B. 100 v C. 75 v D. 50 v
Solution:

Q-factor = EL / Ep

EL = ( Qfactor ) ( Ep )

= ( 20 ) (10 )

EL = 200 V

73. A 75 KVA, 13,200/240, 60Hz, single-phase transformer has 1% resistance and 3%


reactance. Find the copper loss of the transformer at 3/4 of full-load and unity power
factor.
A.155 W B. 422 W C. 368 W D. 508 W
74. The starting current of a 15 hp, 460 v, 3 phase induction motor is 130 A. What
voltage in volts should be applied to the motor so that the starting current will not
exceed 84.8 A.
REGISTERED ELECTRICAL ENGINEERS PRE-BOARD EXAMINATION
AUGUST 2018
PROFESSIONAL ELECTRICAL ENGINEERING SUBJECTS

A. 238 B. 295 C. 300 D. 310

75. The condition of a liquid electrolyte is measured in terms of its


A. current value B. specific gravity C.acid content D.voltage output

76. If the of a dc shunt motor is opened


A. it will continue to run at its rated speed
B.the speed of the motor will become very high
C.The motor will stop
D.the speed of the motor will decrease

77. An ideal full-wave rectifier with non-inductive load has zero transformation losses.
What is the efficiency?
A. 70% B. 80% C. 90% D. 100%
78. In an ACB sequence balanced 3 phase system, the angle between the line voltage
and the line current is 50°, what is the lagging power factor of the system?
A. 0.30 B. 0.50 C. 0.64 D. 0.94
79. A power station supplies 60 KW to the load over 2,500 ft of 000 2-conductor copper
feeder the resistance of which is 0.078 ohm per 1,000 ft. The bus-bar voltage is
maintained constant at 600 volts. Determine the maximum power which can be
transmitted.
A. 60 KW B. 230.7 KW C. 120 KW D. 150 KW

80. IF a certain conductor has an area of 336.400 circular mils, what is the radius of this
conductor in cm?
A. 0.0663 B. 0.810 C. 0.737 D. 0.921
Solution:

Acm = d2 d = 2r r = [290 x (1/100 in)] [2.54 cm/1in]

d2 = 336,400 r=d/2 r = 0.737


REGISTERED ELECTRICAL ENGINEERS PRE-BOARD EXAMINATION
AUGUST 2018
PROFESSIONAL ELECTRICAL ENGINEERING SUBJECTS

d = 580 r = 580 / 2

81. In laboratory experiment, the impedance of a coil was obtained at 60Hz and 30Hz.
These are 75,48 ohms and 57.44 ohms , respectively . What is the inductance of the
coil?
A. 150 mH B. 182.5 mH C. 42.5 mH D. 2.1 mH
Solution:

Z2 = R2 + XL2 XL = 2π fL

60 Hz , ( 75.48 )2 = R2 2π 60 L 2
-------- eq. 1

30 Hz , ( 57.44 )2 = R2 2π 30 L 2
-------- eq. 2

Solve 2 equations

L = 150 mH

82. The dummy coil in dc machines is used to


A. eliminate reactance voltage
B. eliminate armature reaction
C. bring about mechanical balance of armature
D. eliminate harmonics developed in the machine

83. A half wave rectified sine wave has a averange value of 100 amperes. What is its
effective value ?
A. 157 A B. 70.71 A C. 100 A D. 141.4 A
Solution:

Half wave , Vave = 0.318 Vmax Vrms = 0.5 Vmax

100 = 0.318 Vmax Vrms = (0.5)(314.465)

Vmax = 100 / 0. 318 Vrms = 157.23 A

Vmax = 314.465 v
REGISTERED ELECTRICAL ENGINEERS PRE-BOARD EXAMINATION
AUGUST 2018
PROFESSIONAL ELECTRICAL ENGINEERING SUBJECTS

84. A 3-phase transmission line has two aluminum conductors in bundle. The self GMD
of each conductor is 8.90 mm and the distance between the conductors and phase is
40cm and 9m, respectively. What is the inductance of the line per phase ?
A. 1 µH/m B. 2 µH/m C. 3 µH/m D. 4 µH/m
Solution:

L = 2x10-7 ln ( Dm / Ds )

Dm = ? d1 = 40 cm = 0.4 m

Ds = 8.9 mm d2 = 9 m

for bundled conductor

Dm = Dmxy = squareroot d1d2

= (0.4) (9) = 1.879m x (1000mm / 1m) = 1879mm

L = 2x10-7 ln (1879 / 8.9)

L = 1.07x10-6 H / m

L = 1.07 µH/m
85. The purpose of insulating oil when used on power circuit breaker are the following.
EXPECT:
A. Interrupter B. Coolant C. Insulation D. Quencher
86. A wind generator with an efficiency of 80% has a blade diameter of 20 m. If the wind
velocity is 30 Km/hr, how much power in KW is obtained from the generator?
A. 58.47 B. 45.78 C. 48.75 D. 54.87
Solution:

For wind plant

Pin = 0.2962 D2 V3 P = Watts

D = Blade diameter (m) = 20m V = wind velocity ( m/s )

V = ( 30 Km / hr ) ( 1 hr / 3600 sec ) ( 1000 / 1 km )

V = ( 25/3)m/s
REGISTERED ELECTRICAL ENGINEERS PRE-BOARD EXAMINATION
AUGUST 2018
PROFESSIONAL ELECTRICAL ENGINEERING SUBJECTS

Pin = ( 0.2962 ) ( 20 )2 ( 25/3 )

Pin = 68.56 KW

PRating = n ( Pin )

= 0.8 ( 68.56 )

PRating = 54.85 KW
87. The field system of a 50 Hz alternator has a sinusoidal flux per pole of 0.1 Wb .
Calculate the emf generated in one turn which span two-thirds of a pole pitch.
A. 17.4 v B. 19.2 v C. 12.2 v D. 9.6 v
Solution:

E = 4.44 Kd Kp f Φ N

Kd = 1 ( since not stated / assumed )

Kp = sin ( 90deg x pitch )

Kp = sin ( 90 x 2/3 )

Kp = 0.866

E = (4.44)(1)(0.866)(50)

E = 19. 2 V
88. A dc shunt motor develops 15 hp at 120 v . If the effective armature resistance is
0.061 ohm and the field winding draws 2 amperes, what is the overall efficiency?
A. 90% B. 93% C. 94% D. 95%
Solution:

Pd = 15hp x 746 = 11.19 KW PspL = 0 since not given

Pd = Pout + PspL PLoss = Pcu + PspL

= 11.19 + 0 PLoss = Ia2 Ra + Vsh Ish

Pd = 11.19 KW PLoss = (98.15)2 (0.061) + (120)(2)

PLoss = 827.60 W
REGISTERED ELECTRICAL ENGINEERS PRE-BOARD EXAMINATION
AUGUST 2018
PROFESSIONAL ELECTRICAL ENGINEERING SUBJECTS

η= ( Pout / Pin ) = ( Pout / Pout + PLoss )


P d = E b Ia , E b = V T - Ia

11.19x103 = ( 120 - Ia(0.061)) ( Ia )

Ia = 98.15 A

η= ( 11.19 KW / ( 11.19 KW + 0.8276 KW ))


η= 93.11 % = 93%

89. An 8-pole triplex wave-wound armature carries a total of 660 amperes. If there are 8
brush arms, calculate the current in each armature conductor.
A. 110 A B. 66 A C. 22 A D. 11 A
Solution:

Ia = 660 A

a = 2m

a = (2) (3)

Ia/conductor = ( Ia / a ) = 660/6 = 110 A

90. Inside a hollow spherical conductor


A. electric field is zero
B. electric field is constant
C. electric field changes with distance from the center of the sphere
D. electric field is unity

91. Residual magnetism is necessary in a


A. separately excited generator C. both of these
B. self-excited generator D. none of these
92. Three impedance Zab=4-j3 ohms, Zbc=3+j4 ohms and Zac=10+j0 ohms are
connected in delta across a 220 volts, three phase balanced source. What is the total
power of the circuit?
REGISTERED ELECTRICAL ENGINEERS PRE-BOARD EXAMINATION
AUGUST 2018
PROFESSIONAL ELECTRICAL ENGINEERING SUBJECTS

A. 1,839 W B. 1,836 W C. 6,130 W D. 18,392 W


Solution:

PT = ? assume ABC sequence

PT = Iab2 ( Rab ) + Ibc2 ( Rbc ) + Ica2 (Rca)

Iab = | ( Vab / Zab ) | = | (220<0) / (4-j3) | = 44 A

Ibc = | ( Vbc / Zbc ) | = | (220<-120) / (3+j4) | = 44 A

Ica = | ( Vca / Zca ) | = | (220<120) / (10+j0) | = 44 A

PT = (44)2 ( 4 ) + (44)2 (3) + (22)2 (10)

PT = 18,392 W

93. A light source located 2.75 m from a surface produces an illumination of 528 lux on
the surface.Find the illumination if the distance is chaged to 1.55m.
A. 298 lux B. 1,662 lux C. 937 lux D. 168 lux
Solution:

E = K / d2

E1 d 1 = E 2 d 2

E2 = E1 d12

E2 = E1 d12 = (5.28)(2.75)2 / 1.552

E2 = 1662 lux

94. In case of delta connected circuit, when one resistor is open, power will be
A. unaltered B. reduced to 1/9 C. reduced to 1/16 D. reduced by 1/3

95. Two similar poles, each of 1 Wb, placed 1m apart in ar will experience a replusive
force of_____.
A. 63,000 N B. 796 N C. 8 x1012 N D. 63 x10-3 N

Solution:
REGISTERED ELECTRICAL ENGINEERS PRE-BOARD EXAMINATION
AUGUST 2018
PROFESSIONAL ELECTRICAL ENGINEERING SUBJECTS

F = Km1m2 / d2

for MKS

K = 1 / 4πµoµr

K = 1/4π 4π x10-7 ) (1)

K = 63325.74

F = 63325.74 (1)(1) / 12

F = 63 ,328 N = 63 000 N

96. In a Transformer, the core loss is found to be 60 watts at 30 Hz and 90 watts at


40Hz measured at the same maximum flux density. Compute the eddy current and
hysteresis losses at 60 Hz in watts.
A. 90,75 B. 85,60 C. 95,69 D. 88,72
Solution:

Pcore = Phrs + PEddy

@30Hz

60 = Phrs1 + PEddy1

@40Hz

with respect to frequency change

P = K f2 ---- eddy

K = PEddy1 / f12 = PEddy2 / f22 = PEddy3 / f32

Phrs = K f2

( PEddy1 / PEddy2 ) = ( f1 / f2 )2

( PEddy1 / PEddy2 ) = ( 30 /40 )2

PEddy1 = 9/16 PEddy2

( Phrs1 / Phrs2 ) = ( 30 /40 )

Phrs1 = 3/4 Phrs2


REGISTERED ELECTRICAL ENGINEERS PRE-BOARD EXAMINATION
AUGUST 2018
PROFESSIONAL ELECTRICAL ENGINEERING SUBJECTS

90 = Phrs2 + PEddy2
60 = Phrs1 + PEddy1
60 = (9/16)PEddy2 + (3/4) Phrs2 -------- eq. 1
90 = Phrs2 + PEddy2 -------- eq. 2
PEddy2 = 40
Phrs2 = 50
( PEddy3 / PEddy2 ) = ( f3 / f2 )2 = ( 60 / 40 )2
PEddy3 = ( 40 ) ( 60 / 40 )2
PEddy3 = 90 W --------@60Hz
( Phrs3 / Phrs2 ) = ( f3 / f2 ) = ( 60 / 40 )
Phrs3 = ( 50 ) ( 60 / 40 )
Phrs3 = 75 W ---------60Hz
97. Transient current in an RLC circuit is oscillatory when
A. R=0 B. R= 2√L/C C. R>2√L/C D. R<2√L/C

98. The current from neutral to ground connection is 12 A. What is the zero sequence
component in phase?
A. 12 A B. 36 A C. 4 A D. 3 A
Solution:

In = 12

In = Ia + Ib + Ic

from fortescues theorem

Ico = Ibo = Iao = 1/3 ( Ia + Ib + Ic )

= 1/3 ( In )

= ( 1/3 ) ( 12 )

= 12/3
REGISTERED ELECTRICAL ENGINEERS PRE-BOARD EXAMINATION
AUGUST 2018
PROFESSIONAL ELECTRICAL ENGINEERING SUBJECTS

Iao = 4 A
99. With reference to armature windings, wave windings, wave windings are often
called_______ windings.
A. Cascade B. Series C. parallel D. ring

100. This is the compact arrangement of switching devices.


A. Circuit protector B. Switchboard C. Switchgear D. Panelboard

Você também pode gostar